You are on page 1of 58

Chapter 7: Cost of Capital

Introduction
“Cost of capital is the minimum rate of return expected by an investor on his investment.”
OR
“Cost of capital is the expense incurred by a firm to use a particular fund.” E.g., interest paid on the
debt, dividend paid on preference shares, etc.
In general, the concept of cost of capital is applicable to the long-term funds.

The Importance of the concept of cost of capital


The importance of the concept of cost of capital can be seen under the following heads—
1. Helpful in capital budgeting process: Cost of capital is quite helpful in capital budgeting
process. It works as basis for decisions. E.g., in discounted cash flow methods, the future
cash inflows of a project are discounted by this rate.
2. Helpful in capital structure decisions: Cost of capital is helpful in capital structure
decisions. When the management of the firm has to decide the optimum capital structure for
the company then cost of capital should be minimized and the value of the firm should be
maximized.
3. Helpful in comparative analysis of various sources of finance: Cost of capital is helpful
in comparative analysis of various alternative sources of finance. Which source should be
chosen, can be determined on the basis of cost of capital.
4. Helpful in evaluation of financial efficiency of top management: Cost of capital is helpful
in evaluation of financial efficiency of top management. The top management prepares
investment budgets for various projects. In these budgets future profitability and costs are
estimated. After implementation of the project, by comparison of estimated and actual costs,
it can be seen how much the top management has been successful in preparation of budgets
and implementation of projects. Ability to generate profit is no longer a test of profit
adequacy. Ability to generate Economic Value Added (EVA™) is the only test of adequacy.
“Any surplus generated from operating activities over and above the cost of capital is termed
as Economic Value Added”. EVA™ is defined as— “Excess Profits of a firm after charging Cost
of Capital”.
5. Useful in other areas: Cost of capital is useful in dividend policy and working capital policy
decisions also.

Type of cost of capital


Cost of
Capital (𝑘)

Theoretical Practical Classification or


Classification Specific Cost of Capital

Implicit Explicit Cost of Cost of Cost of Cost of Overall


Cost of Cost of Debt Pref. Equity Retained Cost of
Capital Capital (𝑘𝑖 /𝑘𝑑 ) Share (𝑘𝑒 ) Earnings Capital
Capital (𝑘𝑟 ) / WACC
(𝑘𝑝 ) (𝑘𝑂 )

Chapter 7, Cost of Capital: 1


Theoretical Classification
1. Implicit Cost of Capital: Implicit cost of capital is the opportunity cost of capital i.e. the
benefit that could have been earned if the funds were used elsewhere instead of using in the
business. E.g., a business purchased a plant and machinery of ₹5,00,000 for production.
Company could earn ₹50,000 per annum as interest if these funds were invested in the
bank. But in order to get the benefit from the machinery the company is sacrificing the
benefit of the interest. So, this sacrificed cost is called implicit cost.
2. Explicit Cost of Capital: Explicit cost of capital is that expense which is actually incurred for
using any fund. E.g., interest paid on debt, dividend on preference shares, etc. Formula for
the computation of explicit cost of capital is as follows—
𝑛
𝐶𝑂𝑡 𝐶𝑂𝑃𝑛
𝐶𝐼0 = ∑ + , 𝑜𝑟
(1 + 𝑘)𝑡 (1 + 𝑘)𝑛
𝑡=1
𝐶𝑂1 𝐶𝑂2 𝐶𝑂3 𝐶𝑂𝑛 𝐶𝑂𝑃𝑛
𝐶𝐼0 = 1
+ 2
+ 3
+ ⋯⋯⋯+ 𝑛
+
(1 + 𝑘) (1 + 𝑘) (1 + 𝑘) (1 + 𝑘) (1 + 𝑘)𝑛
Where,
𝐶𝐼0 =Cash inflows of the fund issued in zero year
𝐶𝑂𝑡 =Cash outflows in the tth year in the form of interest or dividend etc.
𝐶𝑂𝑃𝑛 = Cash outflows in the nth year in the form of principal
𝑘 =Cost of capital, 𝑡 =time period from 1 to n, 𝑛 =Number of years to maturity

Practical Classification
1. Cost of Debt (𝒌𝒊 /𝒌𝒅 )
Face Value ……
Debt may be of two types i.e., irredeemable and
(-) Disct on issue (……)
redeemable. In case of both, the methods for the
+ Premium on issue ……
computation are different.
Issue Price ……
(-) Floatation costs (……)
A. Irredeemable Debt Sales Value/NP ……
𝐼 Note: If nothing is mentioned then
𝑘𝑖 (𝐵𝑒𝑓𝑜𝑟𝑒 𝑡𝑎𝑥 𝑐𝑜𝑠𝑡 𝑜𝑓 𝑑𝑒𝑏𝑡) =
𝑆𝑉 flotation costs are taken as a
𝐼(1 − 𝑡) percentage of the face value.
𝑘𝑑 𝑜𝑟 𝑘𝑏 (𝐴𝑓𝑡𝑒𝑟 𝑡𝑎𝑥 𝑐𝑜𝑠𝑡 𝑜𝑓 𝑑𝑒𝑏𝑡) =
𝑆𝑉
Where, 𝐼 =Interest on Debt, 𝑆𝑉 =Sales Value
(If market value is given then it shall be treated as the SV), 𝑡 =Tax Rate

B. Redeemable Debt
In case of redeemable debt, the cost of debt may be defined as—that rate of discount which equates
the present value of cash inflows of debt with the present value of cash outflows of debt.
Symbolically—
𝑛
𝐶𝑂𝐼𝑡 (1 − 𝑡𝑎𝑥) 𝐶𝑂𝑃𝑛
𝐶𝐼0 = ∑ 𝑡
+ (𝑊ℎ𝑒𝑛 𝑝𝑟𝑖𝑛𝑐𝑖𝑝𝑎𝑙 𝑖𝑠 𝑟𝑒𝑝𝑎𝑖𝑑 𝑖𝑛 𝑡ℎ𝑒 𝑙𝑎𝑠𝑡 𝑦𝑒𝑎𝑟)
(1 + 𝑘𝑑 ) (1 + 𝑘𝑑 )𝑛
𝑡=1

𝑛
𝐶𝑂𝐼𝑡 (1 − 𝑡𝑎𝑥) + 𝐶𝑂𝑃𝑡
𝐶𝐼0 = ∑ (𝑊ℎ𝑒𝑛 𝑝𝑟𝑖𝑛𝑐𝑖𝑝𝑎𝑙 𝑖𝑠 𝑟𝑒𝑝𝑎𝑖𝑑 𝑖𝑛 𝑖𝑛𝑠𝑡𝑎𝑙𝑙𝑚𝑒𝑛𝑡𝑠)
(1 + 𝑘𝑑 )𝑡
𝑡=1
Where, 𝐶𝐼0 = Cash inflows in zero time period or SV (Sales Value)
𝐶𝑂𝐼𝑡 = Cash outflows in the form of interest in 𝑡𝑡ℎ year
𝐶𝑂𝑃𝑡 = Cash outflows in the form of principal in 𝑡𝑡ℎ year
𝐶𝑂𝑃𝑛 = Cash outflows in the form of principal in 𝑛𝑡ℎ year
𝑘𝑑 = Cost of debt (After tax)
𝑛 = Number of years of maturity

Chapter 7, Cost of Capital: 2


In the above formula put that value of 𝑘𝑑 which can equate the both sides. But we have no idea of
that value of the 𝑘𝑑 , so, we can try different values of the 𝑘𝑑 . Once we get two values (percentages)
of 𝑘𝑑 (at one percentage (higher) the right side is less than the left side and at another percentage
(lower) the right side is more than the left side), go to the next step i.e., interpolation.
Formula for the interpolation (If using trial and error method)
𝑃𝑉𝐿𝐷𝑅 − 𝐶𝐼0 𝑜𝑟 𝑆𝑉
𝑘𝑑 = 𝐿𝐷𝑅 + × |𝐷𝑖𝑓𝑓𝑒𝑟𝑒𝑛𝑐𝑒 𝑜𝑓 𝑅𝑎𝑡𝑒𝑠|
𝑃𝑉𝐿𝐷𝑅 − 𝑃𝑉𝐻𝐷𝑅
𝑃𝑉 = 𝑃𝑟𝑒𝑠𝑒𝑛𝑡 𝑣𝑎𝑙𝑢𝑒 𝑜𝑓 𝑐𝑎𝑠ℎ 𝑜𝑢𝑡𝑓𝑙𝑜𝑤𝑠
𝐿𝐷𝑅 = 𝐿𝑜𝑤𝑒𝑟 𝐷𝑖𝑠𝑐𝑜𝑢𝑛𝑡 𝑅𝑎𝑡𝑒
𝐻𝐷𝑅 = 𝐻𝑖𝑔ℎ𝑒𝑟 𝐷𝑜𝑠𝑐𝑜𝑢𝑛𝑡 𝑅𝑎𝑡𝑒
Difference of rates is taken after ignoring the signs.
Shortcut can be used when principal is repaid in the last year of maturity
𝐼(1 − 𝑡𝑎𝑥) + (𝑓 + 𝑑 + 𝑝𝑟 − 𝑝𝑖 )/𝑁𝑚
𝑘𝑑 = 𝑜𝑟
(𝑅𝑉 + 𝑆𝑉)/2
𝐼(1 − 𝑡𝑎𝑥) + (𝑅𝑉 − 𝑆𝑉)/𝑁𝑚
𝑘𝑑 =
(𝑅𝑉 + 𝑆𝑉)/2
Where,
𝐼 = Interest on debt
𝑓 = Floatation costs
𝑑 = Discount on issue
𝑝𝑟 = Premium on redemption
𝑝𝑖 = Premium on issue
𝑁𝑚 = Number of years to maturity
𝑅𝑉 = Redeemable value
𝑆𝑉 = Sales value (If market value is given then it shall be treated as the SV)

2. Cost of preference share capital


(𝒌𝒑 ) Face Value ……
(-) Disct on issue (……)
Preference share capital may be of two types i.e.,
+ Premium on issue ……
irredeemable and redeemable. In case of both, the
Issue Price ……
method of computation is different.
(-) Floatation costs (……)
Sales Value/NP ……
A. Irredeemable Preference Share Note: If nothing is mentioned then
Capital flotation costs are taken as a
𝐷𝑝 (1 + 𝐷𝑡 ) percentage of face value.
𝑘𝑝 =
𝑆𝑉
Where,
𝐷𝑝 = Dividend on preference share capital
𝐷𝑡 = Dividend distribution tax rate
𝑆𝑉 = Sales value (If market value is given then it shall be treated as the SV)

B. Redeemable Preference Share Capital


In case of redeemable preference share capital, the cost of preference share capital may be defined
as—that rate of discount which equates the present value of cash inflows of preference share capital
with the present value of cash outflows of preference share capital. Symbolically—
𝑛
𝐷𝑝(𝑡) (1 + 𝐷𝑡 ) 𝐶𝑂𝑃𝑛
𝐶𝐼0 = ∑ 𝑡
+ 𝑛 (𝑊ℎ𝑒𝑛 𝑝𝑟𝑖𝑛𝑐𝑖𝑝𝑎𝑙 𝑖𝑠 𝑟𝑒𝑝𝑎𝑖𝑑 𝑖𝑛 𝑡ℎ𝑒 𝑙𝑎𝑠𝑡 𝑦𝑒𝑎𝑟)
(1 + 𝑘𝑝 ) (1 + 𝑘𝑝 )
𝑡=1

𝑛
𝐷𝑝(𝑡) (1 + 𝐷𝑡 ) + 𝐶𝑂𝑃𝑡
𝐶𝐼0 = ∑ (𝑊ℎ𝑒𝑛 𝑝𝑟𝑖𝑛𝑐𝑖𝑝𝑎𝑙 𝑖𝑠 𝑟𝑒𝑝𝑎𝑖𝑑 𝑖𝑛 𝑖𝑛𝑠𝑡𝑎𝑙𝑙𝑚𝑒𝑛𝑡𝑠)
(1 + 𝑘𝑝 )𝑡
𝑡=1
Where,
𝐶𝐼0 = Cash inflows in zero time period or SV (Sales Value)

Chapter 7, Cost of Capital: 3


𝐷𝑝(𝑡) = Cash outflows in the form of dividend in 𝑡𝑡ℎ year
𝐷𝑡 = Rate of dividend distribution tax
𝐶𝑂𝑃𝑡 = Cash outflows in the form of principal in 𝑡𝑡ℎ year
𝐶𝑂𝑃𝑛 = Cash outflows in the form of principal in 𝑛𝑡ℎ year
𝑘𝑝 = Cost of preference share capital
𝑛 = Number of years of maturity
In the above formula put that value of 𝑘𝑝 which can equate the both sides. But we have no idea of
that value of the 𝑘𝑝 , so, we can try different values of the 𝑘𝑝 . Once we get two values (percentages)
of 𝑘𝑝 (at one percentage (higher) the right side is less than the left side and at another percentage
(lower) the right side is more than the left side), go to the next step i.e., interpolation.
Formula for the interpolation (If using trial and error method)
𝑃𝑉𝐿𝐷𝑅 − 𝐶𝐼0 𝑜𝑟 𝑆𝑉
𝑘𝑝 = 𝐿𝐷𝑅 + × |𝐷𝑖𝑓𝑓𝑒𝑟𝑒𝑛𝑐𝑒 𝑜𝑓 𝑅𝑎𝑡𝑒𝑠|
𝑃𝑉𝐿𝐷𝑅 − 𝑃𝑉𝐻𝐷𝑅
𝑃𝑉 = 𝑃𝑟𝑒𝑠𝑒𝑛𝑡 𝑣𝑎𝑙𝑢𝑒 𝑜𝑓 𝑐𝑎𝑠ℎ 𝑜𝑢𝑡𝑓𝑙𝑜𝑤𝑠
𝐿𝐷𝑅 = 𝐿𝑜𝑤𝑒𝑟 𝐷𝑖𝑠𝑐𝑜𝑢𝑛𝑡 𝑅𝑎𝑡𝑒
𝐻𝐷𝑅 = 𝐻𝑖𝑔ℎ𝑒𝑟 𝐷𝑜𝑠𝑐𝑜𝑢𝑛𝑡 𝑅𝑎𝑡𝑒
Difference of rates is taken after ignoring the signs.
Shortcut can be used when principal is repaid in last year of maturity
𝐷𝑝 (1 + 𝐷𝑡 ) + (𝑓 + 𝑑 + 𝑝𝑟 − 𝑝𝑖 )/𝑁𝑚
𝑘𝑝 = 𝑜𝑟
(𝑅𝑉 + 𝑆𝑉)/2
𝐷𝑝 (1 + 𝐷𝑡 ) + (𝑅𝑉 − 𝑆𝑉)/𝑁𝑚
𝑘𝑝 =
(𝑅𝑉 + 𝑆𝑉)/2
Where,
𝐷𝑝 = Dividend on preference share
𝐷𝑡 = Dividend distribution tax rate
𝑓 = Floatation costs
𝑑 = Discount on issue
𝑝𝑟 = Premium on redemption
𝑝𝑖 = Premium on issue
𝑁𝑚 = Number of years to maturity
𝑅𝑉 = Redeemable value
𝑆𝑉 = Sales value (If market value is given then it shall be treated as the SV)

3. Cost of equity share capital (𝒌𝒆 )


In the case of debt and preference share capital we have the base to compute the cost of capital i.e.,
the interest paid and the dividend paid. But in case of equity shares the rate of dividend is not fixed
and sometimes no dividends are paid. In such a case it is very difficult to compute the cost of equity
share capital. But there is a solution to this problem—one has to estimate the future dividends to be
paid on the equity shares. So, the base for the computation of the cost of equity share capital is the
expected/future dividends.
There are 5 methods to compute the equity share capital. These are—
1. Dividend Price Model
2. Dividend Price Growth Model
3. Price Earning Method
4. Capital Asset Pricing Model (CAPM)
5. Realized Yield Approach

1. Dividend Price Model


𝐷 𝑜𝑟 𝐷1 𝐷 𝑜𝑟 𝐷1
𝑘𝑒 = ⇒ 𝑃0 =
𝑃0 𝑘𝑒
Where,
𝐷 = Dividend paid
𝐷1 = Dividend to be paid on equity shares or expected dividend
𝑃0 = Net proceeds from issue of an equity share after adjusting discount/premium,
floatation cost, etc. of the share. If market price is given then use market price only.
Chapter 7, Cost of Capital: 4
In case of existing equity share capital there is no need to subtract the floatation cost.

2. Dividend Price Growth Model


There are 3 assumptions of this model—
1. 𝐷0 > 0.
2. Dividend payout ratio is constant.
3. 𝑘𝑒 > 𝑔.
Model is as follows—
𝐷1 𝐷1
𝑘𝑒 = + 𝑔 ⇒ 𝑃0 =
𝑃0 𝑘𝑒 − 𝑔
(Proving the Dividend Growth Model or Gordon Growth Model)
Where,
𝑃0 = Net proceeds from issue of an equity share after adjusting discount/premium,
floatation cost, etc. of the share. If market price is given then use market price only.
In case of existing equity share capital there is no need to subtract the floatation cost.

𝐷1 = Dividend to be paid on equity shares OR Expected dividend


𝐷0 = Dividend last paid OR Dividend paid last year

𝐷1 = 𝐷0 (1 + 𝑔)1
𝐷2 = 𝐷0 (1 + 𝑔)2 𝑜𝑟 𝐷2 = 𝐷0 (1 + 𝑔)(1 + 𝑔) 𝑜𝑟 𝐷2 = 𝐷1 (1 + 𝑔)
𝐷3 = 𝐷0 (1 + 𝑔)3 𝑜𝑟 𝐷3 = 𝐷2 (1 + 𝑔)

𝑔 = Growth rate
𝑔 = 𝑏. 𝑟 (Where, 𝑏 = Retention ratio; 𝑟 = Rate of return)
𝐸𝑃𝑆 − 𝐷𝑃𝑆(1 + 𝐷𝑡 ) 𝐸𝑃𝑆
𝑏= ,𝑟 =
𝐸𝑃𝑆 𝑃0
𝐸𝑃𝑆 − 𝐷𝑃𝑆(1 + 𝐷𝑡 ) 𝐸𝑃𝑆 𝐸𝑃𝑆 − 𝐷𝑃𝑆(1 + 𝐷𝑡 )
𝑆𝑜, 𝑔 = × =
𝐸𝑃𝑆 𝑃0 𝑃0
Alternatively, 𝒈 can also be computed as follows—
Calculate the growth factor—
𝐿𝑎𝑡𝑒𝑠𝑡 𝐷𝑖𝑣𝑖𝑑𝑒𝑛𝑑
𝐺𝑟𝑜𝑤𝑡ℎ 𝑓𝑎𝑐𝑡𝑜𝑟 =
𝐼𝑛𝑖𝑡𝑖𝑎𝑙 𝐷𝑖𝑣𝑖𝑑𝑒𝑛𝑑
Now locate the growth factor in the table A-1 corresponding to the period 𝑛 − 1 and note
down the percentage corresponding to this growth factor. This percentage is the growth
rate. In case of two percentages (when growth factor lies between two values then two
percentages will be there) average may be used or g can be interpolated as follows—
𝑔 = 𝐻𝑖𝑔ℎ𝑒𝑟 𝑝𝑒𝑟𝑐𝑒𝑛𝑡𝑎𝑔𝑒
𝐹𝑎𝑐𝑡𝑜𝑟 𝑎𝑡 ℎ𝑖𝑔ℎ𝑒𝑟 𝑝𝑒𝑟𝑐𝑒𝑛𝑡𝑎𝑔𝑒 − 𝐺𝑟𝑜𝑤𝑡ℎ 𝑓𝑎𝑐𝑡𝑜𝑟

𝐹𝑎𝑐𝑡𝑜𝑟 𝑎𝑡 ℎ𝑖𝑔ℎ𝑒𝑟 𝑝𝑒𝑟𝑐𝑒𝑛𝑡𝑎𝑔𝑒 − 𝐹𝑎𝑐𝑡𝑜𝑟 𝑎𝑡 𝑙𝑜𝑤𝑒𝑟 𝑝𝑒𝑟𝑐𝑒𝑛𝑡𝑎𝑔𝑒
× |𝐷𝑖𝑓𝑓𝑒𝑟𝑒𝑛𝑐𝑒 𝑜𝑓 𝑟𝑎𝑡𝑒𝑠|
or
𝑔 = 𝐿𝑜𝑤𝑒𝑟 𝑝𝑒𝑟𝑐𝑒𝑛𝑡𝑎𝑔𝑒
𝐹𝑎𝑐𝑡𝑜𝑟 𝑎𝑡 𝑙𝑜𝑤𝑒𝑟 𝑝𝑒𝑟𝑐𝑒𝑛𝑡𝑎𝑔𝑒 − 𝐺𝑟𝑜𝑤𝑡ℎ 𝑓𝑎𝑐𝑡𝑜𝑟
+
𝐹𝑎𝑐𝑡𝑜𝑟 𝑎𝑡 𝑙𝑜𝑤𝑒𝑟 𝑝𝑒𝑟𝑐𝑒𝑛𝑡𝑎𝑔𝑒 − 𝐹𝑎𝑐𝑡𝑜𝑟 𝑎𝑡 ℎ𝑖𝑔ℎ𝑒𝑟 𝑝𝑒𝑟𝑐𝑒𝑛𝑡𝑎𝑔𝑒
× |𝐷𝑖𝑓𝑓𝑒𝑟𝑒𝑛𝑐𝑒 𝑜𝑓 𝑟𝑎𝑡𝑒𝑠|
Note: The above formulae for the interpolation give the approximate answers.

3. Price Earning Method


𝐸𝑃𝑆 1
𝑘𝑒 = ⇒ 𝑘𝑒 =
𝑃0 𝑜𝑟 𝑀𝑃 𝑃/𝐸 𝑅𝑎𝑡𝑖𝑜
Where,
𝐸𝑃𝑆 = Earning per shares
𝑃0 = Net proceeds of an equity share
𝑀𝑃 = Market price of the share
In case of existing equity share capital there is no need to subtract the floatation cost.
Chapter 7, Cost of Capital: 5
4. Capital Assets Pricing Model (CAPM) or CAP Model
𝑘𝑒 = 𝑅𝑓 + 𝛽(𝐾𝑚 − 𝑅𝑓 )
Where,
𝑅𝑓 = Risk free rate of return i.e., rate of return on government securities
𝐶𝑎𝑝𝑖𝑡𝑎𝑙 𝑔𝑎𝑖𝑛 + 𝑅𝑒𝑣𝑒𝑛𝑢𝑒 𝑔𝑎𝑖𝑛
𝐾𝑚 (𝑅𝑎𝑡𝑒 𝑜𝑓 𝑟𝑒𝑡𝑢𝑟𝑛 𝑜𝑛 𝑚𝑎𝑟𝑘𝑒𝑡 𝑝𝑜𝑟𝑡𝑓𝑜𝑙𝑖𝑜) =
𝐼𝑛𝑖𝑡𝑖𝑎𝑙 𝑣𝑎𝑙𝑢𝑒 𝑜𝑓 𝑡ℎ𝑒 𝑝𝑜𝑟𝑡𝑓𝑜𝑙𝑖𝑜
(𝑀𝑎𝑟𝑘𝑒𝑡 𝑣𝑎𝑙𝑢𝑒 𝑜𝑓 𝑡ℎ𝑒 𝑝𝑜𝑟𝑡𝑓𝑜𝑙𝑖𝑜 − 𝐼𝑛𝑖𝑡𝑖𝑎𝑙 𝑣𝑎𝑙𝑢𝑒 𝑜𝑓 𝑡ℎ𝑒 𝑝𝑜𝑟𝑡𝑓𝑜𝑙𝑖𝑜) + 𝐷𝑖𝑣. 𝑎𝑛𝑑 𝐼𝑛𝑡. 𝑟𝑒𝑐𝑒𝑖𝑣𝑒𝑑

𝐼𝑛𝑖𝑡𝑖𝑎𝑙 𝑣𝑎𝑙𝑢𝑒 𝑜𝑓 𝑡ℎ𝑒 𝑝𝑜𝑟𝑡𝑓𝑜𝑙𝑖𝑜
𝛽 = Beta is the measurement of systematic risk (Also called Beta Coefficient). Systematic
risk is the risk which is un-diversifiable in nature. In other words, we can say that we cannot
eliminate this risk just by diversifying our portfolio. Calculation of  is as follows—
𝜌𝑖𝑚 . 𝜎𝑖 . 𝜎𝑚 𝜌𝑖𝑚 . 𝜎𝑖 . 𝜎𝑚 𝜌𝑖𝑚 . 𝜎𝑖
𝛽𝑖 = 2 ⇒ ⇒
𝜎𝑚 𝜎𝑚 . 𝜎𝑚 𝜎𝑚
𝐶𝑜𝑟𝑟𝑒𝑙𝑎𝑡𝑖𝑜𝑛 𝑏𝑒𝑡𝑤𝑒𝑒𝑛 𝑠𝑒𝑐𝑢𝑟𝑖𝑡𝑦 𝑟𝑒𝑡𝑢𝑟𝑛 𝑎𝑛𝑑 𝑚𝑎𝑟𝑘𝑒𝑡 𝑟𝑒𝑡𝑢𝑟𝑛 × 𝑆𝑡𝑑. 𝐷𝑒𝑣. 𝑜𝑓 𝑠𝑒𝑐𝑢𝑟𝑖𝑡𝑦

𝑆𝑡𝑑. 𝐷𝑒𝑣. 𝑜𝑓 𝑚𝑎𝑟𝑘𝑒𝑡 𝑟𝑒𝑡𝑢𝑟𝑛

5. Realized yield approach


Under this approach we calculate the IRR. Price paid to purchase the share is taken as cash
outflows. Dividends received and selling price of the share are cash inflows. The process to calculate
the IRR has already been discussed in detail in the chapter Capital Budgeting.
Note: In order to use this method, there should be given outflows and inflows (dividends and the
future selling price) of the equity share.

A final note on floatation cost


Cost of equity
Existing equity New equity
Shall not be subtracted from market Shall be subtracted from market
price or issue price or sales value. price or issue price.
Floatation
Note: In case of any confusion, students Note: In case of any confusion, students
costs
may append a note and then accordingly may append a note and then accordingly
may decide regarding subtraction of may decide regarding subtraction of
floatation cost. floatation cost.

Which approach to use?


IMPORTANT • In the case of companies with stable income and with constant dividend policies the
TO REMEMBER Dividend Model may be a good way of measuring the cost of equity share capital.
• In the case of companies whose earnings accrue in cycles, it would be better if the Price
Earning Method is used.
• In the case of growth companies, where expectations of growth are more important, the cost
of equity share capital may be determined on the basis of Dividend Growth Model.
• In case of companies enjoying a steady growth rate, as well as a steady rate of dividend, the
realized value approach may be useful.
• The basic factor in determining the cost of ordinary share capital is to measure the
expectations of investors from the ordinary shares of that particular company. Therefore,
the whole question of determining the cost of ordinary share depends on the factors which
go in to the expectations of particular group of investors in a company of a particular risk
class.

4 Cost of retained earnings (𝒌𝒓 )


It may appear to carry no cost since they represent funds which have not been raised from outside.
The contention that retained earnings are free of cost, however, is not correct. On the contrary, they
do involve cost like any other source.

Chapter 7, Cost of Capital: 6


The cost of retained earnings is closely related to the equity shares. If the earnings are not retained,
they will be paid out to the equity shareholders as dividends. Retained earnings are often looked as
fully subscribed issue of new equity, since they increase the shareholder’s equity in the same way
that a new issue of equity shares would. The cost of retained earnings must, therefore, be viewed as
the opportunity cost of the foregone dividends to the existing shareholders. Cost of retained
earnings can also be computed with the help of above methods namely—
1. Dividend Model
2. Dividend Growth Model
3. Price Earning Method
4. Capital Asset Pricing Model (CAPM)
Cost of retained earnings can also be computed using the following formula—
𝑘𝑟 = 𝑘𝑒 (1 − 𝑡)(1 − 𝑏)(1 − 𝑓)
Where,
𝑡 = Tax rate on income of shareholders
𝑏 = Brokerage rate to be paid by the shareholder for making new investments
𝑓 = Floatation cost on issue of new share capital

A final note on floatation cost


Cost of retained earnings
Floatation
Shall not be subtracted from market price or issue price or sales value
costs

5. Weighted average cost of capital (WACC) or Overall cost of capital


(𝒌𝑶 )
WACC can be computed using the following steps—
Step I: Compute the specific cost of capital i.e., 𝑘𝑑 , 𝑘𝑝 , 𝑘𝑒 , and 𝑘𝑟 .
Step II: Assign weights to specific costs.
Step III: Multiply the cost of each source by the appropriate weight and obtain the product.
Step IV: Compute the weighted average cost of capital by adding the products obtained in step III.
So,
𝑘𝑂 = [𝑘𝑑 × 𝑤𝑑 ] + [𝑘𝑝 × 𝑤𝑝 ] + [𝑘𝑒 × 𝑤𝑒 ] + [𝑘𝑟 × 𝑤𝑟 ]
Where,
𝑤 = Weight i.e., Proportion of particular fund in total funds
Formula to compute the weight is—
𝐴𝑚𝑜𝑢𝑛𝑡 𝑜𝑓 𝑠𝑝𝑒𝑐𝑖𝑓𝑖𝑐 𝑙𝑜𝑛𝑔 − 𝑡𝑒𝑟𝑚 𝑓𝑢𝑛𝑑
𝑤=
𝑇𝑜𝑡𝑎𝑙 𝑙𝑜𝑛𝑔 − 𝑡𝑒𝑟𝑚 𝑓𝑢𝑛𝑑𝑠
Where,
𝑤𝑑 = Proportion of debt in total funds (Weight of debt)
𝑤𝑝 = Proportion of preference share capital in total funds (Weight of pref. share capital)
𝑤𝑒 = Proportion of equity share capital in total funds (Weight of equity share capital)
𝑤𝑟 =Proportion of retained earnings in total funds (Weight of retained earnings)

Type of weights (w)

MARKET VALUE
WEIGHTS
HISTORICAL
WEIGHTS
BOOK VALUE
WEIGHTS WEIGHTS

MARGINAL
WEIGHTS

Chapter 7, Cost of Capital: 7


Method–I
1 2 3 4 5=34
Product (Amount 
Source of Fund Amount Proportion Cost of Capital
Cost of Capital)
Debt √ 𝑤 𝑘𝑑 𝑤 × 𝑘𝑑
Preference Share Capital √ 𝑤𝑝 𝑘𝑝 𝑤𝑝 × 𝑘𝑝
Equity Share Capital √ 𝑤𝑒 𝑘𝑒 𝑤𝑒 × 𝑘𝑒
Retained Earnings √ 𝑤𝑟 𝑘𝑟 𝑤𝑟 × 𝑘𝑟
𝑘𝑂 𝑜𝑟 𝑊𝐴𝐶𝐶 = Sum of products

Method–II
𝑇𝑜𝑡𝑎𝑙 𝑎𝑏𝑠𝑜𝑙𝑢𝑡𝑒 𝑐𝑜𝑠𝑡 𝑜𝑟 𝑡𝑜𝑡𝑎𝑙 𝑒𝑥𝑝𝑙𝑖𝑐𝑖𝑡 𝑐𝑜𝑠𝑡
𝑘𝑂 =
𝑇𝑜𝑡𝑎𝑙 𝑙𝑜𝑛𝑔 𝑡𝑒𝑟𝑚 𝑓𝑢𝑛𝑑𝑠
1 2 3 4=23
Cost of
Source of Fund Amount Product (Amount  Cost of Capital)
Capital
Debt √ 𝑘𝑑 √
Preference Share Capital √ 𝑘𝑝 √
Equity Share Capital √ 𝑘𝑒 √
Retained Earnings √ 𝑘𝑟 √
Total (A) Total (B)
Now,
𝑆𝑢𝑚 𝑜𝑓 𝑃𝑟𝑜𝑑𝑢𝑐𝑡𝑠 (𝐶𝑜𝑙𝑢𝑚𝑛 4) 𝐵
𝑘𝑂 = =
𝑇𝑜𝑡𝑎𝑙 𝑓𝑢𝑛𝑑𝑠 (𝐶𝑜𝑙𝑢𝑚𝑛 2) 𝐴

Addendum: Proving the Dividend Growth Model or Gordon


Growth Model [Not in syllabus]
Formula for the Gordon Growth Model is as follows—
𝐷0 (1 + 𝑔)
𝑃0 =
𝑘𝑒 − 𝑔
Where,
𝑃0 =Current price of the equity share
𝐷0 = Dividend declared/paid
𝑘𝑒 = Cost of equity
𝑔 = Growth rate
Current price of an equity share is the present value of future dividends—
𝐷1 𝐷2 𝐷3 𝐷𝑛 𝐷∞
𝑃0 = 1
+ 2
+ 3
+ ⋯⋯⋯+ 𝑛
+ ⋯⋯⋯+
(1 + 𝑘𝑒 ) (1 + 𝑘𝑒 ) (1 + 𝑘𝑒 ) (1 + 𝑘𝑒 ) (1 + 𝑘𝑒 )∞

𝐷0 (1 + 𝑔)𝑡
𝑃𝑟𝑒𝑠𝑒𝑛𝑡 𝑣𝑎𝑙𝑢𝑒 𝑜𝑓 𝑓𝑢𝑡𝑢𝑟𝑒 𝑑𝑖𝑣𝑖𝑑𝑒𝑛𝑑𝑠 = 𝑃0 = ∑
(1 + 𝑘𝑒 )𝑡
𝑡=1
Let us assume that—
(1 + 𝑔)
=𝑎
(1 + 𝑘𝑒 )
Now equation would be—

𝐷0 ∑(𝑎𝑛 ) = 𝐷0 (𝑎1 + 𝑎2 + 𝑎3 + 𝑎4 + ⋯ )
𝑡=1
Sum of a geometric series is—

𝑟
∑(𝑟𝑎𝑛 ) = ⋯ ⋯ ⋯ 𝑝𝑟𝑜𝑣𝑖𝑑𝑒𝑑 |𝑎| < 1
1−𝑎
𝑡=0
When 𝑟 = 1, the equation can be simplified to—

Chapter 7, Cost of Capital: 8


1
1−𝑎
Our geometric series starts from 1, in order to begin 𝑡 at zero, let us factor out an 𝑎 from all the
terms of the series—

𝐷0 ∑(𝑎𝑛 ) = 𝐷0 𝑎(𝑎0 + 𝑎1 + 𝑎2 + 𝑎3 + 𝑎4 + ⋯ )
𝑡=0

𝐷0 ∑(𝑎𝑛 ) = 𝐷0 𝑎(1 + 𝑎1 + 𝑎2 + 𝑎3 + 𝑎4 + ⋯ )
𝑡=0
Because 𝑟 = 1, it simplifies to—

1
𝐷0 ∑(𝑎𝑛 ) = 𝐷0 𝑎
1−𝑎
𝑡=0
Simplifying again, we get—
𝐷0 𝑎
1−𝑎
We know that—
(1 + 𝑔)
=𝑎
(1 + 𝑘𝑒 )
So, replacing 𝑎—
1+𝑔
𝐷0
1 + 𝑘𝑒
( )
1+𝑔
1−
1 + 𝑘𝑒
If we multiply the numerator and denominator by (1 + 𝑟), this results in—
𝐷0 (1 + 𝑔)
( )
(1 + 𝑘𝑒 ) − (1 + 𝑔)
Simplifying the denominator—
𝐷0 (1 + 𝑔)
( )
𝑘𝑒 − 𝑔
So—

𝐷0 (1 + 𝑔)𝑡 𝐷0 (1 + 𝑔) 𝐷1
𝑃𝑟𝑒𝑠𝑒𝑛𝑡 𝑣𝑎𝑙𝑢𝑒 𝑜𝑓 𝑓𝑢𝑡𝑢𝑟𝑒 𝑑𝑖𝑣𝑖𝑑𝑒𝑛𝑑𝑠 = 𝑃0 = ∑ = ( ) = ∎
(1 + 𝑘𝑒 )𝑡 𝑘𝑒 − 𝑔 𝑘𝑒 − 𝑔
𝑡=1

Chapter 7, Cost of Capital: 9


Illustration
Example 1
Face value of a 12% debenture is ₹1,000. Tax rate is 30% and floatation cost is 2% of the face
value. Calculate before tax and after-tax cost of debt if this debenture is issued—(a) at par; (b) at
premium of 10% and (c) at a discount of 10%.

Solution
Maturity period is not given in the question, so, it’s an irredeemable debenture.
At premium of At discount of
Particulars At par 10% 10%
Face value 1,000 1,000 1,000
Add: Premium on issue @ 10% 0 100 0
Less: Discount on issue @ 10% 0 0 -100
Issue price 1,000 1,100 900
Less: Floatation cost @ 2% of the face value -20 -20 -20
Sales value 980 1,080 880

𝐴𝑚𝑜𝑢𝑛𝑡 𝑜𝑓 𝑖𝑛𝑡𝑒𝑟𝑒𝑠𝑡 = ₹1,000 × 12% ₹120 ₹120 ₹120

𝐼 120 120 120


𝑘𝑖 (𝐵𝑒𝑓𝑜𝑟𝑒 𝑡𝑎𝑥 𝑐𝑜𝑠𝑡 𝑜𝑓 𝑑𝑒𝑏𝑡) = = 0.1224 = 0.1111 = 0.1364
𝑆𝑉 980 1,080 880

120(1 − 0.30) 120(1 − 0.30) 120(1 − 0.30)


𝐼(1 − 𝑡)
𝑘𝑑 𝑜𝑟 𝑘𝑏 (𝐴𝑓𝑡𝑒𝑟 𝑡𝑎𝑥 𝑐𝑜𝑠𝑡 𝑜𝑓 𝑑𝑒𝑏𝑡) = 980 1,080 880
𝑆𝑉 = 0.08571 = 0.07778 = 0.0955

Example 2
A company issues a 12% debenture of ₹1,000 at 5% discount. It is redeemable after 5 years at a
premium of 10%. Floatation cost is 2% of the face value and tax rate is 30%. Calculate the after-tax
cost of debt.

Solution:

Trial and error method


𝑛
𝐶𝑂𝐼𝑡 (1 − 𝑡𝑎𝑥) 𝐶𝑂𝑃𝑛
𝐶𝐼0 = ∑ 𝑡
+
(1 + 𝑘𝑑 ) (1 + 𝑘𝑑 )𝑛
𝑡=1
120(1 − 0.30) 120(1 − 0.30) 120(1 − 0.30) 120(1 − 0.30)
930 (Note − 2) = + + +
(1 + 𝑘𝑑 )1 (1 + 𝑘𝑑 )2 (1 + 𝑘𝑑 )3 (1 + 𝑘𝑑 )4
120(1 − 0.30) 1,100
+ 5
+
(1 + 𝑘𝑑 ) (1 + 𝑘𝑑 )5
Let us use different percentages to check whether two sides are equal or not.
When 𝑘𝑑 is assumed to be 10% then, 930 < 1,001.46 (Note-1)
When 𝑘𝑑 is assumed to be 11% then, 930 < 962.764 (Note-1)
When 𝑘𝑑 is assumed to be 12% then, 930 > 926.520 (Note-1)
So, at 12% the present value of cash outflows (₹926.520) is approximately equal to the cash inflows
(sales value) of ₹930. So, the 𝑘𝑑 is 12%.
₹930 lies between ₹962.764 (at 11%) and ₹926.520 (at 12%). In order to get the exact answer, we
can interpolate the 𝑘𝑑 as follows—
𝑃𝑉𝐿𝐷𝑅 − 𝐶𝐼0 𝑜𝑟 𝑆𝑉 962.764 − 930
𝑘𝑑 = 𝐿𝐷𝑅 + × |𝐷𝑖𝑓𝑓𝑒𝑟𝑒𝑛𝑐𝑒 𝑜𝑓 𝑅𝑎𝑡𝑒𝑠| = 11 + ×1
𝑃𝑉𝐿𝐷𝑅 − 𝑃𝑉𝐻𝐷𝑅 962.764 − 926.520
= 11 + 0.9040 = 11.9040
If we calculate the present value of cash outflows at 11.9040% then it will be ₹930.374₹930.
Note – 1:
Chapter 7, Cost of Capital: 10
PVF @ PVF @ PVF @ PV @ PV @ PV @
Year 𝑪𝑶𝑰𝟏−𝟓 and 𝑪𝑶𝑷𝟓 10% 11% 12% 10% 11% 12%
1 120 (1-0.30)=84 0.909 0.901 0.893 76.356 75.684 75.012
2 120 (1-0.30)=84 0.826 0.812 0.797 69.384 68.208 66.948
3 120 (1-0.30)=84 0.751 0.731 0.712 63.084 61.404 59.808
4 120 (1-0.30)=84 0.683 0.659 0.636 57.372 55.356 53.424
5 120 (1-0.30)=84 0.621 0.593 0.567 52.164 49.812 47.628
5 1,100 0.621 0.593 0.567 683.100 652.300 623.700
Present value of cash outflows 1001.46 962.764 926.520
Note – 2:
Sales value =1,000 − 50 − 20 = 930

Shortcut method
𝐼(1 − 𝑡𝑎𝑥) + (𝑓 + 𝑑 + 𝑝𝑟 − 𝑝𝑖 )/𝑁𝑚 𝐼(1 − 𝑡𝑎𝑥) + (𝑅𝑉 − 𝑆𝑉)/𝑁𝑚
𝑘𝑑 = 𝑜𝑟 𝑘𝑑 =
(𝑅𝑉 + 𝑆𝑉)/2 (𝑅𝑉 + 𝑆𝑉)/2
120(1 − 0.30) + (20 + 50 + 100 − 0)/5
𝑘𝑑 = = 0.1163
(1,100 + 930)/2
120(1 − 0.30) + (1,100 − 930)/5
𝑜𝑟 𝑘𝑑 = = 0.1163
(1,100 + 930)/2

Example 3
A company issues a 12% debenture of ₹1,000 at 5% discount. It is redeemable in 5 equal annual
installments. Floatation cost is 2% of the face value and tax rate is 30%. Calculate the after-tax cost
of debt.

Solution

Trial and error method


𝑛
𝐶𝑂𝐼𝑡 (1 − 𝑡𝑎𝑥) + 𝐶𝑂𝑃𝑡
𝐶𝐼0 = ∑
(1 + 𝑘𝑑 )𝑡
𝑡=1
120(1 − 0.30) + 200 96(1 − 0.30) + 200 72(1 − 0.30) + 200
930 (Note − 1) = + +
(1 + 𝑘𝑑 )1 (1 + 𝑘𝑑 )2 (1 + 𝑘𝑑 )3
48(1 − 0.30) + 200 24(1 − 0.30) + 200
+ +
(1 + 𝑘𝑑 )4 (1 + 𝑘𝑑 )5
Let us use different percentages to check whether two sides are equal or not.
When 𝑘𝑑 is assumed to be 10% then, 930 < 961.095 (Note-1)
When 𝑘𝑑 is assumed to be 11% then, 930 < 938.398 (Note-1)
When 𝑘𝑑 is assumed to be 12% then, 930 > 916.350 (Note-1)
₹930 lies between ₹938.398 (at 11%) and ₹916.350 (at 12%). In order to get the exact answer, we
can interpolate the 𝑘𝑑 as follows—
𝑃𝑉𝐿𝐷𝑅 − 𝐶𝐼0 𝑜𝑟 𝑆𝑉 938.398 − 930
𝑘𝑑 = 𝐿𝐷𝑅 + × |𝐷𝑖𝑓𝑓𝑒𝑟𝑒𝑛𝑐𝑒 𝑜𝑓 𝑅𝑎𝑡𝑒𝑠| = 11 + ×1
𝑃𝑉𝐿𝐷𝑅 − 𝑃𝑉𝐻𝐷𝑅 938.398 − 916.350
= 11 + 0.3809 = 11.3809
If we calculate the present value of cash outflows at 11.3809% then it will be ₹929.845₹930.
Note – 1:
𝑪𝑶𝑰𝟏−𝟓 PVF @ PVF @ PVF @ PV @ PV @ PV @
Year 𝑪𝑶𝑰𝟏−𝟓 𝑪𝑶𝑷𝟏−𝟓 + 𝑪𝑶𝑷(𝟏−𝟓) 10% 11% 12% 10% 11% 12%
1 120 (1-0.3)=84 200 284 0.909 0.901 0.893 258.156 255.884 253.612
2 96 (1-0.3)=67.2 200 267.2 0.826 0.812 0.797 220.707 216.966 212.958
3 72 (1-0.3)=50.4 200 250.4 0.751 0.731 0.712 188.050 183.042 178.285
4 48 (1-0.3)=33.6 200 233.6 0.683 0.659 0.636 159.549 153.942 148.570
5 24 (1-0.3)=16.8 200 216.8 0.621 0.593 0.567 134.633 128.562 122.926
Present value of cash outflows 961.095 938.398 916.350
Note – 2:
Sales value =1,000 − 50 − 20 = 930

Chapter 7, Cost of Capital: 11


Shortcut method
Shortcut method cannot be applied because debenture is redeemable in installments.

Example 4
Face value of a preference share carrying a dividend rate of 12% is ₹1,000. Tax rate is 30% and
floatation cost is 2% of the face value. Assume that there is no dividend distribution tax. Calculate
cost of preference share capital if the share is issued —(a) at par; (b) at premium of 10% and (c) at a
discount of 10%.

Solution
Nothing is mentioned about the maturity of the preference share, so, it’s an irredeemable
preference share.
At premium of At discount of
Particulars At par 10% 10%
Face value 1,000 1,000 1,000
Add: Premium on issue @ 10% 0 100 0
Less: Discount on issue @ 10% 0 0 -100
Issue price 1,000 1,100 900
Less: Floatation cost @ 2% of the face value -20 -20 -20
Sales value 980 1,080 880
𝐴𝑚𝑜𝑢𝑛𝑡 𝑜𝑓 𝑑𝑖𝑣𝑖𝑑𝑒𝑛𝑑(𝐷𝑝 ) = ₹1,000 × 12% 120 120 120
120(1 + 0) 120(1 + 0) 120(1 + 0)
𝐷𝑝 (1 + 𝐷𝑡 )
𝑘𝑝 = 980 1,080 880
𝑆𝑉 = 0.1224 = 0.1111 = 0.1364

Example 5
A company issues a preference share of ₹1,000 carrying a dividend rate of 15% at 5% discount. It
is redeemable after 5 years at a premium of 10%. Floatation cost is 2% of the face value and tax rate
is 30%. Calculate the cost of preference share capital.

Solution

Trial and error method


𝑛
𝐷𝑝(𝑡) (1 + 𝐷𝑡 ) 𝐶𝑂𝑃𝑛
𝐶𝐼0 = ∑ 𝑡
+ 𝑛
(1 + 𝑘𝑝 ) (1 + 𝑘𝑝 )
𝑡=1
150(1 + 0) 150(1 + 0) 150(1 + 0) 150(1 + 0) 150(1 + 0)
930 (Note − 2) = + + + +
(1 + 𝑘𝑝 )1 (1 + 𝑘𝑝 )2 (1 + 𝑘𝑝 )3 (1 + 𝑘𝑝 )4 (1 + 𝑘𝑝 )5
1,100
+ 5
(1 + 𝑘𝑝 )
Let us use different percentages to check whether two sides are equal or not.
When 𝑘𝑝 is assumed to be 18% then, 930 < 949.750 (Note-1)
When 𝑘𝑝 is assumed to be 19% then, 930 > 919.450 (Note-1)
₹930 lies between ₹949.750 (at 18%) and ₹919.450 (at 19%). In order to get the exact answer, we
can interpolate the 𝑘𝑝 as follows—
𝑃𝑉𝐿𝐷𝑅 − 𝐶𝐼0 𝑜𝑟 𝑆𝑉 949.750 − 930
𝑘𝑝 = 𝐿𝐷𝑅 + × |𝐷𝑖𝑓𝑓𝑒𝑟𝑒𝑛𝑐𝑒 𝑜𝑓 𝑅𝑎𝑡𝑒𝑠| = 18 + ×1
𝑃𝑉𝐿𝐷𝑅 − 𝑃𝑉𝐻𝐷𝑅 949.750 − 919.450
= 18 + 0.652 = 18.652
If we calculate the present value of cash outflows at 18.652% then it will be ₹929.985₹930.
Note – 1:
Year 𝑫𝒑(𝟏−𝟓) 𝒂𝒏𝒅 𝑪𝑶𝑷𝟓 PVF @ 18% PVF @ 19% PV @ 18% PV @ 19%
1 150 0.847 0.840 127.050 126.000
2 150 0.718 0.706 107.700 105.900
3 150 0.609 0.593 91.350 88.950
Chapter 7, Cost of Capital: 12
4 150 0.516 0.499 77.400 74.850
5 150 0.437 0.419 65.550 62.850
5 1100 0.437 0.419 480.700 460.900
Present value of cash outflows 949.750 919.450
Note – 2:
Sales value =1,000 − 50 − 20 = 930

Shortcut method
𝐷𝑝 (1 + 𝐷𝑡 ) + (𝑓 + 𝑑 + 𝑝𝑟 − 𝑝𝑖 )/𝑁𝑚 𝐷𝑝 (1 + 𝐷𝑡 ) + (𝑅𝑉 − 𝑆𝑉)/𝑁𝑚
𝑘𝑝 = 𝑜𝑟 𝑘𝑝 =
(𝑅𝑉 + 𝑆𝑉)/2 (𝑅𝑉 + 𝑆𝑉)/2
150(1 + 0) + (20 + 50 + 100 − 0)/5
𝑘𝑝 = = 0.1813
(1,100 + 930)/2
150(1 + 0) + (1,100 − 930)/5
𝑜𝑟 𝑘𝑝 = = 0.1813
(1,100 + 930)/2

Example 6
A company issues a preference share of ₹1,000 carrying dividend rate of 15% at 5% discount. It is
redeemable in 5 equal annual installments. Floatation cost is 2% of the face value and tax rate is
30%. Calculate the after-tax cost of preference share capital.

Solution

Trial and error method


𝑛
𝐷𝑝(𝑡) (1 + 𝐷𝑡 ) + 𝐶𝑂𝑃𝑡
𝐶𝐼0 = ∑
(1 + 𝑘𝑝 )𝑡
𝑡=1
150(1 + 0) + 200 120(1 + 0) + 200 90(1 + 0) + 200
930 (Note − 2) = + +
(1 + 𝑘𝑝 )1 (1 + 𝑘𝑝 )2 (1 + 𝑘𝑝 )3
60(1 + 0) + 200 30(1 + 0) + 200
+ +
(1 + 𝑘𝑝 )4 (1 + 𝑘𝑝 )5
Let us use different percentages to check whether two sides are equal or not.
When 𝑘𝑝 is assumed to be 18% then, 930 < 937.490 (Note-1)
When 𝑘𝑑 is assumed to be 19% then, 930 > 918.000 (Note-1)
₹930 lies between ₹937.490 (at 18%) and ₹918.000 (at 19%). In order to get the exact answer, we
can interpolate the 𝑘𝑝 as follows—
𝑃𝑉𝐿𝐷𝑅 − 𝐶𝐼0 𝑜𝑟 𝑆𝑉 937.490 − 930
𝑘𝑝 = 𝐿𝐷𝑅 + × |𝐷𝑖𝑓𝑓𝑒𝑟𝑒𝑛𝑐𝑒 𝑜𝑓 𝑅𝑎𝑡𝑒𝑠| = 18 + ×1
𝑃𝑉𝐿𝐷𝑅 − 𝑃𝑉𝐻𝐷𝑅 937.490 − 918.000
= 18 + 0.3843 = 18.3843
If we calculate the present value of cash outflows at 18.3843% then it will be ₹930.051₹930.
Note – 1:
Year 𝑫𝒑(𝟏−𝟓) 𝑪𝑶𝑷𝟏−𝟓 𝑫𝒑(𝟏−𝟓) + 𝑪𝑶𝑷(𝟏−𝟓) PVF @ 18% PVF @ 19% PV @ 18% PV @ 19%
1 150 200 350 0.847 0.840 296.450 294.000
2 120 200 320 0.718 0.706 229.760 225.920
3 90 200 290 0.609 0.593 176.610 171.970
4 60 200 260 0.516 0.499 134.160 129.740
5 30 200 230 0.437 0.419 100.510 96.370
Present value of cash outflows 937.490 918.000
Note – 2:
Sales value =1,000 − 50 − 20 = 930

Shortcut method
Shortcut method cannot be applied because debenture is redeemable in installments.

Chapter 7, Cost of Capital: 13


Example 7
Face value of an equity share is ₹10 (market price is ₹240). Dividend paid is ₹15 per share.
Calculate the cost of equity.

Solution
Dividend model shall be used to calculate the 𝑘𝑒 —
𝐷 15
𝑘𝑒 = ⇒ 𝑘𝑒 = ⇒ 𝑘𝑒 = 0.0625
𝑃0 240

Example 8
Face value of an equity share is ₹10 (market price is ₹240). Dividend paid is ₹15 per share.
Growth rate is 6%. Calculate the cost of equity.

Solution
Dividend growth model shall be used to calculate the 𝑘𝑒 —
𝐷1 15.90
𝑘𝑒 = + 𝑔 ⇒ 𝑘𝑒 = + 0.06 ⇒ 𝑘𝑒 = 0.12625
𝑃0 240

𝐷1 = 𝐷0 (1 + 𝑔)1 ⇒ 𝐷1 = 15(1 + 0.06) ⇒ 𝐷1 = 15.90

Example 9 (Illustration 9)
Raja Ltd. is planning to issue new equity shares (face value ₹10 per share). The dividend declared
by the company in the last 5 years is as under—
Years Dividend (₹)
1 10.00
2 11.50
3 12.10
4 12.90
5 13.60
Floatation cost may be taken as 4% of the selling price. If the current market price is ₹130 per share
then calculate cost of existing equity and new equity.

Solution
𝐷1 𝐷0 (1 + 𝑔) 13.60(1 + 0.08)
𝑘𝑒 (𝐸𝑥𝑖𝑠𝑡𝑖𝑛𝑔) = + 𝑔 ⇒ 𝑘𝑒 = + 𝑔 ⇒ 𝑘𝑒 = + 0.08 = 0.1930
𝑃0 𝑃0 130
Market price is the selling price = 130*4% = 5.2
(FLOATATION COST) that is subtracted from 130 to get P0
𝐷1 13.60(1 + 0.08)
𝑘𝑒 (𝑁𝑒𝑤) = + 𝑔 ⇒ 𝑘𝑒 = + 0.08 = 0.1977
𝑃0 130(1 − 0.04)

Calculate the growth factor—


𝐿𝑎𝑡𝑒𝑠𝑡 𝐷𝑖𝑣𝑖𝑑𝑒𝑛𝑑 13.60
𝐺𝑟𝑜𝑤𝑡ℎ 𝑓𝑎𝑐𝑡𝑜𝑟 = ⇒ 𝐺𝑟𝑜𝑤𝑡ℎ 𝑓𝑎𝑐𝑡𝑜𝑟 = ⇒ 𝐺𝑟𝑜𝑤𝑡ℎ 𝑓𝑎𝑐𝑡𝑜𝑟 = 1.36
𝐼𝑛𝑖𝑡𝑖𝑎𝑙 𝐷𝑖𝑣𝑖𝑑𝑒𝑛𝑑 10.00
Now locate the growth factor i.e., 1.36 in the table A-1 corresponding to the period 𝑛 − 1 and note
TABLE IN: down the percentage corresponding to this growth factor. The percentage is 8% and this is the
CAPITAL
BUDGETING growth rate.
PDF PG. 91
Example 10
Current market price of a share is ₹60. Expected dividend is ₹6 per share. Cost of equity for
companies of similar risk is 18%. What is the growth rate?
What is expected to happen if due to some adverse development in the capital market, the growth
rate projection is revised down to 5%?
(B. Com Honors, Delhi University, 2006)

Chapter 7, Cost of Capital: 14


Solution

(a) Growth rate


𝐷1 6
𝑘𝑒 = + 𝑔 ⇒ 0.18 = + 𝑔 ⇒ 0.18 = 0.10 + 𝑔 ⇒ 𝑔 = 0.08
𝑃0 60

(b) If the growth rate revised down to 5%


𝐷1 5.838
𝑃0 = ⇒ 𝑃0 = ⇒ 𝑃0 = 44.91
𝑘𝑒 − 𝑔 0.18 − 0.05

𝐷1 is already given in the question is ₹6 per share. Calculate 𝐷0 as follows—


6
𝐷1 = 𝐷0 (1 + 𝑔)1 ⇒ 6 = 𝐷0 (1 + 0.08) ⇒ 𝐷0 = ⇒ 𝐷0 = 5.56
1.08
Now calculate 𝐷1 using 𝐷0 and revised growth rate of 5% as follows—
𝐷1 = 𝐷0 (1 + 𝑔)1 ⇒ 𝐷1 = 5.56(1 + 0.05) ⇒ 𝐷1 = 5.838

Example 11
The current market price of an equity share is ₹95. The expected dividend per share is ₹5. The
dividend is expected to grow at 6%. Calculate the cost of equity. Also calculate the market price of
the share at the end of year 1 and 2 assuming constant growth rate of dividend.

Solution
𝐷1 5 P0 - MARKET PRICE AT THE END OF CURRENT
𝑘𝑒 = + 𝑔 ⇒ 𝑘𝑒 = + 0.06 ⇒ 𝑘𝑒 = 0.1126 P1 - MARKET PRICE AT THE END OF 1ST YEAR
𝑃0 95 P2 - MARKET PRICE AT THE END OF 2ND YEAR

𝐷2 5.30
𝑃1 = ⇒ 𝑃1 = ⇒ 𝑃1 = 100.76
𝑘𝑒 − 𝑔 0.1126 − 0.06
𝐷3 5.62
𝑃2 = ⇒ 𝑃2 = ⇒ 𝑃2 = 106.84
𝑘𝑒 − 𝑔 0.1126 − 0.06

𝐷2 = 𝐷0 (1 + 𝑔)2 𝑜𝑟 𝐷1 (1 + 𝑔) ⇒ 𝐷2 = 5(1 + 0.06) ⇒ 𝐷2 = 5.30


𝐷3 = 𝐷0 (1 + 𝑔)3 𝑜𝑟 𝐷1 (1 + 𝑔)2 ⇒ 𝐷3 = 5(1 + 0.06)2 ⇒ 𝐷3 = 5.62

Example 12
Following equation is applicable in case of varying growth rates—
𝑛 ∞
𝐷0 (1 + 𝑔1 )𝑡 𝐷n (1 + 𝑔2 )𝑡−𝑛
𝑃0 = ∑ + ∑ 𝑜𝑟 𝑃0
(1 + 𝑘𝑒 )𝑡 (1 + 𝑘𝑒 )𝑡
𝑡=1 𝑡=𝑛+1
𝑛
𝐷0 (1 + 𝑔1 )𝑡 𝐷𝑛+1 1
=∑ + ×
(1 + 𝑘𝑒 )𝑡 𝑘𝑒 − 𝑔2 (1 + 𝑘𝑒 )𝑛(𝑆𝑒𝑒 𝑁𝑜𝑡𝑒)
𝑡=1

Suppose growth rate is 10% per annum for the first 5 years, then 15% per annum for the next 5
years and then 20% per annum infinitely, then equation would be as follows—
5 10 ∞
𝐷0 (1 + 0.10)𝑡 𝐷5 (1 + 0.15)𝑡−5 𝐷10 (1 + 0.20)𝑡−10
𝑃0 = ∑ + ∑ + ∑ 𝑜𝑟
(1 + 𝑘𝑒 )𝑡 (1 + 𝑘𝑒 )𝑡 (1 + 𝑘𝑒 )𝑡
𝑡=1 𝑡=5+1 𝑡=10+1
5 10
𝐷0 (1 + 0.10)𝑡 𝐷5 (1 + 0.15)𝑡−5
𝑃0 = ∑ 𝑡
+ ∑
(1 + 𝑘𝑒 ) (1 + 𝑘𝑒 )𝑡
𝑡=1 𝑡=5+1
𝐷11 1 𝐷10 (1 + 0.20)𝑡−10 1
+[ × 𝑜𝑟 × ]
𝑘𝑒 − 0.20 (1 + 𝑘𝑒 )𝑛 𝑘𝑒 − 0.20 (1 + 𝑘𝑒 ) (𝑆𝑒𝑒 𝑁𝑜𝑡𝑒)
𝑛

Note: The value of 𝑛 will be the one which has been used in the second last expression of the
equation.

Chapter 7, Cost of Capital: 15


Current dividend per share is ₹4.24. Growth rate is 18% for 5 years and then 12% for ever.
Cost of equity share capital is 14%. Calculate the current market price of the share.

Solution
𝑛 ∞ 5 ∞
𝐷0 (1 + 𝑔1 )𝑡 𝐷n (1 + 𝑔2 )𝑡−𝑛 𝐷0 (1 + 0.18)𝑡 𝐷5 (1 + 0.12)𝑡−5
𝑃0 = ∑ + ∑ ⇒ 𝑃0 = ∑ + ∑
(1 + 𝑘𝑒 )𝑡 (1 + 𝑘𝑒 )𝑡 (1 + 0.14)𝑡 (1 + 0.14)𝑡
𝑡=1 𝑡=𝑛+1 𝑡=1 𝑡=6
or
5
𝐷0 (1 + 0.18)𝑡 𝐷5+1 1
𝑃0 = ∑ 𝑡
+ ×
(1 + 0.14) 0.14 − 0.12 (1 + 0.14)5(𝑆𝑒𝑒 𝑁𝑜𝑡𝑒)
𝑡=1
5
4.24(1 + 0.18)𝑡 𝐷6 1
⇒ 𝑃0 = ∑ + ×
(1 + 0.14)𝑡 0.14 − 0.12 (1 + 0.14)5
𝑡=1
4.24(1 + 0.18)1 4.24(1 + 0.18)2 4.24(1 + 0.18)3 4.24(1 + 0.18)4
⇒ 𝑃0 = + + +
(1 + 0.14)1 (1 + 0.14)2 (1 + 0.14)3 (1 + 0.14)4
5
4.24(1 + 0.18) 10.86 1
+ 5
+ ×
(1 + 0.14) 0.14 − 0.12 (1 + 0.14)5(𝑆𝑒𝑒 𝑁𝑜𝑡𝑒)
⇒ 𝑃0 = 305.80

𝐷6 = 𝐷0 (1 + 𝑔1 )5 (1 + 𝑔2 ) ⇒ 𝐷6 = 4.24(1 + 0.18)5 (1 + 0.12) ⇒ 𝐷6 = 10.86


𝑜𝑟
𝐷6 = 𝐷5 (1 + 𝑔2 ) ⇒ 𝐷6 = 9.70(1 + 0.12) ⇒ 𝐷6 = 10.86
Note: The value of 𝑛 is the one which has been used in the second last expression of the equation.

Example 13 (Example 12)


Airocon Ltd. is planning to issue new equity shares (face value ₹10 per share). The dividend
declared by the company in the last 5 years is as under—
Years Dividend (₹)
1 09.50
2 10.00
3 10.50
4 11.00
5 11.50
6 12.00
Floatation cost may be taken at ₹4 per share. If the current market price is ₹140 per share then
calculate cost of existing equity and new equity.

Solution
𝐷1 12(1 + 0.05)
𝑘𝑒 (𝐸𝑥𝑖𝑠𝑡𝑖𝑛𝑔) = + 𝑔 ⇒ 𝑘𝑒 = + 0.05 = 0.14
𝑃0 140

𝐷1 12(1 + 0.05)
𝑘𝑒 (𝑁𝑒𝑤) = + 𝑔 ⇒ 𝑘𝑒 = + 0.05 = 0.1426
𝑃0 140 − 4
Calculate the growth factor— DOUBT
𝐿𝑎𝑡𝑒𝑠𝑡 𝐷𝑖𝑣𝑖𝑑𝑒𝑛𝑑 12
𝐺𝑟𝑜𝑤𝑡ℎ 𝑓𝑎𝑐𝑡𝑜𝑟 = ⇒ 𝐺𝑟𝑜𝑤𝑡ℎ 𝑓𝑎𝑐𝑡𝑜𝑟 = ⇒ 𝐺𝑟𝑜𝑤𝑡ℎ 𝑓𝑎𝑐𝑡𝑜𝑟 = 1.263
𝐼𝑛𝑖𝑡𝑖𝑎𝑙 𝐷𝑖𝑣𝑖𝑑𝑒𝑛𝑑 9.50
Now locate the growth factor i.e., 1.263 in the table A-1 corresponding to the period 𝑛 − 1 and note
down the percentage corresponding to this growth factor. The percentage is 5% and this is the
growth rate.

Example 14
A company is planning to declare a dividend of ₹20 per share next year. Growth rate is 6%.
Current market price of a share is ₹120. What is cost of equity? Also find out the price of the share at
the end of the year 2.
Chapter 7, Cost of Capital: 16
Solution
𝐷1 20
𝑘𝑒 = + 𝑔 ⇒ 𝑘𝑒 = + 0.06 ⇒ 𝑘𝑒 = 0.2267
𝑃0 120

𝐷3 𝐷1 (1 + 𝑔)2 20(1 + 0.06)2


𝑃2 = ⇒ 𝑃2 = ⇒ 𝑃2 = ⇒ 𝑃2 = 134.81
𝑘𝑒 − 𝑔 𝑘𝑒 − 𝑔 0.2267 − 0.06

Example 15
The present market price of a share is ₹100. The company’s present earnings are ₹20,00,000.
Number of shares outstanding are 2,00,000. The company wants to raise additional funds of
₹6,00,000. The floatation cost is ₹10 (10% of the market price) per share and the company can sell
shares at a discount of 10%. Find out the cost of equity.

Solution
In this question the price earning model shall be used.
𝐸𝑃𝑆 10
𝑘𝑒 = ⇒ 𝑘𝑒 = ⇒ 𝑘𝑒 = 0.125
𝑃0 𝑜𝑟 𝑀𝑃 80
𝐸𝑎𝑟𝑛𝑖𝑛𝑔𝑠 𝑜𝑓 𝑡ℎ𝑒 𝑐𝑜𝑚𝑝𝑎𝑛𝑦 20,00,000
𝐸𝑃𝑆 = ⇒ 𝐸𝑃𝑆 = = ₹10 𝑝𝑒𝑟 𝑠ℎ𝑎𝑟𝑒
𝑁𝑢𝑚𝑏𝑒𝑟 𝑜𝑓 𝑒𝑞𝑢𝑖𝑡𝑦 𝑠ℎ𝑎𝑟𝑒𝑠 𝑜𝑢𝑡𝑠𝑡𝑎𝑛𝑑𝑖𝑛𝑔 2,00,000

1 1
𝑘𝑒 = ⇒ 𝑘𝑒 = ⇒ 𝑘𝑒 = 0.125
𝑃/𝐸 𝑅𝑎𝑡𝑖𝑜 8
𝑃 0 𝑜𝑟 𝑀𝑃 80
𝑃/𝐸 𝑅𝑎𝑡𝑖𝑜 = ⇒ 𝑃/𝐸 𝑅𝑎𝑡𝑖𝑜 = ⇒ 𝑃/𝐸 𝑅𝑎𝑡𝑖𝑜 = 8 𝑡𝑖𝑚𝑒𝑠
𝐸𝑎𝑟𝑛𝑖𝑛𝑔 𝑝𝑒𝑟 𝑠ℎ𝑎𝑟𝑒 10

𝑃 0 𝑜𝑟 𝑀𝑃 = 𝐶𝑢𝑟𝑟𝑒𝑛𝑡 𝑚𝑎𝑟𝑘𝑒𝑡 𝑝𝑟𝑖𝑐𝑒 − 𝐹𝑙𝑜𝑎𝑡𝑎𝑡𝑖𝑜𝑛 𝑐𝑜𝑠𝑡 − 𝐷𝑖𝑠𝑐𝑜𝑢𝑛𝑡 𝑜𝑛 𝑖𝑠𝑠𝑢𝑒


⇒ 𝑃 0 𝑜𝑟 𝑀𝑃 = 100 − 10 − 10
⇒ 𝑃 0 𝑜𝑟 𝑀𝑃 = ₹80

Example 16 (Illustration 13)


A company declared dividend of ₹4 per share last year and expects to maintain growth rate as per
schedule—
Year Growth rate
1 to 3 8% (g1)
4 6% (g2)
5 8% (g3)
6th year onwards 9% (g4)
If the expected rate of return is 15%, advise whether the share should be bought if its price is ₹80?

Solution
In such a question when different dividends and growth rates are given and it is asked to decide
whether to purchase the share at a particular price, then intrinsic value of the equity share shall be
calculated. Then the purchase price shall be compared with this. If the purchase price is less than
the intrinsic value then share should be purchased and vice versa.
Formula for the calculation of the intrinsic value is as follows—
𝐷1 𝐷2 𝐷3 𝐷𝑛 𝐷∞
𝑃0 = 1
+ 2
+ 3
+ ⋯⋯⋯+ 𝑛
+ ⋯⋯⋯+
(1 + 𝑘𝑒 ) (1 + 𝑘𝑒 ) (1 + 𝑘𝑒 ) (1 + 𝑘𝑒 ) (1 + 𝑘𝑒 )∞
𝑛⇒∞
𝐷𝑡
𝑃0 = ∑
(1 + 𝑘𝑒 )𝑡
𝑡=1
Where,
𝑃0 = Intrinsic value of the equity share
𝐷1 , 𝐷2 , 𝐷1 , ⋯ ⋯ ⋯ , 𝐷𝑛 = Dividends in periods 1,2,3, ⋯ ⋯ ⋯ , 𝑛
𝑘𝑒 = Cost of equity

Chapter 7, Cost of Capital: 17


But we have varying growth rates, so, the following equation shall be used—
3 4 5
𝐷0 (1 + 𝑔1 )𝑡 𝐷3 (1 + 𝑔2 )𝑡−3 𝐷4 (1 + 𝑔3 )𝑡−4
𝑃0 = ∑ + ∑ + ∑
(1 + 𝑘𝑒 )𝑡 (1 + 𝑘𝑒 )𝑡 (1 + 𝑘𝑒 )𝑡
𝑡=1 𝑡=3+1 𝑡=4+1
𝐷5 (1 + 𝑔4 )𝑡−4 1
+[ × 𝑛(𝑆𝑒𝑒 𝑁𝑜𝑡𝑒 2)
]
𝑘𝑒 − 𝑔4 (1 + 𝑘𝑒 )
𝐷0 (1 + 0.08)1 𝐷0 (1 + 0.08)2 𝐷0 (1 + 0.08)3 𝐷3 (1 + 0.06)1 𝐷4 (1 + 0.08)1
⇒ 𝑃0 = + + + +
(1 + 0.15)1 (1 + 0.15)2 (1 + 0.15)3 (1 + 0.15)1 (1 + 0.15)1
1
𝐷5 (1 + 0.09) 1
+[ × ]
𝑘𝑒 − 𝑔4 (1 + 𝑘𝑒 )5(𝑆𝑒𝑒 𝑁𝑜𝑡𝑒 2)
4(1 + 0.08)1 4(1 + 0.08)2 4(1 + 0.08)3 5.04(1 + 0.06)1 5.34(1 + 0.08)1
⇒ 𝑃0 = + + + +
(1 + 0.15)1 (1 + 0.15)2 (1 + 0.15)3 (1 + 0.15)4 (1 + 0.15)5
1
5.77(1 + 0.09) 1
+[ × ]
0.15 − 0.09 (1 + 0.15)5
4.32 4.67 5.04 5.34 5.77
⇒ 𝑃0 = 1
+ 2
+ 3
+ 4
+
(1 + 0.15) (1 + 0.15) (1 + 0.15) (1 + 0.15) (1 + 0.15)5
6.29 1
+[ × ]
0.15 − 0.09 (1 + 0.15)5(𝑆𝑒𝑒 𝑁𝑜𝑡𝑒 2)
⇒ 𝑃0 = 3.76 + 3.53 + 3.32 + 3.05 + 2.88 + 52.10
⇒ 𝑃0 = ₹68.64
Note:
1. Various dividends can also be calculated as follows and then can be incorporated in the above
equation—
𝐷0 = 4 (𝐺𝑖𝑣𝑒𝑛)
𝐷1 = 𝐷0 (1 + 𝑔1 )1 ⇒ 4(1 + 0.08)1 ⇒ 4.32
𝐷2 = 𝐷1 (1 + 𝑔2 )1 ⇒ 4. 32(1 + 0.08)1 = 4.67
𝐷3 = 𝐷2 (1 + 𝑔3 )1 ⇒ 4. 67(1 + 0.08)1 = 5.04
𝐷4 = 𝐷3 (1 + 𝑔4 )1 ⇒ 5.04(1 + 0.06)1 = 5.34
𝐷5 = 𝐷4 (1 + 𝑔5 )1 ⇒ 5.34(1 + 0.08)1 = 5.77
𝐷6 = 𝐷5 (1 + 𝑔6 )1 ⇒ 5.77(1 + 0.09)1 = 6.29
2. The value of n will be the one which has been used in the second last expression of the equation.

Alternative solution
Year Dividend (₹) PVAF @ 15% Total PV (₹)
0 —NA— —NA— —NA—
1 4.32 0.870 03.76
2 4.67 0.756 03.53
3 5.04 0.658 03.32
4 5.34 0.572 03.05
5 5.77 0.497 02.88
Total present value of the future dividends: 16.54
𝐷6 𝐷5 (1 + 𝑔4 )1 5.77(1 + 0.09)1
𝑃5 = ⇒ 𝑃5 = ⇒ 𝑃5 = ⇒ 𝑃5 = ₹104.83
𝑘𝑒 − 𝑔4 𝑘𝑒 − 𝑔4 0.15 − 0.09
1 1
𝑃𝑟𝑒𝑠𝑒𝑛𝑡 𝑣𝑎𝑙𝑢𝑒 𝑜𝑓 𝑃5 = 𝑃5 × 5
⇒ ₹104.83 × ⇒ ₹52.10
(1 + 𝑘𝑒 ) (1 + 0.15)5

𝑇𝑜𝑡𝑎𝑙 𝑖𝑛𝑡𝑟𝑖𝑛𝑠𝑖𝑐 𝑣𝑎𝑙𝑢𝑒 = 𝑇𝑜𝑡𝑎𝑙 𝑝𝑟𝑒𝑠𝑒𝑛𝑡 𝑣𝑎𝑙𝑢𝑒 𝑜𝑓 𝑓𝑢𝑡𝑢𝑟𝑒 𝑑𝑖𝑣𝑖𝑑𝑒𝑛𝑑𝑠 + 𝑃𝑟𝑒𝑠𝑒𝑛𝑡 𝑣𝑎𝑙𝑢𝑒 𝑜𝑓 𝑃5
⇒ 𝑇𝑜𝑡𝑎𝑙 𝑖𝑛𝑡𝑟𝑖𝑛𝑠𝑖𝑐 𝑣𝑎𝑙𝑢𝑒 = 16.54 + 52.10
⇒ 𝑇𝑜𝑡𝑎𝑙 𝑖𝑛𝑡𝑟𝑖𝑛𝑠𝑖𝑐 𝑣𝑎𝑙𝑢𝑒 = ₹68.64
Decision: As the intrinsic value of the share is ₹68.64 whereas it is available in the market at ₹80.
So, the market price is more than the intrinsic value of the share. Hence, the share should not be
purchased at ₹80.

Chapter 7, Cost of Capital: 18


Example 17
Growth rate is 10%. The company has paid a dividend of ₹3 per share. The rate of return on
market portfolio is 12% and the risk-free rate of return in the market is 8%. The beta coefficient of
the company is 1.5. What is the equilibrium price per share as per the dividend growth model?
Solution:

Capital asset pricing model


𝑘𝑒 = 𝑅𝑓 + 𝛽(𝐾𝑚 − 𝑅𝑓 )
⇒ 𝑘𝑒 = 0.08 + 1.5(0.12 − 0.08)
⇒ 𝑘𝑒 = 0.08 + 1.5(0.04)
⇒ 𝑘𝑒 = 0.08 + 0.06
⇒ 𝑘𝑒 = 0.14

Equilibrium price as per dividend growth model


𝐷1 𝐷1 𝐷0 (1 + 𝑔)1 3(1 + 0.10)1 3.30
𝑘𝑒 = + 𝑔 ⇒ 𝑃0 = ⇒ 𝑃0 = ⇒ 𝑃0 = ⇒ 𝑃0 =
𝑃0 𝑘𝑒 − 𝑔 𝑘𝑒 − 𝑔 0.14 − 0.10 0.14 − 0.10
⇒ 𝑃0 = ₹82.50

Example 18 (Example 15)


IRR Ajay purchased a share in a company at a cost of ₹200 on January 1, 2016. He sold the share on
January 1, 2021 for ₹252. The yearly dividends received by him were as follows—
Years Dividend (₹)
2016 11.00
2017 11.00
2018 11.50
2019 11.50
2020 11.50 Calculate the cost of equity capital.

Solution
In this question the realized yield approach shall be used. Under this approach we calculate the IRR.
Price paid to purchase the share is taken as cash outflows. Dividends received and selling price of
the share are cash inflows. The process to calculate the IRR has already been discussed in detail in
the chapter Capital Budgeting. Calculation is as follows—
Year Cash outflows Cash inflows is Present value Total present
i.e., purchase dividends and factors @ 10% value
price selling price
January 1, 2016 -200.00 —NA— 1 -200.00
December 31, 2016 -- 11.00 0.909 9.99
December 31, 2017 -- 11.00 0.826 9.09
December 31, 2018 -- 11.50 0.751 8.63
December 31, 2019 -- 11.50 0.683 7.85
December 31, 2020 -- 11.50 0.621 7.14
January 1, 2021 -- 252 0.621 156.49
Net present value -0.810
At 10% the cash outflows of ₹200 are approximately equal to the present value of cash inflows of
₹199.10 or the NPV of ₹0.81 is approximately equal to ₹0. So, 10% is the cost of equity.

Example 19
Ajay wants to purchase a share in a company at a cost of ₹200 on January 1, 2016. The cost of
equity is 10% and growth rate is 5%. The expected yearly dividends to be received by him are as
follows—
Years Dividend (₹)
2016 11.00
2017 11.00
Chapter 7, Cost of Capital: 19
2018 11.50
2019 11.50
2020 11.50 Is it worth buying the share at ₹200?
INTRINSIC VALUE > MARKET VALUE - buy
INTRINSIC VALUE < MV - sell
Solution
In this question the intrinsic value of the share shall be calculated. If the selling price of the share is
more than the intrinsic value of the share then he must sell it else not. Let us calculate the intrinsic
value of the share as follows—
𝐷1 𝐷2 𝐷3 𝐷4 𝐷5 𝑃5
𝑃0 = 1
+ 2
+ 3
+ 4
+ 5
+
(1 + 𝑘𝑒 ) (1 + 𝑘𝑒 ) (1 + 𝑘𝑒 ) (1 + 𝑘𝑒 ) (1 + 𝑘𝑒 ) (1 + 𝑘𝑒 )𝑛
Where,
𝑃0 = Intrinsic value of the equity share
𝐷1 , 𝐷2 , 𝐷1 , ⋯ ⋯ ⋯ , 𝐷𝑛 = Dividends in periods 1,2,3, ⋯ ⋯ ⋯ , 𝑛
𝑘𝑒 = Cost of equity
𝐷6 𝐷5 (1 + 𝑔) 11.50(1 + 0.05) 12.075
DOUBT 𝑃5 = ⇒ ⇒ ⇒ ⇒ ₹241.50
WHAT IS INTRINSIC
𝑘𝑒 − 𝑔 𝑘𝑒 − 𝑔 0.10 − 0.05 0.05
VALUE AND HOW (P5)
is calculated? 11 11 11.50 11.50 11.50 241.50
𝑃0 = 1
+ 2
+ 3
+ 4
+ 5
+
(1 + 0.10) (1 + 0.10) (1 + 0.10) (1 + 0.10) (1 + 0.10) (1 + 0.10)5
⇒ 𝑃0 = 9.99 + 9.09 + 8.63 + 7.85 + 7.14 + 149.95
⇒ 𝑃0 = ₹192.65
Decisions: Purchase price of ₹200 is more than the intrinsic value of ₹192.65. So, Mr. Ajay should
not buy the share at ₹200.

Example 19A (Self) CONCEPT OF GROWTH RATE


XYZ Limited recently paid a dividend of ₹2.00 per share and it is fairly risky company with a cost of
equity of 25% (or 0.25). A summary of dividends and earnings per share is given below—
Year Dividends Earnings
2014 ₹2.00 ₹4.50
2013 ₹1.80 ₹3.50
2012 ₹1.70 ₹4.00
2011 ₹1.40 ₹3.00
2010 ₹1.30 ₹2.50
Any new investment by XYZ Limited is expected to yield a return comparable to the cost of equity.
Show two methods of estimating 𝑔, using the above data and use each of these to calculate a share
price for XYZ Limited.
[Answer: Based on dividends: 𝑔 =11.40% (0.1140), 𝑃0 =₹16.38. Based on retained earnings:
𝑔 = 𝑏. 𝑟 =14% (0.140), 𝑃0 =₹20.73]

Solution
There are 2 methods using which the growth rate can be calculated viz. using dividends and using
retained earnings. Let us calculate the growth rate using both the methods.

Calculation of growth rate using dividends


First of call, calculate the growth factor—
𝐿𝑎𝑡𝑒𝑠𝑡 𝐷𝑖𝑣𝑖𝑑𝑒𝑛𝑑 2
𝐺𝑟𝑜𝑤𝑡ℎ 𝑓𝑎𝑐𝑡𝑜𝑟 = ⇒ ⇒ 1.538461538 ≅ 1.5384
𝐼𝑛𝑖𝑡𝑖𝑎𝑙 𝐷𝑖𝑣𝑖𝑑𝑒𝑛𝑑 1.3
After locating the growth factor in A-1 table (extract is given at the end) we get—
Period 11% 12%
4 1.5181 1.5735
The growth factor i.e., 1.5384 lies between 1.5181 and 1.5735, so, the growth rate lies between 11%
and 12%. In case of two percentages (when growth factor lies between two values then two
percentages will be there) average may be calculated or g can be calculated (interpolated) as
follows—

Chapter 7, Cost of Capital: 20


𝑔 = 𝐻𝑖𝑔ℎ𝑒𝑟 𝑝𝑒𝑟𝑐𝑒𝑛𝑡𝑎𝑔𝑒
𝐹𝑎𝑐𝑡𝑜𝑟 𝑎𝑡 ℎ𝑖𝑔ℎ𝑒𝑟 𝑝𝑒𝑟𝑐𝑒𝑛𝑡𝑎𝑔𝑒 − 𝐺𝑟𝑜𝑤𝑡ℎ 𝑓𝑎𝑐𝑡𝑜𝑟

𝐹𝑎𝑐𝑡𝑜𝑟 𝑎𝑡 ℎ𝑖𝑔ℎ𝑒𝑟 𝑝𝑒𝑟𝑐𝑒𝑛𝑡𝑎𝑔𝑒 − 𝐹𝑎𝑐𝑡𝑜𝑟 𝑎𝑡 𝑙𝑜𝑤𝑒𝑟 𝑝𝑒𝑟𝑐𝑒𝑛𝑡𝑎𝑔𝑒
1.5735 − 1.5384
× |𝐷𝑖𝑓𝑓𝑒𝑟𝑒𝑛𝑐𝑒 𝑜𝑓 𝑟𝑎𝑡𝑒𝑠| ⇒ 12 − × |1| ⇒ 11.3675368
1.5735 − 1.5181
≅ 11.40
or
𝑔 = 𝐿𝑜𝑤𝑒𝑟 𝑝𝑒𝑟𝑐𝑒𝑛𝑡𝑎𝑔𝑒
𝐹𝑎𝑐𝑡𝑜𝑟 𝑎𝑡 𝑙𝑜𝑤𝑒𝑟 𝑝𝑒𝑟𝑐𝑒𝑛𝑡𝑎𝑔𝑒 − 𝐺𝑟𝑜𝑤𝑡ℎ 𝑓𝑎𝑐𝑡𝑜𝑟
+
𝐹𝑎𝑐𝑡𝑜𝑟 𝑎𝑡 𝑙𝑜𝑤𝑒𝑟 𝑝𝑒𝑟𝑐𝑒𝑛𝑡𝑎𝑔𝑒 − 𝐹𝑎𝑐𝑡𝑜𝑟 𝑎𝑡 ℎ𝑖𝑔ℎ𝑒𝑟 𝑝𝑒𝑟𝑐𝑒𝑛𝑡𝑎𝑔𝑒
1.5181 − 1.5384
× |𝐷𝑖𝑓𝑓𝑒𝑟𝑒𝑛𝑐𝑒 𝑜𝑓 𝑟𝑎𝑡𝑒𝑠| ⇒ 11 + × |1| ⇒ 11.3675368
1.5181 − 1.5735
≅ 11.40
So, the growth rate is 11.40%.
Note: The above formulae for the interpolation give the approximate answers.
Now, calculate the share price as follows—
𝐷1 𝐷0 (1 + 𝑔) 2(1 + 0.1140) 2.228
𝑃0 = ⇒ ⇒ ⇒ ⇒ ₹16.38
𝑘𝑒 − 𝑔 𝑘𝑒 − 𝑔 0.25 − 0.1140 0.136
Extract of the Table A-1: Future Value Interest Factors for One Rupee Compounded at 𝒌
Percent for 𝒏 Periods: 𝐹𝑉𝐼𝐹𝑘.𝑛 = (1 + 𝑘)𝑛
Period 11% 12% 13% 14% 15% 16% 20%
1 1.1100 1.1200 1.1300 1.1400 1.1500 1.1600 1.2000
2 1.2321 1.2544 1.2769 1.2996 1.3225 1.3456 1.4400
3 1.3676 1.4049 1.4429 1.4815 1.5209 1.5609 1.7280
4 1.5181 1.5735 1.6305 1.6890 1.7490 1.8106 2.0736
5 1.6851 1.7623 1.8424 1.9254 2.0114 2.1003 2.4883

Calculation of growth rate using retained earnings


First of all, calculate the growth rate as follows—
Formula for growth rate –
𝑔 = 𝑏. 𝑟 (𝑅𝑒𝑡𝑒𝑛𝑡𝑖𝑜𝑛 𝑟𝑎𝑡𝑖𝑜 × 𝑅𝑎𝑡𝑒 𝑜𝑓 𝑟𝑒𝑡𝑢𝑟𝑛)
Where,
𝐸𝑃𝑆 − 𝐷𝑃𝑆(1 + 𝐷𝑡 )
𝑏(𝑅𝑒𝑡𝑒𝑛𝑡𝑖𝑜𝑛 𝑟𝑎𝑡𝑖𝑜) =
𝐸𝑃𝑆
𝐸𝑃𝑆
𝑟(𝑅𝑎𝑡𝑒 𝑜𝑓 𝑟𝑒𝑡𝑢𝑟𝑛) =
𝑃0
As per the statement, any new investment by XYZ Ltd. is expected to yield a return comparable to
the cost of equity which is 25% (first line of the statement). For the calculation purpose we are
using the recent data i.e., year 2014. Let us put the data in the formula—
𝑔 = 𝑏. 𝑟 ⇒ 0.55̅ × 0.25 ⇒ 0.1388̅ ≅ 0.14 𝑜𝑟 14%
𝐸𝑃𝑆 − 𝐷𝑃𝑆(1 + 𝐷𝑡 ) 4.5 − 2 2.5
𝑏= ⇒ ⇒ ⇒ 0.55̅
𝐸𝑃𝑆 4.5 4.5
𝑟 = 0.25 (𝑒𝑞𝑢𝑎𝑙 𝑡𝑜 𝑡ℎ𝑒 𝑐𝑜𝑠𝑡 𝑜𝑓 𝑒𝑞𝑢𝑖𝑡𝑦)
So, the growth rate is 14%.
Now, calculate the share price as follows—
𝐷1 𝐷0 (1 + 𝑔) 2(1 + 0.14) 2.28
𝑃0 = ⇒ ⇒ ⇒ ⇒ ₹20.73
𝑘𝑒 − 𝑔 𝑘𝑒 − 𝑔 0.25 − 0.14 0.11

Example 20
The cost of equity is 25%. Floatation cost is 2% in case company decides to issue new equity
shares. Shareholders of the company are in the tax bracket of 30% and are supposed to pay a
brokerage of 2% for making any new investment. Calculate the cost of retained earnings.

Solution
𝑘𝑟 = 𝑘𝑒 (1 − 𝑡)(1 − 𝑏)(1 − 𝑓)
⇒ 𝑘𝑟 = 0.25(1 − 0.30)(1 − 0.02)(1 − 0.02)
Chapter 7, Cost of Capital: 21
⇒ 𝑘𝑟 = 0.16807

Example 21 (Illustration 4)
Keenam Limited is planning to raise ₹1 crore by the issue of 12% preference share of ₹100 each at
10% discount. The underwriting expenses are expected to be 2%. Find out the cost of preference
share capital in each of the following cases—
(i) If preference shares are irredeemable.
(ii) If preference shares are redeemable at the end of 10th year at 15% premium. Use shortcut
method.
(B. Com. Honors, Delhi University, 2009)

Solution

(i) If preference shares are irredeemable


𝐷𝑝 (1 + 𝐷𝑡 ) 12(1 + 0)
𝑘𝑝 = ⇒ 𝑘𝑝 = ⇒ 𝑘𝑝 = 0.1364
𝑆𝑉 88
𝑆𝑉 = 100 − 10 (𝐷𝑖𝑠𝑐𝑜𝑢𝑛𝑡 @ 10%) − 2 (𝑈𝑛𝑑𝑒𝑟𝑤𝑟𝑖𝑡𝑖𝑛𝑔 𝑒𝑥𝑝𝑒𝑛𝑠𝑒𝑠) ⇒ 𝑆𝑉 = 88

(ii) If preference shares are redeemable at the end of 10th year at 15%
premium (Shortcut method)
𝐷𝑝 (1 + 𝐷𝑡 ) + (𝑓 + 𝑑 + 𝑝𝑟 − 𝑝𝑖 )/𝑁𝑚 𝐷𝑝 (1 + 𝐷𝑡 ) + (𝑅𝑉 − 𝑆𝑉)/𝑁𝑚
𝑘𝑝 = 𝑜𝑟 𝑘𝑝 =
(𝑅𝑉 + 𝑆𝑉)/2 (𝑅𝑉 + 𝑆𝑉)/2
12(1 + 0) + (2 + 10 + 15 − 0)/10
𝑘𝑝 = = 0.1448
(115 + 88)/2
12(1 + 0) + (115 − 88)/10
𝑜𝑟 𝑘𝑝 = = 0.1448
(115 + 88)/2

𝐷𝑝 = 100 × 0.12 = 𝑅𝑠. 12


𝑆𝑉 = 100 − 10(𝐷𝑖𝑠𝑐𝑜𝑢𝑛𝑡 𝑖𝑠 10%) − 2(𝑈𝑛𝑑𝑒𝑟𝑤𝑟𝑖𝑡𝑖𝑛𝑔 𝑒𝑥𝑝𝑒𝑛𝑠𝑒𝑠 𝑎𝑟𝑒 2%) ⇒ 𝑆𝑉 = 88
𝑅𝑉 = 100 + 15(𝑃𝑟𝑒𝑚𝑖𝑢𝑚 𝑜𝑛 𝑟𝑒𝑑𝑒𝑚𝑝𝑡𝑖𝑜𝑛 𝑖𝑠 15%) ⇒ 𝑅𝑉 = 115

Example 22 (Illustration 5)
2,00,000 debentures of ₹250 each are being issued at 5% discount. Coupon rate is 15%. Floatation
costs are likely to be 5% of the face value. Redemption will be after 8 years at a premium of 5%. Tax
rate is 40%. Determine the true cost of this debt.
(B. Com. Honors, Delhi University, 2007)

Solution
𝐼(1 − 𝑡) + (𝑅𝑉 − 𝑆𝑉)/𝑁𝑚 37.50(1 − 0.40) + (262.5 − 225)/8
𝑘𝑑 = ⇒ 𝑘𝑑 =
(𝑅𝑉 + 𝑆𝑉)/2 (262.5 + 225)/2
22.5 + 4.6875
⇒ 𝑘𝑑 = ⇒ 𝑘𝑑 = 0.1115
243.75

𝐼 = 250 × 0.15 = 𝑅𝑠. 37.5


𝑆𝑉 = 250 − 12.5(𝐷𝑖𝑠𝑐𝑜𝑢𝑛𝑡 𝑖𝑠 5%) − 12.5(𝐹𝑙𝑜𝑎𝑡𝑎𝑡𝑖𝑜𝑛 𝑐𝑜𝑠𝑡 𝑖𝑠 5%) ⇒ 𝑆𝑉 = 225
𝑅𝑉 = 250 + 12.5(𝑃𝑟𝑒𝑚𝑖𝑢𝑚 𝑜𝑛 𝑟𝑒𝑑𝑒𝑚𝑝𝑡𝑖𝑜𝑛 𝑖𝑠 5%) ⇒ 𝑅𝑉 = 262.5

Example 23 (Illustration 7)
Assuming that the firm pays income tax at 40% rate, compute the after-tax cost of capital in the
following cases—
(i) 15% preference shares sold at par.
(ii) A perpetual bond sold at par, coupon rate being 15%.
(iii) A ten years 8%, ₹1,000 per bond sold at ₹950.

Chapter 7, Cost of Capital: 22


(iv) An equity share selling at a market price of ₹110 and paying a current dividend of ₹10 per
share which is expected to grow at a rate of 10%.
(B. Com. Honors, Delhi University, 2011)

Solution

(i) 15% preference shares sold at par


𝐷𝑝 (1 + 𝐷𝑡 ) 15(1 + 0)
𝑘𝑝 = ⇒ 𝑘𝑝 = ⇒ 𝑘𝑝 = 0.15
𝑆𝑉 100
The face value of the preference share is not given, so, any amount of the face value can be assumed.
We have assumed the face value as ₹100 per share.

(ii) A perpetual bond sold at par, coupon rate being 15%


𝐼(1 − 𝑡) 15(1 − 0.40)
𝑘𝑑 = ⇒ 𝑘𝑑 = ⇒ 𝑘𝑑 = 0.09
𝑆𝑉 100
The face value of the bond is not given, so, any amount of the face value can be assumed. We have
assumed the face value as ₹100 per bond.

(iii) A ten year 8%, ₹1,000 per bond sold at ₹950


𝐼(1 − 𝑡) + (𝑅𝑉 − 𝑆𝑉)/𝑁𝑚 80(1 − 0.40) + (1,000 − 950)/10
𝑘𝑑 = ⇒ 𝑘𝑑 =
(𝑅𝑉 + 𝑆𝑉)/2 (1,000 + 950)/2
48 + 5
⇒ 𝑘𝑑 = ⇒ 𝑘𝑑 = 0.0543
975

𝐼 = 1,000 × 0.08 = 𝑅𝑠. 80


𝑆𝑉 = 950(𝐺𝑖𝑣𝑒𝑛)

(iv) An equity share selling at a market price of ₹110 and paying a


current dividend of ₹10 per share which is expected to grow at a rate of
10%
𝐷1 11
𝑘𝑒 = + 𝑔 ⇒ 𝑘𝑒 = + 0.10 ⇒ 𝑘𝑒 = 0.20
𝑃0 110

𝐷1 = 𝐷0 (1 + 𝑔)1 ⇒ 𝐷1 = 10(1 + 0.10) ⇒ 𝐷1 = 11

Example 24 (Illustration 11)


Equity shares of A Limited are currently selling in the market at ₹100 per share. Dividends paid in
the last 5 years are ₹4.00; ₹4.25; ₹4.60; ₹4.85 and ₹5.05. The company wants to issue new equity
shares and has been advised to price them at ₹90 per share. Floatation costs are likely to be ₹8 per
share. Calculate the growth rate, cost of existing and new equity for the company.
(B. Com. Honors, Delhi University, 2006)

Solution
𝐷1 5.05(1 + 0.06) 5.353
𝑘𝑒 (𝐸𝑥𝑖𝑠𝑡𝑖𝑛𝑔) = + 𝑔 ⇒ 𝑘𝑒 = + 0.06 ⇒ 𝑘𝑒 = + 0.06 = 0.11353
𝑃0 100 100

𝐷1 5.05(1 + 0.06) 5.353


𝑘𝑒 (𝑁𝑒𝑤) = + 𝑔 ⇒ 𝑘𝑒 = + 0.06 ⇒ 𝑘𝑒 = + 0.06 ⇒ 𝑘𝑒 = 0.1253
𝑃0 90 − 8 82

𝐷1 = 𝐷0 (1 + 𝑔)1 ⇒ 𝐷1 = 5.05(1 + 0.06) ⇒ 𝐷1 = 5.353

Calculate the growth factor—


𝐿𝑎𝑡𝑒𝑠𝑡 𝐷𝑖𝑣𝑖𝑑𝑒𝑛𝑑 5.05
𝐺𝑟𝑜𝑤𝑡ℎ 𝑓𝑎𝑐𝑡𝑜𝑟 = ⇒ 𝐺𝑟𝑜𝑤𝑡ℎ 𝑓𝑎𝑐𝑡𝑜𝑟 = ⇒ 𝐺𝑟𝑜𝑤𝑡ℎ 𝑓𝑎𝑐𝑡𝑜𝑟 = 1.2625
𝐼𝑛𝑖𝑡𝑖𝑎𝑙 𝐷𝑖𝑣𝑖𝑑𝑒𝑛𝑑 4
Chapter 7, Cost of Capital: 23
Now locate the growth factor i.e., 1.2625 in the table A-1 corresponding to the period 𝑛 − 1 and note
down the percentage corresponding to this growth factor. The percentage is 6% and this is the
growth rate.

Example 25 (Illustration 12)


Your company’s share is quoted in the market at ₹20 currently. The company pays a dividend of
₹1 per share and the investor’s market expects a growth rate of 5 percent per year. You are required
to compute—
(i) The company’s equity cost of capital.
(ii) If the company’s cost of capital is 8% and the anticipated growth rate is 5 percent per
annum, calculate market price if the dividend of ₹1 is to be maintained.
(B. Com. Honors, Delhi University, 2006)

Solution

(i) The company’s equity cost of capital.


𝐷1 1(1 + 0.05) 1.05
𝑘𝑒 = + 𝑔 ⇒ 𝑘𝑒 = + 0.05 ⇒ 𝑘𝑒 = + 0.05 = 0.1025
𝑃0 20 20

𝐷1 = 𝐷0 (1 + 𝑔)1 ⇒ 𝐷1 = 1(1 + 0.05) ⇒ 𝐷1 = 1.05

(ii) If the company’s cost of capital is 8% and the anticipated growth rate
is 5 percent per annum, calculate market price if the dividend of ₹1 is to
be maintained.
𝐷1 1
𝑃0 = ⇒ 𝑃0 = ⇒ 𝑃0 = ₹33.33
𝑘𝑒 − 𝑔 0.08 − 0.05

WACC start Example 26 (Illustration 14)


The following figures are taken from the current Balance Sheet of a company—

Capital (Face value of ₹10) 8,00,000
Share premium 2,00,000
Reserves 6,00,000
Shareholders’ funds 16,00,000
12% perpetual debentures 4,00,000
An annual ordinary dividend of ₹2 per share has just been paid. In the past, ordinary dividends have
grown at a rate of 10% per annum and this rate of growth is expected to continue. Annual interest
has recently been paid on the debentures. The ordinary shares are currently quoted at ₹27.50 and
the debentures at 80%. Ignore taxation. You are required to estimate the weighted average cost of
capital (based on market values) for the company.
(B. Com. Honors, Delhi University, 2014)

Solution

(i) Cost of equity


𝐷1 2(1 + 0.10) 2.20
𝑘𝑒 = + 𝑔 ⇒ 𝑘𝑒 = + 0.10 ⇒ 𝑘𝑒 = + 0.10 = 0.18
𝑃0 27.50 27.50

𝐷1 = 𝐷0 (1 + 𝑔)1 ⇒ 𝐷1 = 2(1 + 0.10) ⇒ 𝐷1 = 2.20

(ii) Cost of retained earnings (Share premium and reserves)


𝑘𝑟 = 𝑘𝑒 ⇒ 𝑘𝑟 = 0.18

Chapter 7, Cost of Capital: 24


or
𝐷1 2(1 + 0.10) 2.20
𝑘𝑟 = + 𝑔 ⇒ 𝑘𝑟 = + 0.10 ⇒ 𝑘𝑟 = + 0.10 = 0.18
𝑃0 27.50 27.50

𝐷1 = 𝐷0 (1 + 𝑔)1 ⇒ 𝐷1 = 2(1 + .10) ⇒ 𝐷1 = 2.20

(iii) Cost of debt


𝐼 48,000
𝑘𝑖 (𝐵𝑒𝑓𝑜𝑟𝑒 𝑡𝑎𝑥 𝑐𝑜𝑠𝑡 𝑜𝑓 𝑑𝑒𝑏𝑡) = ⇒ 𝑘𝑖 = ⇒ 𝑘𝑖 = 0.15
𝑆𝑉 3,20,000

𝑆𝑉 = 𝑅𝑠. 4,00,000 × 80% ⇒ 𝑆𝑉 = ₹3,20,000

(iv) Weighted average cost of capital (WACC)


Specific cost of Composite
Market value Weights capital cost
Funds (𝑀𝑉) (𝑤) (𝑘) (𝑤 × 𝑘)
Equity capital 11,00,000 𝑤𝑒 = 0.4365 𝑘𝑒 = 0.18 0.0786
Share premium 2,75,000 𝑤𝑟 = 0.1091 𝑘𝑟 = 0.18 0.0196
Reserves 8,25,000 𝑤𝑟 = 0.3274 𝑘𝑟 = 0.18 0.0589
12% perpetual debentures 3,20,000 𝑤𝑑 = 0.1270 𝑘𝑑 = 0.15 0.0191
25,20,000 1.0000 0.1762
So weighted average cost of capital (WACC) is 0.1762
Notes:
1. Market values of various funds
𝐸𝑞𝑢𝑖𝑡𝑦 𝑠ℎ𝑎𝑟𝑒 𝑐𝑎𝑝𝑖𝑡𝑎𝑙 8,00,000
∎𝑁𝑢𝑚𝑏𝑒𝑟 𝑜𝑓 𝑒𝑞𝑢𝑖𝑡𝑦 𝑠ℎ𝑎𝑟𝑒𝑠 = ⇒ ⇒ 80,000 𝑠ℎ𝑎𝑟𝑒𝑠
𝐹𝑎𝑐𝑒 𝑣𝑎𝑙𝑢𝑒 𝑜𝑓 𝑒𝑞𝑢𝑖𝑡𝑦 𝑠ℎ𝑎𝑟𝑒 ₹10 𝑝𝑒𝑟 𝑠ℎ𝑎𝑟𝑒

∎𝑀𝑎𝑟𝑘𝑒𝑡 𝑣𝑎𝑙𝑢𝑒 𝑜𝑓 𝑠ℎ𝑎𝑟𝑒ℎ𝑜𝑙𝑑𝑒𝑟𝑠 ′ 𝑓𝑢𝑛𝑑𝑠 𝑖𝑠 = 80,000 𝑠𝑎ℎ𝑟𝑒𝑠 × ₹27.50 ⇒ ₹22,00,000

The market value of shareholders’ fund shall be divided in the ratio of book value of funds.
22,00,000
∎𝐸𝑞𝑢𝑖𝑡𝑦 𝑐𝑎𝑝𝑖𝑡𝑎𝑙 = × 8,00,000 ⇒ ₹11,00,000
16,00,000
22,00,000
∎𝑆ℎ𝑎𝑟𝑒 𝑝𝑟𝑒𝑚𝑖𝑢𝑚 = × 2,00,000 ⇒ ₹2,75,000
16,00,000
22,00,000
∎𝑅𝑒𝑠𝑒𝑟𝑣𝑒𝑠 = × 6,00,000 ⇒ ₹8,25,000
16,00,000
∎12% 𝑝𝑒𝑟𝑝𝑒𝑡𝑢𝑎𝑙 𝑑𝑒𝑏𝑒𝑛𝑡𝑢𝑟𝑒𝑠 = 4,00,000 × 80% ⇒ ₹3,20,000
2. Calculation of weights
11,00,000
∎𝑤𝑒(𝐸𝑞𝑢𝑖𝑡𝑦 𝑐𝑎𝑝𝑖𝑡𝑎𝑙) = = 0.4365
25,20,000
2,75,000
∎𝑤𝑟(𝑆ℎ𝑎𝑟𝑒 𝑝𝑟𝑒𝑚𝑖𝑢𝑚 𝑜𝑟 𝑟𝑒𝑡𝑎𝑖𝑛𝑒𝑑 𝑒𝑎𝑟𝑛𝑖𝑛𝑔𝑠) = = 0.1091
25,20,000
8,25,000
∎𝑤𝑟(𝑅𝑒𝑠𝑒𝑟𝑣𝑒𝑠 𝑜𝑟 𝑟𝑒𝑡𝑎𝑖𝑛𝑒𝑑 𝑒𝑎𝑟𝑛𝑖𝑛𝑔𝑠) = = 0.3274
25,20,000
3,20,000
∎𝑤𝑑(12% 𝑃𝑒𝑟𝑝𝑒𝑡𝑢𝑎𝑙 𝑑𝑒𝑏𝑡) = = 0.1270
25,20,000
3. Share premium is also a reserve.

Example 27 (Illustration 15)


The following is the capital structure of Blue Cart Limited—

Equity shares: 20,000 shares of ₹100 each 20,00,000
9% preference shares of ₹100 each 5,00,000
8% Debentures of ₹100 each 5,00,000

Chapter 7, Cost of Capital: 25


The market price of the company’s equity share is ₹125 and it is expected that a dividend of ₹12 per
share would be declared after 1 year. Dividend growth rate is 8%.
(i) If the company is in the 40% tax bracket, compute the weighted average cost of capital.
(ii) Assuming that in order to finance an expansion plan, the company intends to borrow a fund
of ₹20,00,000 bearing 11% rate of interest. What will be the company’s revised weighted
average cost of capital? This financing decision is expected to increase the dividend from
₹12 to ₹15 per share. However, the market price of equity share is expected to decline from
₹125 to ₹110 per share.
(B. Com. Honors, Delhi University, 2015)

Solution

(i) If the company is in the 40% tax bracket


(i) Cost of equity
𝐷1 12
𝑘𝑒 = + 𝑔 ⇒ 𝑘𝑒 = + 0.08 ⇒ 𝑘𝑒 = 0.1760
𝑃0 125

(ii) Cost of preference share capital


𝐷𝑝 (1 + 𝐷𝑡 ) 9(1 + 0)
𝑘𝑝 = ⇒ 𝑘𝑝 = ⇒ 𝑘𝑝 = 0.09
𝑆𝑉 100

(iii) Cost of debt


𝐼(1 − 𝑡) 40,000(1 − 0.40)
𝑘𝑑 = ⇒ 𝑘𝑑 = ⇒ 𝑘𝑑 = 0.048
𝑆𝑉 5,00,000

(iv) Weighted average cost of capital (WACC)


Specific cost of Composite
Amount Weights capital cost
Funds (₹) (𝑤) (𝑘) (𝑤 × 𝑘)
Equity capital 20,00,000 𝑤𝑒 = 0.6667 𝑘𝑒 = 0.1760 0.1173392
9% preference share capital 5,00,000 𝑤𝑝 = 0.1667 𝑘𝑝 = 0.0900 0.0150030
8% debentures 5,00,000 𝑤𝑑 = 0.1666 𝑘𝑑 = 0.0480 0.0080016
30,00,000 1.0000 0.1403438
So weighted average cost of capital (WACC) is 0.14034
Calculation of weights
20,00,000
∎𝑤𝑒 = = 0.6667
30,00,000
5,00,000
∎𝑤𝑝 = = 0.1667
30,00,000
5,00,000
∎𝑤𝑑 = = 0.1667
30,00,000

(ii) When additional debt of ₹20,00,000 is raised


(i) New cost of equity
𝐷1 15
𝑘𝑒 = + 𝑔 ⇒ 𝑘𝑒 = + 0.08 ⇒ 𝑘𝑒 = 0.2164
𝑃0 110

(ii) Cost of preference share capital


𝐷𝑝 (1 + 𝐷𝑡 ) 9(1 + 0)
𝑘𝑝 = ⇒ 𝑘𝑝 = ⇒ 𝑘𝑝 = 0.09
𝑆𝑉 100

(iii) Cost of existing debt


Chapter 7, Cost of Capital: 26
𝐼(1 − 𝑡) 40,000(1 − 0.40)
𝑘𝑑 = ⇒ 𝑘𝑑 = ⇒ 𝑘𝑑 = 0.048
𝑆𝑉 5,00,000

(iii) Cost of additional debt


𝐼(1 − 𝑡) 2,20,000(1 − 0.40)
𝑘𝑑 = ⇒ 𝑘𝑑 = ⇒ 𝑘𝑑 = 0.066
𝑆𝑉 20,00,000

(iv) Weighted average cost of capital (WACC)


Specific cost of Composite
Amount Weights capital cost
Funds (₹) (𝑤) (𝑘) (𝑤 × 𝑘)
Equity capital 20,00,000 𝑤 = 0.40 𝑘𝑒 = 0.2164 0.08656
9% preference share capital 5,00,000 𝑤𝑝 = 0.10 𝑘𝑝 = 0.0900 0.00900
8% debentures 5,00,000 𝑤𝑑 = 0.10 𝑘𝑑 = 0.0480 0.00480
11% term loan 20,00,000 𝑤𝑑 = 0.40 𝑘𝑑 = 0.0660 0.02640
50,00,000 1.00 0.12676
So weighted average cost of capital (WACC) is 0.12676
Calculation of weights
20,00,000
∎𝑊𝑒 = = 0.40
50,00,000
5,00,000
∎𝑊𝑝 = = 0.10
50,00,000
5,00,000
∎𝑊𝑑(𝐸𝑥𝑖𝑠𝑡𝑖𝑛𝑔 𝑑𝑒𝑏𝑡) = = 0.10
40,00,000
20,00,000
∎𝑊𝑑(𝐴𝑑𝑑𝑖𝑡𝑖𝑜𝑛𝑎𝑙 𝑑𝑒𝑏𝑡) = = 0.40
50,00,000

Example 28 (Illustration 16)


The following are the book values in the capital structure of XYZ Limited—

Equity shares (Face value of ₹10 each) 15,00,000
8% debentures (Face value of ₹100 each) 4,00,000
12% preference shares (Face value of ₹100 each) 4,00,000
10% term loan 7,00,000
30,00,000
The company paid a dividend of ₹3 per share last year. The dividend is expected to grow at 5% per
annum. The tax rate applicable to the company is 30%. All securities are traded in the capital
markets and recent market prices are—equity shares ₹15; debentures ₹80 and preference shares
₹100.
Find out the weighted average cost of capital using book value weights and market value weights.

(B. Com. Honors, Delhi University, 2017)

Solution
(i) Cost of equity
𝐷1 3.15
𝑘𝑒 = + 𝑔 ⇒ 𝑘𝑒 = + 0.05 ⇒ 𝑘𝑒 = 0.26
𝑃0 15
𝐷1 = 𝐷0 (1 + 𝑔)1 ⇒ 𝐷1 = 3(1 + .05) ⇒ 𝐷1 = 3.15

(ii) Cost of debt


𝐼(1 − 𝑡) 8(1 − 0.30)
𝑘𝑑 = ⇒ 𝑘𝑑 = ⇒ 𝑘𝑑 = 0.070
𝑆𝑉 𝑖. 𝑒. 𝑀𝑎𝑟𝑘𝑒𝑡 𝑝𝑟𝑖𝑐𝑒 80

Chapter 7, Cost of Capital: 27


(iii) Cost of preference share capital
𝐷𝑝 (1 + 𝐷𝑡 ) 12(1 + 0)
𝑘𝑝 = ⇒ 𝑘𝑝 = ⇒ 𝑘𝑝 = 0.12
𝑆𝑉 100

(iv) Cost of term loan


𝐼(1 − 𝑡) 70,000(1 − 0.30)
𝑘𝑑 = ⇒ 𝑘𝑑 = ⇒ 𝑘𝑑 = 0.070
𝑆𝑉 7,00,000

(v) Weighted average cost of capital (WACC) using book value weights
Specific cost of Composite
Book value Weights capital cost
Funds (𝐵𝑉) (𝑤) (𝑘) (𝑤 × 𝑘)
Equity shares 15,00,000 0.50 𝑘𝑒 = 0.26 0.1300
8% debentures 4,00,000 0.13 𝑘𝑑 = 0.07 0.0156
12% preference shares 4,00,000 0.13 𝑘𝑝 = 0.12 0.0091
10% term loan 7,000,000 0.24 𝑘𝑑 = 0.07 0.0168
30,00,000 1.00 0.1715
So weighted average cost of capital (WACC) is 0.1715

(vi) Weighted average cost of capital (WACC) using market value weights
Specific cost of Composite
Market value Weights capital cost
Funds (𝑀𝑉) (𝑤) (𝑘) (𝑤 × 𝑘)
Equity shares (See note 1) 22,50,000 0.613 𝑘𝑒 = 0.26 0.15938
8% debentures (See note 2) 3,20,000 0.109 𝑘𝑑 = 0.12 0.01308
12% preference shares 4,00,000 0.087 𝑘𝑝 = 0.07 0.00609
10% term loan 7,00,000 0.191 𝑘𝑑 = 0.07 0.01337
36,70,000 1.000 0.19192
So weighted average cost of capital (WACC) is 0.19192
Notes:
1. 𝑀𝑎𝑟𝑘𝑒𝑡 𝑣𝑎𝑙𝑢𝑒 𝑜𝑓 𝑒𝑞𝑢𝑖𝑡𝑦 = 1,50,000 𝑠ℎ𝑎𝑟𝑒𝑠 × ₹15 𝑝𝑒𝑟 𝑠ℎ𝑎𝑟𝑒 ⇒ ₹22,50,0000
2. 𝑀𝑎𝑟𝑘𝑒𝑡 𝑣𝑎𝑙𝑢𝑒 𝑜𝑓 𝑑𝑒𝑏𝑒𝑛𝑡𝑢𝑟𝑒𝑠 = 4,000 𝑑𝑒𝑏𝑒𝑛𝑡𝑢𝑟𝑒𝑠 × ₹80 𝑝𝑒𝑟 𝑑𝑒𝑏𝑒𝑛𝑡𝑢𝑟𝑒 ⇒ ₹3,20,000

Example 29 (Illustration 18)


The balance sheet of M/s XYZ Company shows the following items as at 31st December 2018—

Paid up capital 4,00,000 equity shares of ₹10 each 40,00,000
Reserves and surplus 60,00,000
15% non-convertible debentures 20,00,000
14% institutional loan 60,00,000
Other relevant information about the company is given below—
Year ended on Dividend per share Earnings per share Market price per share
31st December (₹) (₹) (₹)
2018 7 7.50 50
2017 3 6.00 40
2016 4 4.50 30
Calculate weighted average cost of capital. Assume a tax rate of 50%.
(B. Com. Honors, Delhi University, 2000)

Solution
(i) Cost of equity (Using price earning method)
𝐸𝑃𝑆 7.50
2018: 𝑘𝑒 = ⇒ 𝑘𝑒 = ⇒ 0.15
𝑃0 𝑜𝑟 𝑀𝑃 50
Chapter 7, Cost of Capital: 28
𝐸𝑃𝑆 6.00
2017: 𝑘𝑒 = ⇒ 𝑘𝑒 = ⇒ 0.15
𝑃0 𝑜𝑟 𝑀𝑃 40
𝐸𝑃𝑆 4.50
2016: 𝑘𝑒 = ⇒ 𝑘𝑒 = ⇒ 0.15
𝑃0 𝑜𝑟 𝑀𝑃 30

(ii) Cost of retained earnings


𝑘𝑟 = 𝑘𝑒 𝑠𝑜 𝑘𝑟 = 0.15

(iii) Cost of 15% non-convertible debentures


𝐼(1 − 𝑡) 3,00,000(1 − 0.50)
𝑘𝑑 = ⇒ 𝑘𝑑 = ⇒ 𝑘𝑑 = 0.075
𝑆𝑉 20,00,000

(iv) Cost of 14% institutional loan


𝐼(1 − 𝑡) 8,40,000(1 − 0.50)
𝑘𝑑 = ⇒ 𝑘𝑑 = ⇒ 𝑘𝑑 = 0.070
𝑆𝑉 60,00,000

(v) Weighted average cost of capital (WACC)


Specific cost of Composite
Book value Weights capital cost
Source of capital (𝐵𝑉) (𝑤) (𝑘) (𝑤 × 𝑘)
Equity capital
Reserves and surplus 40,00,000 0.2222 0.150 0.0333300
15% non-convertible 60,00,000 0.3333 0.150 0.0499950
debentures 20,00,000 0.1111 0.075 0.0083325
14% institutional loan 60,00,000 0.3333 0.070 0.0233310
1,80,00,000 1.0000 0.1149885
So weighted average cost of capital is 0.1150

BRILLIANTExample 30 (Illustration 19)


International Foods Limited has the following capital structure—
Particulars Book value (₹) Market value (₹)
Equity share capital (25,000 shares of ₹10 each) 2,50,000 4,50,000
13% Preference share capital (500 shares of ₹100 each) 50,000 45,000
Reserves and surplus 1,50,000 --
14% Debentures (1500 debentures of ₹100 each) 1,50,000 1,45,000
Total 6,00,000 6,40,000
The expected dividend rate per share is ₹1.40 and the dividend per share is expected to grow at a
rate of 8% forever. Preference shares are redeemable after 5 years at par whereas debentures are
redeemable after 6 years at par. The tax rate for the company is 50 percent.
You are required to compute the weighted average cost of capital for the existing capital structure
using market value as weights.
(B. Com. Honors, Delhi University, 1999)

Solution
(i) Cost of equity
𝐷1 1.40
𝑘𝑒 = + 𝑔 ⇒ 𝑘𝑒 = + 0.08 ⇒ 𝑘𝑒 = 0.1577
𝑃0 18
₹4,50,000
𝑃0 = ⇒ 𝑃0 = ₹18 𝑝𝑒𝑟 𝑠ℎ𝑎𝑟𝑒
25,000 𝑒𝑞𝑢𝑖𝑡𝑦 𝑠ℎ𝑎𝑟𝑒𝑠

(ii) Cost of retained earnings


𝑘𝑟 = 𝑘𝑒 𝑠𝑜 𝑘𝑟 = 0.1577

Chapter 7, Cost of Capital: 29


(iii) Cost of preference share capital
𝐷𝑝 (1 + 𝐷𝑡 ) + (𝑅𝑉 − 𝑆𝑉)/𝑁𝑚 13 + (100 − 90)/5
𝑘𝑝 = ⇒ 𝑘𝑝 = = 0.1579
(𝑅𝑉 + 𝑆𝑉)/2 (100 + 90)/2
𝑀𝑎𝑟𝑘𝑒𝑡 𝑣𝑎𝑙𝑢𝑒 𝑜𝑓 𝑝𝑟𝑒𝑓𝑒𝑟𝑒𝑛𝑐𝑒 𝑠ℎ𝑎𝑟𝑒𝑠 ₹45,000
𝑆𝑉 = ⇒ 𝑆𝑉 = ⇒ 𝑆𝑉
𝑁𝑢𝑚𝑏𝑒𝑟 𝑜𝑓 𝑝𝑟𝑒𝑓𝑒𝑟𝑒𝑛𝑐𝑒 𝑠ℎ𝑎𝑟𝑒𝑠 500 𝑝𝑟𝑒𝑓𝑒𝑟𝑒𝑛𝑐𝑒 𝑠ℎ𝑎𝑟𝑒𝑠
= ₹90 𝑝𝑒𝑟 𝑠ℎ𝑎𝑟𝑒

(iv) Cost of debt


𝐼(1 − 𝑡𝑎𝑥) + (𝑅𝑉 − 𝑆𝑉)/𝑁𝑚 14(1 − 0.50) + (100 − 96.67)/6
𝑘𝑑 = ⇒ 𝑘𝑑 = = 0.07683
(𝑅𝑉 + 𝑆𝑉)/2 (100 + 96.67)/2
𝑀𝑎𝑟𝑘𝑒𝑡 𝑣𝑎𝑙𝑢𝑒 𝑜𝑓 𝑑𝑒𝑏𝑒𝑛𝑡𝑢𝑟𝑒𝑠 ₹1,45,000
𝑆𝑉 = ⇒ 𝑆𝑉 = ⇒ 𝑆𝑉
𝑁𝑢𝑚𝑏𝑒𝑟 𝑜𝑓 𝑑𝑒𝑏𝑒𝑛𝑡𝑢𝑟𝑒𝑠 1,500 𝑑𝑒𝑏𝑒𝑛𝑡𝑢𝑟𝑒𝑠
= ₹96.67 𝑝𝑒𝑟 𝑑𝑒𝑏𝑒𝑛𝑡𝑢𝑟𝑒

(v) Weighted average cost of capital (WACC) using market value weights
Market Specific cost of Composite
value Weights capital cost
Source of capital (𝑀𝑉) (𝑤) (𝑘) (𝑤 × 𝑘)
Equity share capital 2,81,250 0.439 0.1577 0.06923
Reserves and surplus 1,68,750 0.264 0.1577 0.04163
13% Preference share capital 45,000 0.070 0.1579 0.01105
14% Debentures 1,45,000 0.227 0.0768 0.01743
6,40,000 1.0000 0.13934
So weighted average cost of capital is 0.13934
Notes:
∎𝐵𝑜𝑜𝑘 𝑣𝑎𝑙𝑢𝑒 𝑜𝑓 𝑒𝑞𝑢𝑖𝑡𝑦 𝑠ℎ𝑎𝑟𝑒 𝑐𝑎𝑝𝑖𝑡𝑎𝑙 𝑎𝑛𝑑 𝑟𝑒𝑠𝑒𝑟𝑣𝑒𝑠 𝑎𝑛𝑑 𝑠𝑢𝑟𝑝𝑙𝑢𝑠 𝑖𝑠 = ₹2,50,000 + ₹1,50,000
⇒ ₹4,00,000
∎𝑀𝑎𝑟𝑘𝑒𝑡 𝑣𝑎𝑙𝑢𝑒 𝑜𝑓 𝑒𝑞𝑢𝑖𝑡𝑦 𝑠ℎ𝑎𝑟𝑒 𝑐𝑎𝑝𝑖𝑡𝑎𝑙 𝑖𝑠 = ₹4,50,000
correct The market value of equity share capital shall be divided in the ratio of book value of equity capital
and reserves and surplus.
4,50,000
∎𝐸𝑞𝑢𝑖𝑡𝑦 𝑐𝑎𝑝𝑖𝑡𝑎𝑙 = × 2,50,000 ⇒ ₹2,81,250
4,00,000
4,50,000
∎𝑅𝑒𝑠𝑒𝑟𝑣𝑒𝑠 𝑎𝑛𝑑 𝑠𝑢𝑟𝑝𝑙𝑢𝑠 = × 1,50,000 ⇒ ₹1,68,750
4,00,000

Example 31 (Illustration 20)


From the following information, calculate the WACC before tax for Keena ABC Limited—
₹ in lakhs
Shareholders’ funds
Equity Share capital 500
Preference share capital 100
Reserves 300
Borrowed funds
Secured loans 800
Unsecured loans (Including inter-corporate deposits) 700
Total funds 2,400
Additional information—
(i) Normal yield on equity shareholders’ funds is 15%. (ke)
(ii) Dividend rate on preference share is 12%. (kp)
(iii) Interest on secured loans is 16.25%. (kd)
(iv) Interest on unsecured loans is 20%. (kd)
(v) Tax rate for ABC Limited is 40%.
(B. Com. Honors, Delhi University, 2004, 2017)

Chapter 7, Cost of Capital: 30


Solution
This is a special question in which it is asked to calculate the before tax WACC. This is contrary to
what we have studied so far. However, in such a case you need to divide the cost of equity share
capital and cost of preference share capital by the (1 − 𝑡𝑎𝑥 𝑟𝑎𝑡𝑒). In case of debt there is no need to
divide by (1 − 𝑡𝑎𝑥 𝑟𝑎𝑡𝑒).
In PSC and ESC, these are calculated after taxes by default,
hence there is a need to remove taxes from these by dividing
(i) Before tax cost of equity them by (1-tax), but debentures are calculated before tax also,
so they must not be required to be divided by (1-tax)
0.15
𝑘𝑒 = ⇒ 𝑘𝑒 = 0.25
(1 − 0.4)

(ii) Before tax cost of preference share capital


0.12
𝑘𝑝 = ⇒ 𝑘𝑝 = 0.20
(1 − 0.4)

(iii) Before tax cost of retained earnings


𝑘𝑒 = 𝑘𝑟 ⇒ 𝑘𝑟 = 0.25

(iv) Before tax cost of secured loans


ASSUME FACE VALUE OF DEBENTURE BE 100
𝑘𝑑 = 0.1625 (𝑔𝑖𝑣𝑒𝑛) INTEREST - 100*16.25% = 16.25
kd - 16.25/100 (I/SV) = 0.1625
(v) Before tax cost of unsecured loans
𝑘𝑑 = 0.20 (𝑔𝑖𝑣𝑒𝑛)

(vi) Before tax weighted average cost of capital


Specific cost Composite
Book value Weights of capital cost
Source of capital (𝐵𝑉) (𝑤) (𝑘) (𝑤 × 𝑘)
Shareholders’ funds
Equity Share capital 500 0.2083 0.2500 0.0520750
Preference share capital 100 0.0417 0.2000 0.0083400
Reserves 300 0.1250 0.2500 0.0312500
Borrowed funds
Secured loans 800 0.3333 0.1625 0.0541613
Unsecured loans 700 0.2917 0.2000 0.0583400
2,400 1.0000 0.2041663
So, before tax weighted average cost of capital is 0.2042

Example 32 (Illustration 21)


Determine the weighted average cost of capital using book value weights based on the following
data—
Book values ₹
14% debentures (₹100 per debenture) 8,00,000
15% preference shares (₹100 per share) 2,00,000
Equity shares (₹10 per share) 10,00,000
Recent market prices of all these securities are—
Debentures-₹110 per debenture; Preference shares-₹120 per preference share and Equity shares-
₹22 per share.
Expected dividend on equity shares at the end of the year is ₹2 per share; anticipated growth rate in
dividends is 7%. The company pays all its earnings in the form of dividends. Corporate tax rate is
40%.
(B. Com. Honors, Delhi University, 2004, 2007)

Chapter 7, Cost of Capital: 31


Solution
(i) Cost of equity
𝐷1 2
𝑘𝑒 = + 𝑔 ⇒ 𝑘𝑒 = + 0.07 ⇒ 𝑘𝑒 = 0.1609
𝑃0 22

(ii) Cost of debt


𝐼(1 − 𝑡) 14(1 − 0.40)
𝑘𝑑 = ⇒ 𝑘𝑑 = ⇒ 𝑘𝑑 = 0.07636
𝑆𝑉 𝑖. 𝑒. 𝑀𝑎𝑟𝑘𝑒𝑡 𝑝𝑟𝑖𝑐𝑒 110

(iii) Cost of preference share capital


𝐷𝑝 (1 + 𝐷𝑡 ) 15(1 + 0)
𝑘𝑝 = ⇒ 𝑘𝑝 = ⇒ 𝑘𝑝 = 0.125
𝑆𝑉 120

(v) Weighted average cost of capital (WACC) using book value weights
Specific cost of Composite
Book value Weights capital cost
Funds (𝑩𝑽) (𝑤) (𝑘) (𝑤 × 𝑘)
14% debentures 8,00,000 0.40 0.0764 0.03056
15% preference shares 2,00,000 0.10 0.1250 0.01250
Equity shares 10,00,000 0.50 0.1609 0.08045
20,00,000 1.00 0.12351
So weighted average cost of capital (WACC) is 0.12351

Example 33 (Illustration 22)


[Same as Example 27] A limited company has the following capital structure

Equity share capital (2,00,000 shares) 40,00,000
6% preference share capital 10,00,000
8% debentures 30,00,000
The market price of the company’s equity share is ₹20. It is expected that company will pay current
dividend of ₹2 per share. It will grow at 7% forever. The tax rate may be presumed at 50%. You are
required to compute the following—
(i) The weighted average cost of capital based on existing capital structure.
(ii) The new weighted average cost of capital if the company raises an additional debt by issuing
10% ₹20,00,000 debentures. This would result in increasing the expected dividend to ₹3
and leave the growth unchanged but the price of share will fall to ₹15 per share.
(B. Com. Honors, Delhi University, 2013, 2016 and similar but modified question in 2015)

Solution

(i) The weighted average cost of capital based on existing capital


structure
(i) Cost of equity
𝐷1 2
𝑘𝑒 = + 𝑔 ⇒ 𝑘𝑒 = + 0.07 ⇒ 𝑘𝑒 = 0.17
𝑃0 20

(ii) Cost of preference share capital


𝐷𝑝 (1 + 𝐷𝑡 ) 60,000(1 + 0)
𝑘𝑝 = ⇒ 𝑘𝑝 = ⇒ 𝑘𝑝 = 0.06
𝑆𝑉 10,00,000

(iii) Cost of debt


Chapter 7, Cost of Capital: 32
𝐼(1 − 𝑡) 2,40,000(1 − 0.50)
𝑘𝑑 = ⇒ 𝑘𝑑 = ⇒ 𝑘𝑑 = 0.04
𝑆𝑉 30,00,000

(iv) Weighted average cost of capital (WACC)


Specific cost of Composite
Amount Weights capital cost
Funds (₹) (𝑤) (𝑘) (𝑤 × 𝑘)
Equity capital 40,00,000 𝑤𝑒 = 0.500 𝑘𝑒 = 0.17 0.0850
6% preference share capital 10,00,000 𝑤𝑝 = 0.125 𝑘𝑝 = 0.06 0.0075
8% debentures 30,00,000 𝑤𝑑 = 0.375 𝑘𝑑 = 0.04 0.0150
80,00,000 1.000 0.1075
So weighted average cost of capital (WACC) is 0.1075
Calculation of weights
40,00,000
∎𝑤𝑒 = = 0.500
80,00,000
10,00,000
∎𝑤𝑝 = = 0.125
80,00,000
30,00,000
∎𝑤𝑑 = = 0.375
80,00,000

(ii) The weighted average cost of capital additional debentures of


₹20,00,000 are issued
(i) New cost of equity
𝐷1 3
𝑘𝑒 = + 𝑔 ⇒ 𝑘𝑒 = + 0.07 ⇒ 𝑘𝑒 = 0.27
𝑃0 15

(ii) Cost of preference share capital


𝐷𝑝 (1 + 𝐷𝑡 ) 60,000(1 + 0)
𝑘𝑝 = ⇒ 𝑘𝑝 = ⇒ 𝑘𝑝 = 0.06
𝑆𝑉 10,00,000

(iii) Cost of existing debt


𝐼(1 − 𝑡) 2,40,000(1 − 0.50)
𝑘𝑑 = ⇒ 𝑘𝑑 = ⇒ 𝑘𝑑 = 0.04
𝑆𝑉 30,00,000

(iii) Cost of additional debt


𝐼(1 − 𝑡) 2,00,000(1 − 0.50)
𝑘𝑑 = ⇒ 𝑘𝑑 = ⇒ 𝑘𝑑 = 0.05
𝑆𝑉 20,00,000

(iv) Weighted average cost of capital (WACC)


Specific cost of Composite
Amount Weights capital cost
Funds (₹) (𝑤) (𝑘) (𝑤 × 𝑘)
Equity capital 40,00,000 𝑤𝑒 = 0.40 𝑘𝑒 = 0.27 0.1080
6% preference share capital 10,00,000 𝑤𝑝 = 0.10 𝑘𝑝 = 0.06 0.0060
8% debentures 30,00,000 𝑤𝑑 = 0.30 𝑘𝑑 = 0.04 0.0120
10% debentures 20,00,000 𝑤𝑑 = 0.20 𝑘𝑑 = 0.05 0.0100
1,00,00,000 1.00 0.1360
So weighted average cost of capital (WACC) is 0.1360
Calculation of weights
40,00,000
∎𝑤𝑒 = = 0.40
1,00,00,000
10,00,000
∎𝑤𝑝 = = 0.10
1,00,00,000
Chapter 7, Cost of Capital: 33
30,00,000
∎𝑤𝑑 = = 0.30
1,00,00,000
20,00,000
∎𝑤𝑑 = = 0.20
1,00,00,000

Example 34 (Illustration 23)


Silver Limited has the following capital structure—

Equity share capital (5,000 shares of ₹100 each) 5,00,000
9% preference shares 1,50,000
12% debentures 3,50,000
10% term loan 5,00,000
The equity share of the company has current market price of ₹105 and company declared a
dividend of ₹9 per share for the next year. The dividend growth rate is 5% per annum. The tax rate
is 50%. Calculate weighted average cost of capital.
(BBS Honors, University of Delhi, 2014)

Solution
(i) Cost of equity
𝐷1 9
𝑘𝑒 = + 𝑔 ⇒ 𝑘𝑒 = + 0.05 ⇒ 𝑘𝑒 = 0.1357
𝑃0 105

(ii) Cost of preference share capital


𝐷𝑝 (1 + 𝐷𝑡 ) 13,500(1 + 0)
𝑘𝑝 = ⇒ 𝑘𝑝 = ⇒ 𝑘𝑝 = 0.09
𝑆𝑉 1,50,000

(iii) Cost of 12% debentures


𝐼(1 − 𝑡) 42,000(1 − 0.50)
𝑘𝑑 = ⇒ 𝑘𝑑 = ⇒ 𝑘𝑑 = 0.06
𝑆𝑉 3,50,000

(iii) Cost of 10% term loan


𝐼(1 − 𝑡) 50,000(1 − 0.50)
𝑘𝑑 = ⇒ 𝑘𝑑 = ⇒ 𝑘𝑑 = 0.05
𝑆𝑉 5,00,000

(iv) Weighted average cost of capital (WACC)


Specific cost of Composite
Amount Weights capital cost
Funds (₹) (𝑤) (𝑘) (𝑤 × 𝑘)
Equity share capital 5,00,000 0.333 0.1357 0.045188
9% preference shares 1,50,000 0.100 0.0900 0.009000
12% debentures 3,50,000 0.233 0.0600 0.013980
10% term loan 5,00,000 0.334 0.0500 0.016700
15,00,000 1.00 0.084868
So weighted average cost of capital (WACC) is 0.08487

Example 35 (Illustration 24)


Bharat Limited has the following capital structure as per its Balance Sheet as at 31st March 2018—
₹ (Lakhs)
Equity share capital (fully paid shares of ₹10 each) 4
18% preference share capital (fully paid shares of ₹100 each) 3
Retained earnings 1
12.5% debentures (fully paid of ₹100 each) 8
12% term loan 4
Chapter 7, Cost of Capital: 34
20
Additional information—
(i) Currently quoted prices in the stock exchange: Equity shares @ ₹64.25; Preference shares @
₹90 each and Debentures @ ₹95.
(ii) For the last year, the company had paid equity dividend of ₹8 per share which is expected to
grow @ 5% per annum forever. The corporate tax rate is 30%.
Calculate weighted average cost of capital using (a) Book value weights; and (b) Market value
weights.
(BBS Honors, Delhi University, 2011)

Solution
(i) Cost of equity
𝐷1 8.4
𝑘𝑒 = + 𝑔 ⇒ 𝑘𝑒 = + 0.05 ⇒ 𝑘𝑒 = 0.1807
𝑃0 64.25
𝐷1 = 𝐷0 (1 + 𝑔)1 ⇒ 𝐷1 = 8(1 + .05) ⇒ 𝐷1 = 8.40

(ii) Cost of retained earnings


𝑘𝑟 = 𝑘𝑒 ⇒ 𝑘𝑟 = 0.1807

(iii) Cost of preference share capital


𝐷𝑝 (1 + 𝐷𝑡 ) 18(1 + 0)
𝑘𝑝 = ⇒ 𝑘𝑝 = ⇒ 𝑘𝑝 = 0.20
𝑆𝑉 90

(iv) Cost of debt


𝐼(1 − 𝑡) 12.5(1 − 0.30)
𝑘𝑑 = ⇒ 𝑘𝑑 = ⇒ 𝑘𝑑 = 0.0921
𝑆𝑉 𝑖. 𝑒. 𝑀𝑎𝑟𝑘𝑒𝑡 𝑝𝑟𝑖𝑐𝑒 95

(v) Cost of term loan


𝐼(1 − 𝑡) 48,000(1 − 0.30)
𝑘𝑑 = ⇒ 𝑘𝑑 = ⇒ 𝑘𝑑 = 0.084
𝑆𝑉 4,00,000

(vi) Weighted average cost of capital (WACC) using book value weights
Specific cost of Composite
Book value Weights capital cost
Funds (𝐵𝑉) (𝑤) (𝑘) (𝑤 × 𝑘)
Equity share capital 4,00,000 0.20 0.1807 0.036140
9% preference share capital 3,00,000 0.15 0.2000 0.030000
Retained earnings 1,00,000 0.05 0.1807 0.009035
12.5% debentures 8,00,000 0.40 0.0921 0.036840
12% term loan 4,00,000 0.20 0.0840 0.016800
20,00,000 1.00 0.128815
So weighted average cost of capital (WACC) is 0.12882

(vii) Weighted average cost of capital (WACC) using market value weights
Specific cost of Composite
Market value Weights capital cost
Funds (𝑀𝑉) (𝑤) (𝑘) (𝑤 × 𝑘)
Equity share capital 20,56,000 0.5140 0.1807 0.092880
9% preference share capital 2,70,000 0.0675 0.2000 0.013500
Retained earnings 5,14,000 0.1285 0.1807 0.023220
12.5% debentures 7,60,000 0.1900 0.0921 0.017499
12% term loan 4,00,000 0.1000 0.0840 0.008400
40,00,00 1.0000 0.155499
Chapter 7, Cost of Capital: 35
So weighted average cost of capital (WACC) is 0.1555
Notes:
∎𝐵𝑜𝑜𝑘 𝑣𝑎𝑙𝑢𝑒 𝑜𝑓 𝑒𝑞𝑢𝑖𝑡𝑦 𝑠ℎ𝑎𝑟𝑒 𝑐𝑎𝑝𝑖𝑡𝑎𝑙 𝑎𝑛𝑑 𝑟𝑒𝑡𝑎𝑖𝑛𝑒𝑑 𝑒𝑎𝑟𝑛𝑖𝑛𝑔𝑠 𝑖𝑠 = ₹4,00,000 + ₹1,00,000
⇒ ₹5,00,000
∎𝑀𝑎𝑟𝑘𝑒𝑡 𝑣𝑎𝑙𝑢𝑒 𝑜𝑓 𝑒𝑞𝑢𝑖𝑡𝑦 𝑠ℎ𝑎𝑟𝑒 𝑐𝑎𝑝𝑖𝑡𝑎𝑙 𝑖𝑠 = 40,000 𝑠ℎ𝑎𝑟𝑒𝑠 × ₹64.25 = ₹25,70,000
The market value of equity share capital shall be divided in the ratio of book value of equity capital
and retained earnings.
25,70,000
∎𝐸𝑞𝑢𝑖𝑡𝑦 𝑐𝑎𝑝𝑖𝑡𝑎𝑙 = × 4,00,000 ⇒ ₹20,56,000
5,00,000
25,70,000
∎𝑅𝑒𝑡𝑎𝑖𝑛𝑒𝑑 𝑒𝑎𝑟𝑛𝑖𝑛𝑔𝑠 = × 1,00,000 ⇒ ₹5,14,000
5,00,000

Example 36 (Illustration 25) Flotation costs are subtracted from MV to get SV


The capital structure of XYZ Limited is as under— Flotation costs = Percentage*MV

9% debentures 2,75,000
11% preference shares 2,25,000
Equity shares (face value of ₹10 per share) 5,00,000
10,00,000
Additional information—
(i) ₹100 per debenture redeemable at par, has 2% floatation cost and 10 years of maturity. The
market price per debenture is ₹105.
(ii) ₹100 per preference share redeemable at par, has 3% floatation cost and 10 years of
maturity. The market price per preference share is ₹106.
(iii) Equity share has ₹4 floatation cost and market price per share of ₹24. The expected
dividend is ₹2 per share with annual growth of 5%. The firm has a practice of paying all
earnings in the form of dividends.
(iv) Corporate income tax rate is 35%.
Required: Calculate weighted average cost of capital (WACC) using market value weights.
(CA PCE 2009, B. Com. Honors, Delhi University, 2012)

Solution
(i) Cost of debt
𝐼(1 − 𝑡𝑎𝑥) + (𝑅𝑉 − 𝑆𝑉)/𝑁𝑚 9(1 − 0.35) + (100 − 102.90)/10
𝑘𝑑 = ⇒ 𝑘𝑑 = = 0.0548
(𝑅𝑉 + 𝑆𝑉)/2 (100 + 102.90)/2
𝑆𝑎𝑙𝑒𝑠 𝑣𝑎𝑙𝑢𝑒 = 105 − 2.1 (𝐹𝑙𝑜𝑎𝑡𝑎𝑡𝑖𝑜𝑛 𝑐𝑜𝑠𝑡 𝑖𝑠 2%) = 102.9

(ii) Cost of preference share capital


𝐷𝑝 (1 + 𝐷𝑡 ) + (𝑅𝑉 − 𝑆𝑉)/𝑁𝑚 11(1 + 0) + (100 − 102.82)/10
𝑘𝑝 = ⇒ 𝑘𝑝 = = 0.1057
(𝑅𝑉 + 𝑆𝑉)/2 (100 + 102.82)/2
𝑆𝑎𝑙𝑒𝑠 𝑣𝑎𝑙𝑢𝑒 = 106 − 3.18 (𝐹𝑙𝑜𝑎𝑡𝑎𝑡𝑖𝑜𝑛 𝑐𝑜𝑠𝑡 𝑖𝑠 3%) = 102.82

(iii) Cost of equity share capital


𝐷1 2
𝑘𝑒 = + 𝑔 ⇒ 𝑘𝑒 = + 0.05 ⇒ 𝑘𝑒 = 0.15
𝑃0 (24 − 4 (𝐹𝑙𝑜𝑎𝑡𝑎𝑡𝑖𝑜𝑛 𝑐𝑜𝑠𝑡))

(iv) Weighted average cost of capital using market value weights


Specific cost of Composite
Market value Weights capital cost
Funds (𝑀𝑉) (𝑤) (𝑘) (𝑤 × 𝑘)
9% debentures 2,88,750 0.167 0.0548 0.0091516

Chapter 7, Cost of Capital: 36


11% preference share capital 2,38,500 0.138 0.1057 0.0145866
Equity share capital 12,00,000 0.695 0.1500 0.1042500
17,27,250 1.0000 0.1279882
So weighted average cost of capital (WACC) is 0.1280
Notes:
1. Market value of various funds
∎𝑀𝑎𝑟𝑘𝑒𝑡 𝑣𝑎𝑙𝑢𝑒 𝑜𝑓 𝑒𝑞𝑢𝑖𝑡𝑦 𝑠ℎ𝑎𝑟𝑒 𝑐𝑎𝑝𝑖𝑡𝑎𝑙 = 50,000 𝑠ℎ𝑎𝑟𝑒𝑠 × ₹24 = ₹12,00,000
∎𝑀𝑎𝑟𝑘𝑒𝑡 𝑣𝑎𝑙𝑢𝑒 𝑜𝑓 𝑝𝑟𝑒𝑓𝑒𝑟𝑒𝑛𝑐𝑒 𝑠ℎ𝑎𝑟𝑒 𝑐𝑎𝑝𝑖𝑡𝑎𝑙 𝑖𝑠 = 2,250 𝑠ℎ𝑎𝑟𝑒𝑠 × ₹106 = ₹2,38,500
∎𝑀𝑎𝑟𝑘𝑒𝑡 𝑣𝑎𝑙𝑢𝑒 𝑜𝑓 𝑑𝑒𝑏𝑒𝑛𝑡𝑢𝑟𝑒𝑠 𝑖𝑠 = 2,750 𝑑𝑒𝑏𝑒𝑛𝑡𝑢𝑟𝑒𝑠 × ₹105 = ₹2,88,500
2. For computation of the specific cost of capital, always use issue price instead of market
price. In the question we have used the issue price to calculate the specific cost of capital
and ignored the market prices.

Example 37 (Illustration 26)


A manufacturing company has supplied you the following information—
RETAINED Balance Sheet
EARNING IS
DOUBTFUL
Liabilities Amount (₹) Assets Amount (₹)
Equity share capital 12,00,000 Fixed assets 25,00,000
Preference share capital 4,50,000 Current assets 15,00,000
Retained earnings 4,50,000
Debentures 9,00,000
Current liabilities 10,00,000
Total 40,00,000 Total 40,00,000
Additional information—
(i) 20 years, 14% debentures of ₹2,500 face value: redeemable at 5% premium, can be sold
at par, 2% floatation cost.
(ii) 15% preference share: sale price ₹100 per share, 2% floatation cost.
(iii) Equity shares: sale price ₹115 per share, floatation cost ₹5 per share.
The corporate tax rate is 35% and the expected growth in equity dividend is 8% per year. The
expected dividend at the end of the current financial year is ₹11 per share. Assume that the
company is satisfied with its present capital structure and intends to maintain it. Determine
weighed average cost of capital by using book value weights.
(B. Com. Honors, Delhi University, 2017)

Solution
(i) Cost of equity share capital
𝐷1 11
𝑘𝑒 = + 𝑔 ⇒ 𝑘𝑒 = + 0.08 ⇒ 𝑘𝑒 = 0.18
𝑃0 (115 − 5 (𝐹𝑙𝑜𝑎𝑡𝑎𝑡𝑖𝑜𝑛 𝑐𝑜𝑠𝑡))

(ii) Cost of preference share capital


𝐷𝑝 (1 + 𝐷𝑡 ) 15(1 + 0)
𝑘𝑝 = ⇒ 𝑘𝑝 = = 0.1531
𝑆𝑉 98
𝑆𝑎𝑙𝑒𝑠 𝑣𝑎𝑙𝑢𝑒 = 100 − 2 (𝐹𝑙𝑜𝑎𝑡𝑎𝑡𝑖𝑜𝑛 𝑐𝑜𝑠𝑡 𝑖𝑠 2%) = 98

(iii) Cost of retained earnings


𝐷1 11
𝑘𝑟 = + 𝑔 ⇒ 𝑘𝑟 = + 0.08 ⇒ 𝑘𝑟 = 0.1757 WHAT IF I WANT TO CALCULATE IT THROUGH
𝑃0 115 FORMULA OF kr?
Floatation cost shall be ignored while calculating 𝑘𝑟 . BECAUSE THIS IS NOT FRESH EQUITY ISSUED

(iv) Cost of debt


𝐼(1 − 𝑡𝑎𝑥) + (𝑅𝑉 − 𝑆𝑉)/𝑁𝑚 350(1 − 0.35) + (2,625 − 2,450)/20
𝑘𝑑 = ⇒ 𝑘𝑑 = = 0.0931
(𝑅𝑉 + 𝑆𝑉)/2 (2,625 + 2,450)/2 0
𝑆𝑎𝑙𝑒𝑠 𝑣𝑎𝑙𝑢𝑒 = 2,500 − 50 (𝐹𝑙𝑜𝑎𝑡𝑎𝑡𝑖𝑜𝑛 𝑐𝑜𝑠𝑡 𝑖𝑠 2%) = 2,450
𝑅𝑒𝑑𝑒𝑒𝑚𝑎𝑏𝑙𝑒 𝑣𝑎𝑙𝑢𝑒 = 2,500 − 125 (𝑃𝑟𝑒𝑚𝑖𝑢𝑚 𝑜𝑛 𝑟𝑒𝑑𝑒𝑚𝑝𝑡𝑖𝑜𝑛 𝑖𝑠 5%) = 2,625
Chapter 7, Cost of Capital: 37
(v) Weighted average cost of capital using book value weights
Specific cost of Composite
Book value Weights capital cost
Funds (𝐵𝑉) (𝑤) (𝑘) (𝑤 × 𝑘)
Equity share capital 12,00,000 0.40 0.1800 0.072000
Preference share capital 4,50,000 0.15 0.1531 0.022965
Retained earnings 4,50,000 0.15 0.1757 0.026355
Debentures 9,00,000 0.30 0.0931 0.027930
30,00,000 1.00 0.149250
So weighted average cost of capital (WACC) is 0.1493
MANDATORY
Example 38 (Illustration 27)
X Limited has assets of ₹32,00,000 that have been financed by ₹18,00,000 of equity shares (of
₹100 each), general reserve of ₹3,60,000 and debt of ₹10,40,000. For the year ended on 31-03-2010
the company’s total profits before interest and taxes were ₹6,23,000. X Limited pays 8% interest on
borrowed capital and is in a 40% tax bracket. The market value of equity as on 31-03-2010 was
₹150 per share. What is the weighted average cost of capital? Use market values as weights.
(B. Com. Honors, Delhi University, 2010)

Solution
(i) Cost of equity and cost of retained earnings
Price earning model shall be used to calculate the 𝑘𝑒 as growth rate and dividend per share is not
given. Let us calculate the EPS and then 𝑘𝑒 —
Computation of earnings per share Cost of equity
Profits before interest and tax 6,23,000 𝐸𝑃𝑆 ₹17.9933
Less: Interest (10,40,000 × 8%) -83,200 𝑘𝑒 = ⇒ 𝑘𝑒 =
𝑃0 𝑜𝑟 𝑀𝑃 ₹150
Profits after interest or before taxes 5,39,800
⇒ 𝑘𝑒 = 0.119955
Less: Tax @ 40% -2,15,920
≅ 0.12
Profits after tax 3,23,880
18,000 equity Cost of retained earnings
Number of equity shares
shares (General reserves)
𝑃𝑟𝑜𝑓𝑖𝑡𝑠 𝑎𝑓𝑡𝑒𝑟 𝑡𝑎𝑥
𝐸𝑃𝑆 =
𝑁𝑢𝑚𝑏𝑒𝑟 𝑜𝑓 𝑒𝑞𝑢𝑖𝑡𝑦 𝑠ℎ𝑎𝑟𝑒𝑠 ₹17.9933 per
𝑘𝑟 = 𝑘𝑒 ⇒ 𝑘𝑟 = 0.12
3,23,880 share
⇒ 𝐸𝑃𝑆 =
18,000

(ii) Cost of debt


𝐼(1 − 𝑡) 83,200(1 − 0.40)
𝑘𝑑 = ⇒ 𝑘𝑑 = ⇒ 𝑘𝑑 = 0.048
𝑆𝑉 10,40,000

(iii) Weighted average cost of capital using market value weights


Specific cost of
Composite
Market value Weights capital cost
Funds (𝑀𝑉) (𝑤) (𝑘) (𝑤 × 𝑘)
Equity share capital 22,50,000 0.6016 0.120 0.072192
General reserves 4,50,000 0.1203 0.120 0.014436
Debt 10,40,000 0.2781 0.048 0.013349
30,00,000 1.0000 0.099977
So weighted average cost of capital (WACC) is 0.09997 or 0.10
Notes:
∎𝐵𝑜𝑜𝑘 𝑣𝑎𝑙𝑢𝑒 𝑜𝑓 𝑒𝑞𝑢𝑖𝑡𝑦 𝑠ℎ𝑎𝑟𝑒 𝑐𝑎𝑝𝑖𝑡𝑎𝑙 𝑎𝑛𝑑 𝑟𝑒𝑡𝑎𝑖𝑛𝑒𝑑 𝑒𝑎𝑟𝑛𝑖𝑛𝑔𝑠 𝑖𝑠 = ₹18,00,000 + ₹3,60,000
⇒ ₹21,60,000
∎𝑀𝑎𝑟𝑘𝑒𝑡 𝑣𝑎𝑙𝑢𝑒 𝑜𝑓 𝑒𝑞𝑢𝑖𝑡𝑦 𝑠ℎ𝑎𝑟𝑒 𝑐𝑎𝑝𝑖𝑡𝑎𝑙 𝑖𝑠 = 18,000 𝑠ℎ𝑎𝑟𝑒𝑠 × ₹150 = ₹27,00,000

Chapter 7, Cost of Capital: 38


The market value of equity share capital shall be divided in the ratio of book value of equity capital
and retained earnings.
27,00,000
∎𝐸𝑞𝑢𝑖𝑡𝑦 𝑐𝑎𝑝𝑖𝑡𝑎𝑙 = × 18,00,000 ⇒ ₹22,50,000
21,60,000
27,00,000
∎𝐺𝑒𝑛𝑒𝑟𝑎𝑙 𝑟𝑒𝑠𝑒𝑟𝑣𝑒𝑠 = × 3,60,000 ⇒ ₹4,50,4000
21,60,000

Example 39 (Illustration 28)


XYZ Limited has the following book value capital structure—
₹(Crores)
Equity share capital (₹10 each) 15
12% preference share capital (₹100 each) 1
Retained earnings 20
11.5% debentures (₹100 each) 10
11% term loan 12.5
The next expected dividend on equity shares per share is ₹3.60, the dividend per share is expected
to grow at the rate of 7% percent. The market price per share is ₹40.
Preference stock, redeemable after 10 years, is currently selling at ₹75 per share.
Debentures redeemable after 6 years, are selling at ₹80 per debenture.
The income tax rate for the company is 40 percent.
Calculate the weighted average cost of capital by using the (i) book value weights; and (ii) market
value weights.
(B. Com. Honors, Delhi University, 2012 and 2016)

Solution
(i) Cost of equity
𝐷1 3.6
𝑘𝑒 = + 𝑔 ⇒ 𝑘𝑒 = + 0.07 ⇒ 𝑘𝑒 = 0.16
𝑃0 40

(ii) Cost of preference share capital


𝐷𝑝 (1 + 𝐷𝑡 ) + (𝑅𝑉 − 𝑆𝑉)/𝑁𝑚 12(1 + 0) + (100 − 75)/10
𝑘𝑝 = ⇒ 𝑘𝑝 = = 0.1657
(𝑅𝑉 + 𝑆𝑉)/2 (100 + 75)/2

(iii) Cost of retained earnings


𝑘𝑟 = 𝑘𝑒 ⇒ 𝑘𝑟 = 0.16

(iv) Cost of debt


𝐼(1 − 𝑡𝑎𝑥) + (𝑅𝑉 − 𝑆𝑉)/𝑁𝑚 11.5(1 − 0.40) + (100 − 80)/6
𝑘𝑑 = ⇒ 𝑘𝑑 = = 0.1137
(𝑅𝑉 + 𝑆𝑉)/2 (100 + 80)/2

(v) Cost of term loan


𝐼(1 − 𝑡) 1,37,50,000(1 − 0.40)
𝑘𝑑 = ⇒ 𝑘𝑑 = ⇒ 𝑘𝑑 = 0.066
𝑆𝑉 12,50,00,000

(vi) Weighted average cost of capital (WACC) using book value weights
Specific cost of Composite
Book value Weights capital cost
Funds (𝐵𝑉) (𝑤) (𝑘) (𝑤 × 𝑘)
Equity share capital 15,00,00,000 0.256 0.1600 0.0409600
12% preference share 1,00,00,000 0.017 0.1657 0.0028169
capital 20,00,00,000 0.342 0.1600 0.0547200
Retained earnings 10,00,00,000 0.171 0.1137 0.0194427
11.5% debentures 12,50,00,000 0.214 0.0660 0.0141240
Chapter 7, Cost of Capital: 39
11% term loan
58,50,00,000 1.00 0.1320636
So weighted average cost of capital (WACC) is 0.13206 or 0.1321

(vii) Weighted average cost of capital (WACC) using market value weights
Market Specific cost of Composite
value Weights capital cost
Funds (𝑀𝑉) (𝑤) (𝑘) (𝑤 × 𝑘)
Equity share capital 25,71,42,857 0.317 0.1807 0.0507200
12% preference share capital 75,00,000 0.009 0.2000 0.0014913
Retained earnings 34,28,57,143 0.422 0.1807 0.0675200
11.5% debentures 8,00,00,000 0.098 0.0921 0.0111426
11% term loan 12,50,00,000 0.154 0.0840 0.0101640
81,25,00,000 1.0000 0.1410379
So weighted average cost of capital (WACC) is 0.14103 or 0.1410
Notes:
∎𝐵𝑜𝑜𝑘 𝑣𝑎𝑙𝑢𝑒 𝑜𝑓 𝑒𝑞𝑢𝑖𝑡𝑦 𝑠ℎ𝑎𝑟𝑒 𝑐𝑎𝑝𝑖𝑡𝑎𝑙 𝑎𝑛𝑑 𝑟𝑒𝑡𝑎𝑖𝑛𝑒𝑑 𝑒𝑎𝑟𝑛𝑖𝑛𝑔𝑠 𝑖𝑠 = ₹15,00,00,000 + ₹20,00,00,000
⇒ ₹35,00,00,000
∎𝑀𝑎𝑟𝑘𝑒𝑡 𝑣𝑎𝑙𝑢𝑒 𝑜𝑓 𝑒𝑞𝑢𝑖𝑡𝑦 𝑠ℎ𝑎𝑟𝑒 𝑐𝑎𝑝𝑖𝑡𝑎𝑙 𝑖𝑠 = 1,50,00,000 𝑠ℎ𝑎𝑟𝑒𝑠 × ₹40 = ₹60,00,00,000
The market value of equity share capital shall be divided in the ratio of book value of equity capital
and retained earnings.
60,00,00,000
∎𝐸𝑞𝑢𝑖𝑡𝑦 𝑐𝑎𝑝𝑖𝑡𝑎𝑙 = × 15,00,00,000 ⇒ ₹25,71,42,857
35,00,00,000
60,00,00,000
∎𝑅𝑒𝑡𝑎𝑖𝑛𝑒𝑑 𝑒𝑎𝑟𝑛𝑖𝑛𝑔𝑠 = × 20,00,00,000 ⇒ ₹34,28,57,143
35,00,00,000

Example 40 (Illustration 29)


Determine cost of capital using market value weights and using book value weights based on the
following data—
Book values of capital structure:
Debentures (₹1,000 per debentures) ₹16,00,000
Preference shares (₹10 per share) ₹4,00,000
Equity shares (₹100 per share) ₹20,00,000

All securities are traded in the capital markets and recent market prices are:
Debentures ₹1,100 per debenture
Preference shares ₹12 per share
Equity shares ₹200 per share

Anticipated financing opportunities are—


(i) Debentures: ₹1,000 per debenture redeemable at par carrying 8% interest rate. Maturity
period is 20 years and floatation cost is 4%, sale price ₹1,000.
(ii) Preference Shares: ₹10 preference share redeemable at par carrying 10% dividend rate.
Maturity period is 15 years and floatation cost is 5%, sale price ₹10.
(iii) Equity Shares: equity dividend at the end of the year is ₹20 per share, sale price ₹200.
Anticipated growth rate in dividend is 5%.
Corporate tax rate is 35%.
(BBS Honors, Delhi University, 2013)

Solution
(i) Cost of debt
𝐼(1 − 𝑡𝑎𝑥) + (𝑅𝑉 − 𝑆𝑉)/𝑁𝑚 80(1 − 0.35) + (1,000 − 960)/20
𝑘𝑑 = ⇒ 𝑘𝑑 = = 0.0551
(𝑅𝑉 + 𝑆𝑉)/2 (1,000 + 960)/2
𝑆𝑎𝑙𝑒𝑠 𝑣𝑎𝑙𝑢𝑒 = 1,000 − 40 (𝐹𝑙𝑜𝑎𝑡𝑎𝑡𝑖𝑜𝑛 𝑐𝑜𝑠𝑡 𝑖𝑠 4%) = 960

Chapter 7, Cost of Capital: 40


When SV and MV both are given, for the computation of
(RV-SV) and (RV+SV) take SV and subtract flotation cost from
(ii) Cost of preference share capital SV only, ignore MV IN THAT CASE.

𝐷𝑝 (1 + 𝐷𝑡 ) + (𝑅𝑉 − 𝑆𝑉)/𝑁𝑚 1(1 + 0) + (10 − 9.50)/15


𝑘𝑝 = ⇒ 𝑘𝑝 = = 0.1060
(𝑅𝑉 + 𝑆𝑉)/2 (10 + 9.50)/2
𝑆𝑎𝑙𝑒𝑠 𝑣𝑎𝑙𝑢𝑒 = 10 − 0.50 (𝐹𝑙𝑜𝑎𝑡𝑎𝑡𝑖𝑜𝑛 𝑐𝑜𝑠𝑡 𝑖𝑠 5%) = 9.50

(iii) Cost of equity


𝐷1 20
𝑘𝑒 = + 𝑔 ⇒ 𝑘𝑒 = + 0.05 ⇒ 𝑘𝑒 = 0.15
𝑃0 200

(iv) Weighted average cost of capital using market value weights


Specific cost of Composite
Market value Weights capital cost
Funds (𝑀𝑉) (𝑤) (𝑘) (𝑤 × 𝑘)
Debentures 17,60,000 0.2821 0.0551 0.015544
Preference shares 4,80,000 0.0769 0.1060 0.008151
Equity shares 40,00,000 0.6410 0.1500 0.096150
62,40,000 1.00 0.119845
So weighted average cost of capital (WACC) is 0.119845
Notes:
1. For computation of the specific cost of capital, always use issue price instead of market
price. In the question we have used the issue price to calculate the specific cost of capital
and ignored the market prices.
2. For the purpose of calculation of WACC using market value weights, take market value only
and not the issue price. Market value of various funds is as follows—
∎𝑀𝑎𝑟𝑘𝑒𝑡 𝑣𝑎𝑙𝑢𝑒 𝑜𝑓 𝑑𝑒𝑏𝑒𝑛𝑡𝑢𝑟𝑒𝑠 𝑖𝑠 = 1,600 𝑑𝑒𝑏𝑒𝑛𝑡𝑢𝑟𝑒𝑠 × ₹1,100 = ₹17,60,000
∎𝑀𝑎𝑟𝑘𝑒𝑡 𝑣𝑎𝑙𝑢𝑒 𝑜𝑓 𝑝𝑟𝑒𝑓𝑒𝑟𝑒𝑛𝑐𝑒 𝑠ℎ𝑎𝑟𝑒 𝑐𝑎𝑝𝑖𝑡𝑎𝑙 𝑖𝑠 = 40,000 𝑠ℎ𝑎𝑟𝑒𝑠 × ₹12 = ₹4,80,000
∎𝑀𝑎𝑟𝑘𝑒𝑡 𝑣𝑎𝑙𝑢𝑒 𝑜𝑓 𝑒𝑞𝑢𝑖𝑡𝑦 𝑠ℎ𝑎𝑟𝑒 𝑐𝑎𝑝𝑖𝑡𝑎𝑙 = 20,000 𝑠ℎ𝑎𝑟𝑒𝑠 × ₹200 = ₹40,00,000

(v) Weighted average cost of capital using book value weights


Specific cost of Composite
Book value Weights capital cost
Funds (𝐵𝑉) (𝑤) (𝑘) (𝑤 × 𝑘)
Debentures 16,00,000 0.40 0.0551 0.02204
Preference shares 4,00,000 0.10 0.1060 0.01060
Equity shares 20,00,000 0.50 0.1500 0.07500
40,00,000 1.00 0.10764
So weighted average cost of capital (WACC) is 0.10764

Example 41 (Illustration 31)


ABC Limited wishes to raise additional finance of ₹20,00,000 for meeting its investment plans.
VERY GOOD
The company has ₹4,00,000 in the form of retained earnings available for investment purposes. The
following are the further details— Retained earnings are available as EQUITY only
Debt equity ratio is 25:75. Cost of debt at the rate of 10% (before tax) up to ₹2,00,000 and 13%
(before tax) beyond that. Expected EPS (Earnings per share) is ₹12 and dividend payout ratio is
50% of earnings. Expected growth rate in dividend is 10%. Current market price per share is ₹60.
Company’s tax rate 30% and shareholders’ personal tax rate is 20%.
Required: Divident payout ratio is basically expected
divident i.e 50% of EPS in this question 50% of
(i) Calculate the post-tax average cost of additional debt. 12 is 6=D1
(ii) Calculate the cost of retained earnings and cost of equity.
(iii) Calculate the overall weighted average (after tax) cost of additional finance.
(CA PCC, 2008)

Solution
Pattern of additional financing will be as follows—

Chapter 7, Cost of Capital: 41


Debt Equity
25% 75%
₹20,00,000 × 25% = ₹5,00,000 ₹20,00,000 × 75% = ₹15,00,000
10% debt ₹2,00,000 Retained earnings (given) ₹4,00,000
13% debt ₹3,00,000 Equity (to be raised) (Balancing figure) ₹11,00,000
Total ₹5,00,000 Total ₹15,00,000

(i) Cost of debt


𝐼(1 − 𝑡) (20,000 + 39,000)(1 − 0.30)
𝑘𝑑 = ⇒ 𝑘𝑑 = ⇒ 𝑘𝑑 = 0.0826
𝑆𝑉 5,00,000
𝐴𝑚𝑜𝑢𝑛𝑡 𝑜𝑓 𝑖𝑛𝑡𝑒𝑟𝑒𝑠𝑡 = ₹2,00,000 × 10% + ₹3,00,000 × 13% ⇒ 20,000 + 39,000

(ii) Cost of equity share capital


𝐷1 6
𝑘𝑒 = + 𝑔 ⇒ 𝑘𝑒 = + 0.10 ⇒ 𝑘𝑒 = 0.20
𝑃0 60
𝐷1 = 𝐸𝑥𝑝𝑒𝑐𝑡𝑒𝑑 𝐸𝑃𝑆 × 𝐷𝑖𝑣𝑖𝑑𝑒𝑛𝑑 𝑝𝑎𝑦𝑜𝑢𝑡 𝑟𝑎𝑡𝑖𝑜 ⇒ 𝐷1 = ₹12 × 50% ⇒ 𝐷1 = ₹6

(iii) Cost retained earnings


𝑘𝑟 = 𝑘𝑒 (1 − 𝑡)(1 − 𝑏)(1 − 𝑓)
Where,
𝑡 = Tax rate on shareholders
𝑏 = Brokerage rate to be paid by the shareholder for making new investments
𝑓 = Floatation cost on issue of new share capital

So, 𝑘𝑟 = 𝑘𝑒 (1 − 𝑡)(1 − 𝑏)(1 − 𝑓) ⇒ 𝑘𝑟 = 0.20(1 − 0.20)(1 − 0)(1 − 0) ⇒ 𝑘𝑟 = 0.16

(iv) Weighted average cost of additional finance


Specific cost of Composite
Amount Weights capital cost
Funds (₹) (𝑤) (𝑘) (𝑤 × 𝑘)
Equity capital 11,00,000 0.55 0.2000 0.11000
Retained earnings 4,00,000 0.20 0.1600 0.03200
Debt 5,00,000 0.25 0.0826 0.02065
20,00,000 1.00 0.16265
So weighted average cost of capital (WACC) is 0.16265

Example 42
From the following information provided by MNO Limited, you are required to calculate the
weighted average cost of capital (𝑘𝑂 ) using market value weights. The present book value capital
structure of MNO Limited is—

Debentures (₹100 per debenture) 10,00,000
Preference shares (₹100 per share) 5,00,000
Equity shares (₹10 per share) 20,00,000
Retained earnings 5,00,000
Total 40,00,000
All these securities are traded in the capital markets. Recent prices are—debentures @ ₹110,
preference shares @ ₹120 and equity shares @ ₹22. Anticipated external financing opportunities
are—
(i) ₹100 per debenture redeemable at par, 20 years maturity, 8% coupon rate, 4% floating
costs and sale price ₹100.
(ii) ₹100 per preference share redeemable at par, 15 years maturity, 10% dividend rate, 5%
floating costs and sale price ₹100.
(iii) Equity shares: ₹2 per share floating costs and sales price ₹22.

Chapter 7, Cost of Capital: 42


In addition, the dividend expected on the equity shares at the end of the year is ₹2 per share; the
anticipated growth rate in dividends is 5% and the company has the practice of paying all its
earnings in the form of dividends. The corporate income tax rate is 30%.
Required: Calculate weighted average cost of capital (WACC) using market value weights.
(B. Com. Honors, Delhi University, 2018)

Solution
(i) Cost of debt
𝐼(1 − 𝑡𝑎𝑥) + (𝑅𝑉 − 𝑆𝑉)/𝑁𝑚 8(1 − 0.30) + (100 − 96)/20
𝑘𝑑 = ⇒ 𝑘𝑑 = = 0.0592
(𝑅𝑉 + 𝑆𝑉)/2 (100 + 96)/2
𝑆𝑎𝑙𝑒𝑠 𝑣𝑎𝑙𝑢𝑒 = 100 − 4 (𝐹𝑙𝑜𝑎𝑡𝑎𝑡𝑖𝑜𝑛 𝑐𝑜𝑠𝑡 𝑖𝑠 4%) = 96

(ii) Cost of preference share capital


𝐷𝑝 (1 + 𝐷𝑡 ) + (𝑅𝑉 − 𝑆𝑉)/𝑁𝑚 10(1 + 0) + (100 − 95)/15
𝑘𝑝 = ⇒ 𝑘𝑝 = = 0.1059
(𝑅𝑉 + 𝑆𝑉)/2 (100 + 95)/2
𝑆𝑎𝑙𝑒𝑠 𝑣𝑎𝑙𝑢𝑒 = 100 − 5 (𝐹𝑙𝑜𝑎𝑡𝑎𝑡𝑖𝑜𝑛 𝑐𝑜𝑠𝑡 𝑖𝑠 5%) = 95

(iii) Cost of equity


𝐷1 2
𝑘𝑒 = + 𝑔 ⇒ 𝑘𝑒 = + 0.05 ⇒ 𝑘𝑒 = 0.1409
𝑃0 22
might be because it is an existing equity

(iv) Cost of retained earnings


𝑘𝑟 = 𝑘𝑒 ⇒ 𝑘𝑟 = 0.1409

(v) Weighted average cost of capital using market value weights


Specific cost of Composite
Market value Weights capital cost
Funds (𝑀𝑉) (𝑤) (𝑘) (𝑤 × 𝑘)
Debentures 11,00,000 0.1803 0.0592 0.0106738
Preference shares 6,00,000 0.0984 0.1059 0.0104206
Equity shares 35,20,000 0.5770 0.1409 0.0812993
Retained earnings 8,80,000 0.1443 0.1409 0.0203319
61,00,000 1.0000 0.1227255
So weighted average cost of capital (WACC) is 0.1227
Notes:
1. For computation of the specific cost of capital, always use issue price instead of market
price. In the question we have used the issue price to calculate the specific cost of capital
and ignored the market prices.
2. For the purpose of calculation of WACC using market value weights, take market value only
and not the issue price. Market value of various funds is as follows—
∎𝑀𝑎𝑟𝑘𝑒𝑡 𝑣𝑎𝑙𝑢𝑒 𝑜𝑓 𝑑𝑒𝑏𝑒𝑛𝑡𝑢𝑟𝑒𝑠 𝑖𝑠 = 10,000 𝑑𝑒𝑏𝑒𝑛𝑡𝑢𝑟𝑒𝑠 × ₹110 = ₹11,00,000
∎𝑀𝑎𝑟𝑘𝑒𝑡 𝑣𝑎𝑙𝑢𝑒 𝑜𝑓 𝑝𝑟𝑒𝑓𝑒𝑟𝑒𝑛𝑐𝑒 𝑠ℎ𝑎𝑟𝑒 𝑐𝑎𝑝𝑖𝑡𝑎𝑙 𝑖𝑠 = 5,000 𝑠ℎ𝑎𝑟𝑒𝑠 × ₹120 = ₹6,00,000
∎𝑀𝑎𝑟𝑘𝑒𝑡 𝑣𝑎𝑙𝑢𝑒 𝑜𝑓 𝑒𝑞𝑢𝑖𝑡𝑦 𝑠ℎ𝑎𝑟𝑒 𝑐𝑎𝑝𝑖𝑡𝑎𝑙 = 2,00,000 𝑠ℎ𝑎𝑟𝑒𝑠 × ₹22 = ₹44,00,000
The market value of equity share capital shall be divided in the ratio of book value of equity
capital and retained earnings.
44,00,000
∎𝐸𝑞𝑢𝑖𝑡𝑦 𝑐𝑎𝑝𝑖𝑡𝑎𝑙 = × 20,00,000 ⇒ ₹35,20,000
25,00,000
44,00,000
∎𝑅𝑒𝑡𝑎𝑖𝑛𝑒𝑑 𝑒𝑎𝑟𝑛𝑖𝑛𝑔𝑠 = × 20,00,000 ⇒ ₹8,80,000
25,00,000

Example 43
Calculate the overall cost of capital of Sushant Limited from the information given below using book
value weights and market value weights—
Chapter 7, Cost of Capital: 43
Book value Market value
₹ ₹
Equity shares of ₹10 each 4,00,000 6,40,000
12% preference shares of ₹100 each 1,00,000 75,000
(Redeemable at par after 10 years)
11% debentures of ₹100 each 2,50,000 2,25,000
(Redeemable at par after 5 years)
Reserves 1,00,000 —
Last year the company paid a dividend of ₹2.20 per share. It is expected to declare a dividend of
₹2.42 per share. The company has been maintaining the growth rate in dividends over the years and
is expected to do, so, in future. Tax rate is 40%.
(B. Com. Honors, Delhi University, 2019)

Solution
(i) Cost of equity
𝐷1 2.42
𝑘𝑒 = + 𝑔 ⇒ 𝑘𝑒 = + 0.10 ⇒ 𝑘𝑒 = 0.25125
𝑃0 16

𝐷1 = 𝐷0 (1 + 𝑔)1 ⇒ 2.42 = 2.20(1 + 𝑔)1 ⇒ 1.1 − 1 = 𝑔 ⇒ 𝑔 = 0.10 𝑜𝑟 10%


𝑀𝑎𝑟𝑘𝑒𝑡 𝑣𝑎𝑙𝑢𝑒 6,40,000
𝑀𝑎𝑟𝑘𝑒𝑡 𝑝𝑟𝑖𝑐𝑒 𝑜𝑓 𝑒𝑞𝑢𝑖𝑡𝑦 𝑠ℎ𝑎𝑟𝑒 = ⇒ ⇒ ₹16
𝑁𝑢𝑚𝑏𝑒𝑟 𝑜𝑓 𝑒𝑞𝑢𝑖𝑡𝑦 𝑠ℎ𝑎𝑟𝑒𝑠 40,000

(ii) Cost of preference share capital


𝐷𝑝 (1 + 𝐷𝑡 ) + (𝑅𝑉 − 𝑆𝑉)/𝑁𝑚 12(1 + 0) + (100 − 75)/10
𝑘𝑝 = ⇒ 𝑘𝑝 = = 0.165714
(𝑅𝑉 + 𝑆𝑉)/2 (100 + 75)/2

𝑀𝑎𝑟𝑘𝑒𝑡 𝑣𝑎𝑙𝑢𝑒 75,000


𝑆𝑎𝑙𝑒𝑠 𝑣𝑎𝑙𝑢𝑒 = ⇒ ⇒ ₹75
𝑁𝑢𝑚𝑏𝑒𝑟 𝑜𝑓 𝑝𝑟𝑒𝑓𝑒𝑟𝑒𝑛𝑐𝑒 𝑠ℎ𝑎𝑟𝑒𝑠 1,000

(iii) Cost of debt


𝐼(1 − 𝑡𝑎𝑥) + (𝑅𝑉 − 𝑆𝑉)/𝑁𝑚 11(1 − 0.40) + (100 − 90)/5
𝑘𝑑 = ⇒ 𝑘𝑑 = = 0.09053
(𝑅𝑉 + 𝑆𝑉)/2 (100 + 90)/2

𝑀𝑎𝑟𝑘𝑒𝑡 𝑣𝑎𝑙𝑢𝑒 2,25,000


𝑆𝑎𝑙𝑒𝑠 𝑣𝑎𝑙𝑢𝑒 = ⇒ ⇒ ₹90
𝑁𝑢𝑚𝑏𝑒𝑟 𝑜𝑓 𝑑𝑒𝑏𝑒𝑛𝑡𝑢𝑟𝑒𝑠 2,500

(iv) Cost of retained earnings


𝑘𝑟 = 𝑘𝑒 ⇒ 𝑘𝑟 = 0.25125

(v) Weighted average cost of capital using book value weights


Specific cost of Composite
Book value Weights capital cost
Funds (𝐵𝑉) (𝑤) (𝑘) (𝑤 × 𝑘)
Equity shares 4,00,000 0.47059 0.251250 0.118235294
Preference shares 1,00,000 0.11765 0.165714 0.019495765
Debentures 2,50,000 0.29412 0.090526 0.026625294
Reserves 1,00,000 0.11765 0.251250 0.029558824
8,50,000 1.00000 0.193915176
So weighted average cost of capital (WACC) is 0.1939

(vi) Weighted average cost of capital using market value weights


Market value Weights Specific cost of Composite
Funds (𝑀𝑉) (𝑤) capital cost
Chapter 7, Cost of Capital: 44
(𝑘) (𝑤 × 𝑘)
Equity shares 5,12,000 0.54468 0.251250 0.136851064
Preference shares 75,000 0.07979 0.165714 0.013221862
Debentures 2,25,000 0.23936 0.090526 0.021668457
Reserves 1,28,000 0.13617 0.251250 0.034212766
9,40,000 1.00000 0.205954149
So weighted average cost of capital (WACC) is 0.205944
Notes:
1. In this question market price can be used to calculate the specific cost of capital because
issue prices are not given.
2. For the purpose of calculation of WACC using market value weights, take market value only
and not the issue price. Market value of various funds is as follows—
∎𝑀𝑎𝑟𝑘𝑒𝑡 𝑣𝑎𝑙𝑢𝑒 𝑜𝑓 𝑒𝑞𝑢𝑖𝑡𝑦 𝑠ℎ𝑎𝑟𝑒 𝑐𝑎𝑝𝑖𝑡𝑎𝑙 = ₹6,40,000
The market value of equity share capital shall be divided in the ratio of book value of equity
capital and retained earnings.
6,40,000
∎𝐸𝑞𝑢𝑖𝑡𝑦 𝑐𝑎𝑝𝑖𝑡𝑎𝑙 = × 4,00,000 ⇒ ₹5,12,000
5,00,000
6,40,000
∎𝑅𝑒𝑡𝑎𝑖𝑛𝑒𝑑 𝑒𝑎𝑟𝑛𝑖𝑛𝑔𝑠 = × 1,00,000 ⇒ ₹1,28,000
5,00,000
∎𝑀𝑎𝑟𝑘𝑒𝑡 𝑣𝑎𝑙𝑢𝑒 𝑜𝑓 𝑝𝑟𝑒𝑓𝑒𝑟𝑒𝑛𝑐𝑒 𝑠ℎ𝑎𝑟𝑒 𝑐𝑎𝑝𝑖𝑡𝑎𝑙 𝑖𝑠 = ₹75,000
∎𝑀𝑎𝑟𝑘𝑒𝑡 𝑣𝑎𝑙𝑢𝑒 𝑜𝑓 𝑑𝑒𝑏𝑒𝑛𝑡𝑢𝑟𝑒𝑠 𝑖𝑠 = ₹2,25,000

Example 44 INTEREST
X Limited has operating profits of ₹8,60,000 and a fixed finance burden of ₹60,000. The company
is subject to income tax payment of ₹2,00,000. The company has 3,00,000 equity shares of
₹30,00,000 and 18% debentures of ₹3,12,500. The market price of equity share is ₹12. Find—
(i) EPS
(ii) Cost of equity
(iii) Cost of debt
(B. Com. Honors, Delhi University, 2007)

Solution

(i) Earning per share


𝐸𝐵𝐼𝑇 − 𝐼 − 𝑡 8,60,000 − 60,000 − 2,00,000
𝐸𝑃𝑆 = ⇒ ⇒ ₹2
𝑁𝑢𝑚𝑏𝑒𝑟 𝑜𝑓 𝑒𝑞𝑢𝑖𝑡𝑦 𝑠ℎ𝑎𝑟𝑒𝑠 3,00,000

(ii) Cost of equity


𝐸𝑃𝑆 ₹2 1 1
𝑘𝑒 = ⇒ ⇒ 0.1667 𝑜𝑟 𝑘𝑒 = ⇒ ⇒ 0.1667
𝑃0 𝑜𝑟 𝑀𝑃 ₹12 𝑃/𝐸 𝑅𝑎𝑡𝑖𝑜 6

𝑀𝑎𝑟𝑘𝑒𝑡 𝑝𝑟𝑖𝑐𝑒 ₹12


𝑃/𝐸 𝑟𝑎𝑡𝑖𝑜 = ⇒ ⇒ 6 𝑡𝑖𝑚𝑒𝑠
𝐸𝑃𝑆 ₹2

(iii) Cost of debt


𝐼(1 − 𝑡) 56,250(1 − 0.25)
𝑘𝑑 𝑜𝑟 𝑘𝑏 (𝐴𝑓𝑡𝑒𝑟 𝑡𝑎𝑥 𝑐𝑜𝑠𝑡 𝑜𝑓 𝑑𝑒𝑏𝑡) = ⇒ ⇒ 0.1350
𝑆𝑉 3,12,500

𝐼 (𝐼𝑛𝑡𝑒𝑟𝑒𝑠𝑡) = ₹3,12,500 × 0.18 ⇒ ₹56,250


𝑇𝑎𝑥 𝑎𝑚𝑜𝑢𝑛𝑡 ₹2,00,000
𝑇𝑎𝑥 𝑟𝑎𝑡𝑒 = ⇒ ⇒ 0.25
𝑃𝑟𝑜𝑓𝑖𝑡𝑠 𝑏𝑒𝑓𝑜𝑟𝑒 𝑡𝑎𝑥 − 𝐼𝑛𝑡𝑒𝑟𝑒𝑠𝑡 ₹8,60,000 − ₹60,000(𝐼𝑛𝑡𝑒𝑟𝑒𝑠𝑡)

Example 45
A limited company has the following capital structure

Chapter 7, Cost of Capital: 45



Equity share capital (4,00,000 shares) 40,00,000
12% preference share capital 4,00,000
10% debentures 6,00,000
The equity shares of the company are quoted at ₹110 and the company is expected to declare a
dividend of ₹15 per share. Rate of growth of dividend is 8%, which is expected to be maintained.
(i) Assuming the tax rate of 40%, calculate WACC.
(ii) The company wants to raise the additional term loan of ₹5,00,000 at 10%. Calculate the
revised WACC assuming the market price of equity share has gone down to ₹105.
(B. Com. Honors, Delhi University, 2013)

Solution

(i) The weighted average cost of capital based on existing capital


structure
(i) Cost of equity
𝐷1 15
𝑘𝑒 = + 𝑔 ⇒ 𝑘𝑒 = + 0.08 ⇒ 𝑘𝑒 = 0.2164
𝑃0 110

(ii) Cost of preference share capital


𝐷𝑝 (1 + 𝐷𝑡 ) 48,000(1 + 0)
𝑘𝑝 = ⇒ 𝑘𝑝 = ⇒ 𝑘𝑝 = 0.12
𝑆𝑉 4,00,000

(iii) Cost of debt


𝐼(1 − 𝑡) 60,000(1 − 0.40)
𝑘𝑑 = ⇒ 𝑘𝑑 = ⇒ 𝑘𝑑 = 0.06
𝑆𝑉 6,00,000

(iv) Weighted average cost of capital (WACC)


Specific cost of Composite
Amount Weights capital cost
Funds (₹) (𝑤) (𝑘) (𝑤 × 𝑘)
Equity capital
12% preference share 40,00,000 𝑊𝑒 = 0.80 𝑘𝑒 = 0.2164 0.17312
capital 4,00,000 𝑊𝑝 = 0.08 𝑘𝑝 = 0.1200 0.00960
10% debentures 6,00,000 𝑊𝑑 = 0.12 𝑘𝑑 = 0.0600 0.00720
50,00,000 1.00 0.18992
So weighted average cost of capital (WACC) is 0.1899 or 0.19
Calculation of weights
40,00,000
∎𝑤𝑒 = = 0.80
50,00,000
4,00,000
∎𝑤𝑝 = = 0.08
50,00,000
6,00,000
∎𝑤𝑑 = = 0.12
50,00,000

(ii) The weighted average cost of capital additional debentures of


₹5,00,000 are issued
(i) New cost of equity
𝐷1 15
𝑘𝑒 = + 𝑔 ⇒ 𝑘𝑒 = + 0.08 ⇒ 𝑘𝑒 = 0.2229
𝑃0 105

(ii) Cost of preference share capital


Chapter 7, Cost of Capital: 46
𝐷𝑝 (1 + 𝐷𝑡 ) 48,000(1 + 0)
𝑘𝑝 = ⇒ 𝑘𝑝 = ⇒ 𝑘𝑝 = 0.12
𝑆𝑉 4,00,000

(iii) Cost of existing debt


𝐼(1 − 𝑡) 600,000(1 − 0.40)
𝑘𝑑 = ⇒ 𝑘𝑑 = ⇒ 𝑘𝑑 = 0.06
𝑆𝑉 6,00,000

(iii) Cost of additional debt i.e., term loan


𝐼(1 − 𝑡) 50,000(1 − 0.40)
𝑘𝑑 = ⇒ 𝑘𝑑 = ⇒ 𝑘𝑑 = 0.06
𝑆𝑉 5,00,000

(iv) Weighted average cost of capital (WACC)


Specific cost of Composite
Amount Weights capital cost
Funds (₹) (𝑤) (𝑘) (𝑤 × 𝑘)
Equity capital
12% preference share 40,00,000 𝑊𝑒 = 0.7273 𝑘𝑒 = 0.2229 0.162115
capital 4,00,000 𝑊𝑝 = 0.0727 𝑘𝑝 = 0.12 0.008724
10% debentures 6,00,000 𝑊𝑑 = 0.1091 𝑘𝑑 = 0.06 0.006546
10% term loan 5,00,000 𝑊𝑑 = 0.0909 𝑘𝑑 = 0.06 0.005454
55,00,000 1.00 0.182839
So weighted average cost of capital (WACC) is 0.1828
Calculation of weights
40,00,000
∎𝑊𝑒 = = 0.7273
55,00,000
4,00,000
∎𝑊𝑝 = = 0.0727
55,00,000
6,00,000
∎𝑊𝑑 = = 0.1091
55,00,000
5,00,000
∎𝑊𝑑 = = 0.0909
55,00,000

Example 46
B Limited has 25,000 equity shares of ₹10 each outstanding. These are currently selling at ₹20 per
share. It also has 1,000 debentures of ₹100 each bearing a coupon rate of 10%. Debentures are
selling at ₹125 per debenture in the market. A dividend of ₹3 per share has just been paid on equity
shares. Tax rate is 35% and growth rate is expected to be 5%. Calculate the weighted average cost
of capital.
(B. Com Honors, Delhi University, 2007)

Solution
(i) Cost of equity share capital
𝐷1 3.15
𝑘𝑒 = + 𝑔 ⇒ 𝑘𝑒 = + 0.05 ⇒ 𝑘𝑒 = 0.2075
𝑃0 20
𝐷1 = 𝐷0 (1 + 𝑔) ⇒ 𝑅𝑠. 3(1 + 0.05) ⇒ ₹3.15

(ii) Cost of debt


𝐼(1 − 𝑡) 10(1 − 0.35)
𝑘𝑑 = ⇒ 𝑘𝑑 = ⇒ 𝑘𝑑 = 0.052
𝑆𝑉 125

(iii) Weighted average cost of capital (WACC)

Chapter 7, Cost of Capital: 47


Specific cost of Composite
Amount Weights capital cost
Funds (₹) (𝑤) (𝑘) (𝑤 × 𝑘)
Equity capital (See Note 1) 5,00,000 𝑤𝑒 = 0.8 𝑘𝑒 = 0.2075 0.1660
10% debentures (See Note 1) 1,25,000 𝑤𝑑 = 0.20 𝑘𝑑 = 0.052 0.0104
6,25,000 1.00 0.1764
So weighted average cost of capital (WACC) is 0.1764 (See Note 2)
Note 1:
𝑀𝑎𝑟𝑘𝑒𝑡 𝑣𝑎𝑙𝑢𝑒 𝑜𝑓 𝑒𝑞𝑢𝑖𝑡𝑦 = 𝑁𝑢𝑚𝑏𝑒𝑟 𝑜𝑓 𝑒𝑞𝑢𝑖𝑡𝑦 𝑠ℎ𝑎𝑟𝑒𝑠 × 𝑀𝑎𝑟𝑘𝑒𝑡 𝑝𝑟𝑖𝑐𝑒 𝑝𝑒𝑟 𝑠ℎ𝑎𝑟𝑒
⇒ 25,000 𝑠ℎ𝑎𝑟𝑒𝑠 × ₹20 𝑝𝑒𝑟 𝑠ℎ𝑎𝑟𝑒 ⇒ ₹5,00,000
𝑀𝑎𝑟𝑘𝑒𝑡 𝑣𝑎𝑙𝑢𝑒 𝑜𝑓 𝑑𝑒𝑏𝑒𝑛𝑡𝑢𝑟𝑒𝑠 = 𝑁𝑢𝑚𝑏𝑒𝑟 𝑜𝑓 𝑑𝑒𝑏𝑒𝑛𝑡𝑢𝑟𝑒𝑠 × 𝑀𝑎𝑟𝑘𝑒𝑡 𝑝𝑟𝑖𝑐𝑒 𝑝𝑒𝑟 𝑑𝑒𝑏𝑒𝑛𝑡𝑢𝑟𝑒
⇒ 1,000 𝑑𝑒𝑏𝑒𝑛𝑡𝑢𝑟𝑒𝑠 × ₹125 𝑝𝑒𝑟 𝑑𝑒𝑏𝑒𝑛𝑡𝑢𝑟𝑒 ⇒ ₹1,25,000
Note 2:
WACC has been calculated using market value weights. This can also be calculated using book value
weights, and then it would be 0.16308 (See Note 3).
Note 3:
Specific cost of Composite
Amount Weights capital cost
Funds (₹) (𝑤) (𝑘) (𝑤 × 𝑘)
Equity capital 2,50,000 𝑊𝑒 = 0.7143 𝑘𝑒 = 0.2075 0.14822
10% debentures 1,00,000 𝑊𝑑 = 0.2857 𝑘𝑑 = 0.052 0.01486
3,50,000 1.0000 0.16308
So weighted average cost of capital (WACC) is 0.16308

Chapter 7, Cost of Capital: 48


Q. 1.: What do you meant by cost of capital? What are its
components? How is the cost of retained earnings estimated?

Answer:

Meaning of cost of capital and its basic concepts


“Cost of capital is the minimum rate of return expected by an investor on his investment.”
OR
“Cost of capital is the expense incurred by a firm to use a particular fund.” E.g., interest paid on the
debt, dividend paid on preference shares, etc.
In general, the concept of cost of capital is applicable to the long-term funds and not short-term
funds.

Components of cost of capital


There are three components of the cost of capital (𝑘𝑂 ) i.e.—
𝑘𝑂 = 𝐼𝑓 + 𝑏 + 𝑓
Where,
𝑘𝑂 = Cost of capital
𝐼𝑓 = Risk-free interest rate
𝑏 = Business risk premium
𝑓 = Financial risk premium
Business risk and financial risk are assumed to be constant. Also, whole calculating cost of capital,
tax adjustments is required especially in case of debt.

How is the cost of retained earnings estimated?


It may appear to carry no cost since they represent funds which have not been raised from outside.
The contention that retained earnings are free of cost, however, is not correct. On the contrary, they
do involve cost like any other source.
The cost of retained earnings is closely related to the equity shares. If the earnings are not retained,
they will be paid out to the equity shareholders as dividends. Retained earnings are often looked as
fully subscribed issue of new equity, since they increase the shareholder’s equity in the same way
that a new issue of equity shares would. The cost of retained earnings must, therefore, be viewed as
the opportunity cost of the foregone dividends to the existing shareholders. Cost of retained
earnings can also be computed with the help of above methods namely—
1. Dividend Model
2. Dividend Growth Model
3. Price Earning Method
4. Capital Asset Pricing Model (CAPM)
Cost of retained earnings can also be computed using the following formula—
𝑘𝑟 = 𝑘𝑒 (1 − 𝑡)(1 − 𝑏)(1 − 𝑓)
Where,
𝑡 = Tax rate on income of shareholders
𝑏 = Brokerage rate to be paid by the shareholder for making new investments
𝑓 = Floatation cost on issue of new share capital
Note: Floatation costs shall not be subtracted from market price or issue price or sales value while
calculating cost of retained earnings.

Chapter 7, Cost of Capital: 49


Q. 2.: Explain Implicit and Explicit cost. Do you consider cost of
retained earnings as implicit cost or explicit cost? Give reason in
support of your answer.
Answer:

Implicit and explicit cost


1. Implicit Cost of Capital: Implicit cost of capital is the opportunity cost of capital i.e., the
benefit that could have been earned if the funds were used elsewhere instead of using in the
business. E.g., a business purchased a plant and machinery of ₹5,00,000 for production.
Company could earn ₹50,000 per annum as interest if these funds were invested in the
bank. But in order to get the benefit from the machinery the company is sacrificing the
benefit of the interest. So, this sacrificed cost is called implicit cost.
2. Explicit Cost of Capital: Explicit cost of capital is that expense which is incurred actually on
using any fund. E.g., interest paid on debt, dividend on preference shares, etc.
Formula for the computation of explicit cost of capital is as follows—
𝑛
𝐶𝑂𝑡 𝐶𝑂𝑃𝑛
𝐶𝐼0 = ∑ 𝑡
+ , 𝑜𝑟
(1 + 𝑘) (1 + 𝑘)𝑛
𝑡=1
𝐶𝑂1 𝐶𝑂2 𝐶𝑂3 𝐶𝑂𝑛 𝐶𝑂𝑃𝑛
𝐶𝐼0 = 1
+ 2
+ 3
+ ⋯⋯⋯+ 𝑛
+
(1 + 𝑘) (1 + 𝑘) (1 + 𝑘) (1 + 𝑘) (1 + 𝑘)𝑛
Where,
𝐶𝐼0 =Cash inflows of the fund issued in zero year
𝐶𝑂𝑡 =Cash outflows in the tth year in the form of interest or dividend etc.
𝐶𝑂𝑃𝑛 = Cash outflows in the nth year
𝑘 =Cost of capital, 𝑡 =time period from 1 to n, 𝑛 =Number of years of maturity

Do you consider cost of retained earnings as implicit cost or explicit


cost? Give reason in support of your answer
We consider the cost of retained earnings as implicit cost.
Reason: It may appear that retained earnings carry no cost since they represent funds which have
not been raised from outside. Further there are no cash outflows associated with retained earnings
in the form of dividend or interest. The contention that retained earnings are free of cost, however,
is not correct. On the contrary, they do involve cost (explicit) like any other source. There is implicit
cost of retained earnings. The cost of retained earnings must, therefore, be viewed as the
opportunity cost of the foregone dividends to the existing shareholders.
The cost of retained earnings is closely related to the equity shares. If the earnings are not retained,
they will be paid out to the equity shareholders as dividends. Retained earnings are often looked as
fully subscribed issue of new equity, since they increase the shareholder’s equity in the same way
that a new issue of equity shares would.

Chapter 7, Cost of Capital: 50


Q. 3.: What is weighted average cost of capital? Why is it used?
Should cost of capital be calculated according to book value weights
or market value weights?
Q. 4.: “Market value weights are superior to book value weights”.
Comment.
Q. 5.: Explain in brief the weights that you would take into
consideration for computing weighted average cost of capital. Why
market value weights are superior to book value weights?

Answer:

Weighted average cost of capital


Weighted average cost of capital is the combined/composite/overall cost of capital. Company uses
different sources of fund viz. equity capital, preference capital, debentures, bonds, etc. Also, the
specific costs of capital of these funds are different. So, in order to calculate the weighted average
cost of capital the specific cost of capitals are multiplied with their respective weights and then
aggregated. Weighted average cost of capital (WACC) is represented by 𝑘𝑂 . Formula to calculate the
𝑘𝑂 is—
𝑘𝑂 = [𝑘𝑑 × 𝑤𝑑 ] + [𝑘𝑝 × 𝑤𝑝 ] + [𝑘𝑒 × 𝑤𝑒 ] + [𝑘𝑟 × 𝑤𝑟 ]
Where,
𝑤𝑑 = Proportion of debt in total funds
𝑤𝑝 = Proportion of preference share capital in total funds
𝑤𝑒 = Proportion of equity share capital in total funds
𝑤𝑟 =Proportion of retained earnings in total funds

Why is it used?
1. Use in capital budgeting process: Cost of capital is quite useful in capital budgeting
process. It works as basis for decisions. E.g., in present value method or discounted cash
flow method the future cash inflows of a project are discounted by this rate.
2. Use in capital structure decisions: Cost of capital is used in capital structure decisions.
When the management of the firm is to decide the optimum capital structure for the
company then cost of capital should be minimized and the value of the firm should be
maximized.
3. Use in comparative analysis of various sources of finance: Cost of capital is used in
comparative analysis of various alternative sources of finance. Which source should be
chosen, it can be determined on the basis of cost of capital.
4. Use in evaluation of financial efficiency of top management: Cost of capital is used in
evaluation of financial efficiency of top management. The top management prepares
investment budgets for various projects. In these budgets future profitability and costs are
estimated. After implementation of the project, by comparison of estimated and actual costs,
it can be seen how much the top management has been successful in preparation of budgets
and implementation of projects. Ability to generate profit is no longer a test of profit
adequacy. Ability to generate Economic Value Added (EVA™) is the only test of adequacy.
“Any surplus generated from operating activities over and above the cost of capital is termed
as Economic Value Added”. EVA™ is defined as— “Excess Profits of a firm after charging Cost
of Capital”.
5. Use in other areas: Cost of capital is used in dividend policy and working capital policy
decisions also.

Book value weights or market value weights


Weight is the proportion of particular long-term fund in total long-term funds. Weights are of two
types—
Chapter 7, Cost of Capital: 51
1. Book value weights—Book value weights are proportions of market value of particular
long-term fund in total book value of long-term funds.
2. Market value weights—Market value weights are proportions of market value of particular
long-term fund in total market value of long-term funds.

While calculating 𝑘𝑂 the market value weights should be given priority due to following reasons—
1. Market value represents the approximate actual amount that can be received from the issue
of particular security.
2. Market value weights are based on market value and should be used because fresh capital is
always issued at market price.
3. Market value is very close to the real value of the fund and whereas book value is a historical
value.
4. Market value is relevant whereas book value which is historical is irrelevant.

Q. 6.: What are implicit costs and how are they relevant in calculating
weighted average cost of capital?

Answer:

Implicit cost of capital


Implicit cost of capital is the opportunity cost of capital i.e., the benefit that could have been earned
if the funds were used elsewhere instead of using in the business. E.g., a business purchased a plant
and machinery of ₹5,00,000 for production. Company could earn ₹50,000 per annum as interest if
these funds were invested in the bank. But in order to get the benefit from the machinery the
company is sacrificing the benefit of the interest. So, this sacrificed cost is called implicit cost.

Relevance of implicit cost in calculating weighted average cost of capital


Implicit cost of capital is relevant while calculating weighted average cost of capital. Cost of equity
(𝑘𝑒 ) and cost of retained earnings (𝑘𝑟 ) are part of the weighted average cost of capital. When
company does not pay any dividend on the equity shares then the calculation of cost of equity is
based on the implicit cost of capital i.e., the future dividend which is not paid but is to be paid in
future. Formula for the calculation of the cost of equity is—
𝐷1
𝑘𝑒 = +𝑔
𝑃0
Where, 𝐷1 is the expected dividend, 𝑃0 is the current market price of the equity share and 𝑔 is the
growth rate.
Further, we use implicit cost while calculating cost of retained earnings. It may appear that retained
earnings carry no cost since they represent funds which have not been raised from outside. Further
there are no cash outflows associated with retained earnings in the form of dividend or interest. The
contention that retained earnings are free of cost, however, is not correct. On the contrary, they do
involve cost (explicit) like any other source. There is implicit cost of retained earnings. The cost of
retained earnings must, therefore, be viewed as the opportunity cost of the foregone
dividends to the existing shareholders.
The cost of retained earnings is closely related to the equity shares. If the earnings are not retained,
they will be paid out to the equity shareholders as dividends.
So, form the above discussion it is clear that implicit is relevant in calculating the weighted average
cost of capital.

Chapter 7, Cost of Capital: 52


Q. 7.: Does a firm’s tax rate affects its cost of capital? What is the
effect of floatation costs associated with a new security issue on the
firm’s cost of capital?
Answer:

Firm’s tax rate and its effect on cost of capital


Yes, firm’s tax rate affects its cost of capital. The tax rate on income of the firm reduces the cost of
debt and dividend distribution tax increases the cost of preference share capital (we assume that
there is no dividend distribution tax on equity dividend). Following example will clear the
concept—
Effect on cost of debt (𝑘𝑑 )
Present cost of
debt when tax rate Cost of debt when Cost of debt when
Particulars is NIL tax rate is 20% tax rate is 40%
𝐼(1 − 𝑡)
𝑘𝑑 = 500(1 − 0) 500(1 − 0.20) 500(1 − 0.40)
𝑆𝑉 𝑘𝑑 = 𝑘𝑑 = 𝑘𝑑 =
𝐼 = 𝐼𝑛𝑡𝑒𝑟𝑒𝑠𝑡 = ₹500 5,000 5,000 5,000
𝑡 = 𝑇𝑎𝑥 𝑟𝑎𝑡𝑒 ⇒ 0.10 ⇒ 0.08 ⇒ 0.06
𝑆𝑉 = 𝑆𝑎𝑙𝑒𝑠 𝑣𝑎𝑙𝑢𝑒 = ₹5,000
From the above example it is clear that the firm’s tax rate reduces the cost of debt. It’s because
the interest on debt is deductible for income tax purposes and thus reduce the interest liability.

Effect on cost of preference share capital (𝑘𝑝 )


Present cost of Present cost of Present cost of
preference capital preference capital preference capital
when dividend when dividend when dividend
distribution tax distribution tax distribution tax
Particulars rate is NIL rate is 10% rate is 20%
𝐷𝑝 (1 + 𝐷𝑡 )
𝑘𝑝 = 500(1 + 0) 500(1 + 0.10) 500(1 + 0.20)
𝑆𝑉 𝑘𝑝 = 𝑘𝑑 = 𝑘𝑑 =
𝐷𝑝 = 𝑃𝑟𝑒𝑓𝑒𝑟𝑒𝑛𝑐𝑒 𝑑𝑖𝑣𝑖𝑑𝑒𝑛𝑑 = ₹500 5,000 5,000 5,000
𝐷𝑡 = 𝐷𝑖𝑣𝑖𝑑𝑒𝑛𝑑 𝑑𝑖𝑠𝑡𝑟𝑖𝑏𝑢𝑡𝑖𝑜𝑛 𝑡𝑎𝑥 𝑟𝑎𝑡𝑒 ⇒ 0.10 ⇒ 0.11 ⇒ 0.12
𝑆𝑉 = 𝑆𝑎𝑙𝑒𝑠 𝑣𝑎𝑙𝑢𝑒 = ₹5,000
From the above example it is clear that the dividend distribution tax rate increases the cost of
preference share capital. It’s because the dividend distribution tax rate increases the outflows.

Effect on weighted average cost of capital (WACC) or overall cost of capital (𝑘𝑂 )
What will be the effect of firm’s tax rate on overall cost of capital, it depends on—
(i) the rate of income tax; and
(ii) Rate of dividend distribution tax.
Different rates of tax will lead to different effects on the overall cost of capital.

Effect of floatation cost associated with a new security issue on firm’s


cost of capital
Floatation cost associated with a new security issue increases the firm’s cost of capital. Let us
explain this with the help of 𝑘𝑑 and 𝑘𝑒 —
Issue of debt 15% ₹5,00,000; Issue of 10,000 equity shares at ₹100 each; Expected dividend is
₹15 per share; Growth rate is 6%; Tax rate is 40%
Floatation cost=NIL Floatation cost=2% Floatation cost=5%
𝑃0 (𝑃𝑟𝑖𝑐𝑒 𝑜𝑓 𝑒𝑞𝑢𝑖𝑡𝑦 𝑠ℎ𝑎𝑟𝑒)
𝑃0 = 100 − 0
= 𝐼𝑠𝑠𝑢𝑒 𝑝𝑟𝑖𝑐𝑒 𝑃0 = 100 − 2 ⇒ ₹98 𝑃0 = 100 − 5 ⇒ ₹95
⇒ ₹100
− 𝐹𝑙𝑜𝑎𝑡𝑎𝑡𝑖𝑜𝑛 𝑐𝑜𝑠𝑡

Chapter 7, Cost of Capital: 53


𝑆𝑉 𝑆𝑉
𝑆𝑉 𝑜𝑓 𝑑𝑒𝑏𝑡
𝑆𝑉 = ₹5,00,000 − ₹0 = ₹5,00,000 = ₹5,00,000
= ₹5,00,000
⇒ ₹5,00,000 − ₹10,000 − ₹25,000
− 𝐹𝑙𝑜𝑎𝑡𝑎𝑡𝑖𝑜𝑛 𝑐𝑜𝑠𝑡
⇒ ₹4,90,000 ⇒ ₹4,75,000
𝑘𝑑 𝑘𝑑 𝑘𝑑
𝐼(1 − 𝑡) 75,000(1 − 0.40) 75,000(1 − 0.40) 75,000(1 − 0.40)
𝑘𝑑 = = = =
𝑆𝑉 5,00,000 4,90,000 4,75,000
⇒ 0.09 ⇒ 0.09184 ⇒ 0.09474

𝐷1 15 15 15
𝑘𝑒 = +𝑔 𝑘𝑒 = + 0.06 𝑘𝑒 = + 0.06 𝑘𝑒 = +6
𝑃0 100 98 95
⇒ 0.21 ⇒ 0.2131 ⇒ 0.2179
𝑘𝑂 𝑘𝑂
𝑘𝑂 1 1
𝑘𝑂 = [𝑘𝑑 × 𝑤𝑑 ] 1 = [0.09184 × ] = [0.09474 × ]
+ [𝑘𝑒 = [0.09 × ] 3 3
3 2 2
× 𝑤𝑒 ] 2 + [0.2131 × ] + [0.2179 × ]
+ [0.21 × ] ⇒ 0.17 3 3
3 ⇒ 0.1727 ⇒ 0.1768
In the above example it can be seen that the floatation cost has increased the 𝑘𝑒 , 𝑘𝑑 and finally 𝑘𝑂 .
So, it can be concluded that floatation cost associated with a new security issue increases the firm’s
cost of capital.

Q. 8.: State the different approaches to the computation of cost of


equity capital.

Answer:
There are 5 approaches/methods to compute the equity share capital. These are—
1. Dividend Price Model
2. Dividend Price Growth Model
3. Price Earning Method
4. Capital Asset Pricing Model (CAPM)
5. Realized Yield Approach

1. Dividend Price Model


𝐷1 𝐷1
𝑘𝑒 = ⇒ 𝑃0 =
𝑃0 𝑘𝑒
Where,
𝐷1 = Dividend to be paid on equity shares or expected dividend
𝑃0 = Net proceeds from issue of an equity share after adjusting discount/premium,
floatation cost, etc. of the share. If market price is given then use market price only.

2. Dividend Price Growth Model


There are 3 assumptions in this model—
4. 𝐷0 > 0.
5. Dividend payout ratio is constant.
6. 𝑘𝑒 > 𝑔
Model is as follows—
𝐷1 𝐷1
𝑘𝑒 = + 𝑔 ⇒ 𝑃0 =
𝑃0 𝑘𝑒 − 𝑔
Where,
𝑃0 = Net proceeds from issue of an equity share after adjusting discount/premium,
floatation cost, etc. of the share. If market price is given then use market price only.
In case of existing equity there is no need to subtract the floatation cost.

Chapter 7, Cost of Capital: 54


𝐷1 = Dividend to be paid on equity shares or expected dividend
𝐷0 = Dividend last paid OR Dividend paid last year

3. Price Earning Method


𝐸𝑃𝑆 1
𝑘𝑒 = ⇒ 𝑘𝑒 =
𝑃0 𝑜𝑟 𝑀𝑃 𝑃/𝐸 𝑅𝑎𝑡𝑖𝑜
Where,
𝐸𝑃𝑆 = Earning per shares
𝑃0 = Net proceeds of an equity share
𝑀𝑃 = Market price of the share
In case of existing equity there is no need to subtract the floatation cost.

4. Capital Assets Pricing Model (CAPM) or CAP Model


𝑘𝑒 = 𝑅𝑓 + 𝛽(𝐾𝑚 − 𝑅𝑓 )
Where,
𝑅𝑓 = Risk free rate of return i.e., rate of return on government securities
𝐾𝑚 = Rate of return on market portfolio
𝛽 = Beta is the measurement of systematic risk

5. Realized yield approach


Under this approach we calculate the IRR. Price paid to purchase the share is taken as cash
outflows. Dividends received and selling price of the share are cash inflows.

Q. 9.: “Cost of existing share capital and fresh issue of capital is


always same”. Do you agree? Give reasons.

Answer:
Assumption: Share capital means equity share capital.
There is a fallacy in the statement— “Cost of existing share capital and fresh issue of capital is
always same”. The statement is far away from the practicality. In reality there are no different
categories of the equity share capital. When company issues fresh share capital then it cannot
differentiate the new shares with that of the existing shares. It is because of the provisions of the
law in force.

If theoretically it is assumed that there are different categories of the shares then it is difficult to
comment whether the cost of existing share capital and fresh issue of capital is always same or not.
Why? There are too many factors which affect the cost of share capital. For instance, while issuing
fresh share capital it might be possible that company is in need of funds. In such a case fresh issue
increases the supply of shares in the market and at the same time it may leave an impression on the
existing shareholders that company is running short of internal funds. Now the existing
shareholders may expect more dividends to compensate themselves and due to the fear of dilution
of control. This in turn may increase the overall cost of share capital.

In another case when company has sufficient funds and still issues fresh share capital, it may leave
an impression on the existing shareholders that company needs more funds to invest in new
projects and after some years company will be in a position to pay more dividends. Also, the
existing shareholders may opine that this step of the company may result in the increased market
price of the share after some time, so, they do not expect more dividends because they may be
benefitted by the capital gains rather than dividends. Now, because existing shareholders do not
prefer dividends, so, it may not affect the overall cost of share capital.

A practicality which is involved in the issue of fresh share capital is that it increases the number of
shares in the market and increased number of shares in the market may affect the cost of share
capital in two ways—it may increase the cost of share capital and it may not increase the cost of
share capital. Due to increased supply of the shares in the market there may be fear of dilution of
Chapter 7, Cost of Capital: 55
the EPS provided there is no change in the share price. This dilution of earnings may result in the
higher expectations of the shareholders and thus may increase the cost of share capital. Another
case in which, the increased supply of the shares in the market may decrease the market price of
shares and then after sometime due to speculation in the market and due to optimum utilization of
fresh share capital the price of the shares may increase and this may result in the decreased cost of
share capital provided there is no change in the EPS. But EPS may increase due to the optimum
utilization of fresh share capital which results in the higher profits than before and this in turn may
decrease the cost of share capital.

Conclusion: From the above discussion it is clear that theoretically it is difficult to comment on
whether cost of existing share capital and fresh issue of capital is always same, but in reality,
whenever a company issues fresh share capital the price is decided by the forces of supply and
demand. Also, investors buy these shares not for the dividend which they may receive, but for the
capital gain they may earn after some time by selling their holdings in the market. EPS is not
important for these investors. It may result in decreased cost of share capital. Also, there are cases
when company issues fresh share capital to the institutional investors who buy this fresh issue in
the hope of dividends as well as capital gains. We have excluded that case from the discussion.

Note: If it assumed that the examiner is referring the floatation cost of the existing share capital and
fresh issue of share capital then it can be concluded that the floatation cost is never same for the
existing share capital and fresh issue of share capital.

Q. 10.: “As there is no explicit cost of retained earnings, these funds


are free of cost”. Critically comment.

Answer:
It may appear that retained earnings carry no cost since they represent funds which have not been
raised from outside. Further there are no cash outflows associated with retained earnings in the
form of dividend or interest. The contention that retained earnings are free of cost, however, is not
correct. On the contrary, they do involve cost like any other source.
The cost of retained earnings is closely related to the equity shares. If the earnings are not retained,
they will be paid out to the equity shareholders as dividends. Retained earnings are often looked as
fully subscribed issue of new equity, since they increase the shareholder’s equity in the same way
that a new issue of equity shares would. There is implicit cost of retained earnings. The cost of
retained earnings must, therefore, be viewed as the opportunity cost of the foregone
dividends to the existing shareholders.
Cost of retained can also be computed with the help of above methods namely—
1. Dividend Model
2. Dividend Growth Model
3. Price Earning Method
4. Capital Asset Pricing Model (CAPM)
Cost of retained earnings can also be computed using the following formula:
𝑘𝑟 = 𝑘𝑒 (1 − 𝑡)(1 − 𝑏)(1 − 𝑓)
Where,
𝑡 = Tax rate on income of shareholders
𝑏 = Brokerage rate to be paid by the shareholder for making new investments
𝑓 = Floatation cost on issue of new share capital
Note: Floatation costs shall not be subtracted from market price or issue price or sales value while
calculating cost of retained earnings.

Q. 11.: Other things being equal, how the following events would
affect companies weighted average cost of capital—(i) The company

Chapter 7, Cost of Capital: 56


repays its long-term debt; (ii) The corporate income tax rate is
increased; (iii) Floatation cost of new issue increased.

Answer:

(i) The company repays its long-term debt


When a company repays its long-term debt then this event increases the weighted average cost of
capital. It is because the interest on debt is deductible for tax purposes. Further, the actual interest
liability is less than the interest outflows due to the savings in taxes. When the company repays the
long-term debt then the savings in taxes is no more available. Hence the weighted average cost of
capital increases. Let us explain this with the help of an example—

Example
Long-term debt ₹5,00,000 bearing interest rate of 15%. Equity capital is ₹10,00,000. Tax rate is
40%. 𝑘𝑒 is 20% and 𝑘𝑑 is 9% (𝑖. 𝑒. 15%(1 − 0.40)).
WACC before repayment:
5,00,000 10,00,000
𝑘𝑂 = [𝑘𝑑 × 𝑤𝑑 ] + [𝑘𝑒 × 𝑤𝑒 ] ⇒ 𝑘𝑂 = [0.09 × ] + [0.20 × ] ⇒ 0.1633
15,00,000 15,00,000
WACC after repayment:
0 10,00,000
𝑘𝑂 = [𝑘𝑑 × 𝑤𝑑 ] + [𝑘𝑒 × 𝑤𝑒 ] ⇒ 𝑘𝑂 = [0.09 × ] + [0.20 × ] ⇒ 0.20
10,00,000 10,00,000
Conclusion: So, from the above example it is clear that the repayment of long-term debt increases
the weighted average cost of capital provided other things being equal.

(ii) The corporate income tax rate is increased


The increase in the corporate income tax rate decreases the weighted average cost of capital. This is
because, the increase in the corporate tax rate will reduce the cost of debt (𝑘𝑑 ). Let us explain this
with the help of an example—

Example
Long-term debt ₹5,00,000 bearing interest rate of 15%. Equity capital is ₹10,00,000. Tax rate is
40% and 50%. 𝑘𝑒 is 20%; 𝑘𝑑 is 9% (𝑖. 𝑒. 15%(1 − 0.40)) when tax rate is 40% and 𝑘𝑑 is 7.5%
(𝑖. 𝑒. 15%(1 − 0.50)) when tax rate is 50%.
WACC when tax rate is 40%:
5,00,000 10,00,000
𝑘𝑂 = [𝑘𝑑 × 𝑤𝑑 ] + [𝑘𝑒 × 𝑤𝑒 ] ⇒ 𝑘𝑂 = [0.09 × ] + [0.20 × ] ⇒ 0.1633
15,00,000 15,00,000
WACC when tax rate is 50%:
5,00,000 10,00,000
𝑘𝑂 = [𝑘𝑑 × 𝑤𝑑 ] + [𝑘𝑒 × 𝑤𝑒 ] ⇒ 𝑘𝑂 = [0.075 × ] + [0.20 × ] ⇒ 0.1583
15,00,000 15,00,000
Conclusion: So, from the above example it is clear that increase in corporate tax rate decreases the
weighted average cost of capital provided other things being equal.

(iii) Floatation cost of new issue increased


The increase in the floatation cost of new issue increases the weighted average cost of capital. This
is because, the floatation cost first increases the specific cost of capital such as 𝑘𝑑 or 𝑘𝑝 or 𝑘𝑒 and in
turn increases the 𝑘𝑂 . Let us explain this with the help of an example—
Issue of debt 15% ₹5,00,000; Issue of 10,000 equity shares at ₹100 each; Expected dividend is
₹15 per share; Growth rate is 6%; Tax rate is 40%
Floatation cost=NIL Floatation cost=2% Floatation cost=5%
𝑃0 (𝑃𝑟𝑖𝑐𝑒 𝑜𝑓 𝑒𝑞𝑢𝑖𝑡𝑦 𝑠ℎ𝑎𝑟𝑒)
𝑃0 = 100 − 0
= 𝐼𝑠𝑠𝑢𝑒 𝑝𝑟𝑖𝑐𝑒 𝑃0 = 100 − 2 ⇒ ₹98 𝑃0 = 100 − 5 ⇒ ₹95
⇒ ₹100
− 𝐹𝑙𝑜𝑎𝑡𝑎𝑡𝑖𝑜𝑛 𝑐𝑜𝑠𝑡

Chapter 7, Cost of Capital: 57


𝑆𝑉 𝑆𝑉
𝑆𝑉 𝑜𝑓 𝑑𝑒𝑏𝑡
𝑆𝑉 = ₹5,00,000 − ₹0 = ₹5,00,000 = ₹5,00,000
= ₹5,00,000
⇒ ₹5,00,000 − ₹10,000 − ₹25,000
− 𝐹𝑙𝑜𝑎𝑡𝑎𝑡𝑖𝑜𝑛 𝑐𝑜𝑠𝑡
⇒ ₹4,90,000 ⇒ ₹4,75,000
𝑘𝑑 𝑘𝑑 𝑘𝑑
𝐼(1 − 𝑡) 75,000(1 − 0.40) 75,000(1 − 0.40) 75,000(1 − 0.40)
𝑘𝑑 = = = =
𝑠𝑣 5,00,000 4,90,000 4,75,000
⇒ 0.09 ⇒ 0.09184 ⇒ 0.09474

𝐷1 15 15
15 𝑘𝑒 = +6 𝑘𝑒 = +6
𝑘𝑒 = +𝑔 𝑘𝑒 = + 6 ⇒ 0.21 98 95
𝑃0 100 ⇒ 0.2131 ⇒ 0.2179
𝑘𝑂 𝑘𝑂
𝑘𝑂 1 1
𝑘𝑂 = [𝑘𝑑 × 𝑤𝑑 ] 1 = [0.09184 × ] = [0.09474 × ]
+ [𝑘𝑒 = [0.09 × ] 3 3
3 2 2
× 𝑤𝑒 ] 2 + [0.2131 × ] + [0.2179 × ]
+ [0.21 × ] ⇒ 0.17 3 3
3 ⇒ 0.1727 ⇒ 0.1768
Conclusion: From the above example it is clear that floatation cost of new issue increases the
weighted average cost of capital provided other things being equal.

Q. 12.: “New issue of capital is costlier than retained earnings”. How


and what makes these two to differ?

Answer:
“New issue of capital is costlier than retained earnings”. We agree with this statement. The reason
is—there is floatation cost associated with the new issue of capital but not with retained earnings.
We know that 𝑘𝑒 = 𝑘𝑟 , but floatation cost increases the cost of new issue of capital as compared to
retained earnings. So, floatation cost is an important factor which is responsible for the
difference in the new cost of new issue of capital and cost of retained earnings. Let us explain
this with the help of an example—

Example
Raja Ltd. is planning to issue new equity shares (face value ₹10 per share). The dividend declared
by the company is ₹13.60 per share. Floatation cost may be 4% of the selling price. If the current
market price is ₹130 per share and growth rate is 8% then calculate cost of existing equity. Retained
earnings and new equity.

Solution
𝐷1 13.60(1 + .08)
𝑘𝑒 (𝐸𝑥𝑖𝑠𝑡𝑖𝑛𝑔) = + 𝑔 ⇒ 𝑘𝑒 = + 0.08 = 0.1930
𝑃0 130

𝑘𝑟 = 𝑘𝑒 (𝐸𝑥𝑖𝑠𝑡𝑖𝑛𝑔) ⇒ 𝑘𝑟 = 0.1930

𝐷1 13.60(1 + .08)
𝑘𝑒 (𝑁𝑒𝑤) = + 𝑔 ⇒ 𝑘𝑒 = + 0.08 = 0.1977
𝑃0 130(1 − .04)
Conclusion: From the above example it is clear that the new issue of capital is costlier than retained
earnings.

Chapter 7, Cost of Capital: 58

You might also like